Burns Pediatric Final Exam Study Guide

Réussis tes devoirs et examens dès maintenant avec Quizwiz!

1. 2. The primary care pediatric nurse practitioner sees a child for follow-up care after hospitalization for ARF. The child has polyarthritis but no cardiac involvement. What will the nurse practitioner teach the family about ongoing care for this child? a. a. Aspirin is given for 2 weeks and then tapered to discontinue the medication. b. b. Prophylactic amoxicillin will need to be given for 5 years. c. c. Steroids will be necessary to prevent development of heart disease. d. d. The child will need complete bedrest until all symptoms subside.

ANS: A ASA is given for arthritis for 2 weeks and then will be tapered. Children with ARF will need penicillin prophylaxis, not amoxicillin. Steroids are sometimes used for symptomatic relief but do not prevent chronic heart disease. Bed rest is indicated only when cardiac symptoms occur.

12. The parent of a 16-year-old tells the primary care pediatric nurse practitioner that the teen was recently caught smoking an electronic cigarette (e-cigarette). What will the nurse practitioner tell this parent? a. E-cigarette use may be a risk factor for later substance abuse. b. Experimentation with e-cigarettes does not lead to future tobacco use. c. Most teens who experiment with tobacco usually do not become addicted. d. This form of nicotine ingestion is safer than regular cigarettes.

ANS: A Although many adolescents consider e-cigarettes to be a safe form of tobacco use, increasing evidence indicates that their use may be a significant risk factor for later marijuana and substance abuse. The risk of dependence and addiction is the same for e-cigarettes and other cigarettes, since both use nicotine. Only 41% of teens try tobacco

3. A child is diagnosed with community-acquired pneumonia and will be treated as an outpatient. Which antibiotic will the primary care pediatric nurse practitioner prescribe? a. Amoxicillin b. Azithromycin c. Ceftriaxone d. Oseltamivir

ANS: A Amoxicillin is given to children with community-acquired pneumonia. Azithromycin is used to treat atypical pneumonia. Ceftriaxone is used for inpatient treatment. Oseltamivir is used for viral pneumonia.

1. 5. While the primary care pediatric nurse practitioner is discussing anticipatory guidance with the mother of a 12 month old, the child repeatedly pulls objects out of the mother's purse. Each time, the mother slaps the child's hands as she takes the objects away. What will the nurse practitioner recommend to help the mother manage this child's misbehavior in a developmentally appropriate manner? a. a. Keep her purse up high and out of the child's reach. b. b. Place acceptable objects in her purse for the child to find. c. c. Say "No!" instead of slapping the child's hands. d. d. Use timeout each time the child gets into the purse.

ANS: A Parents should provide a developmentally appropriate environment to minimize children's misbehavior. Children at this age who are naturally curious will explore the environment and will seek out objects within their reach. It is easier to put the purse up high than to have to repeatedly say "No." Putting acceptable objects in her purse only reinforces the undesired behavior.

9. The primary care pediatric nurse practitioner is managing care for a child diagnosed with iron- deficiency anemia who had an initial hemoglobin of 8.8 g/dL and hematocrit of 32% who has been receiving ferrous sulfate as 3 mg/kg/day of elemental iron for 4 weeks. The child's current lab work reveals elevations in Hgb/Hct and reticulocytes with a hemoglobin of 10.5 g/dL and a hematocrit of 36%. What is the next step in management of this patient? a. Continue the current dose of ferrous sulfate and recheck labs in 1 to 2 months. b. Discontinue the supplemental iron and encourage an iron-enriched diet. c. Increase the ferrous sulfate dose to 4 to 6 mg/kg/day of elemental iron. d. Refer the child to a pediatric hematologist to further evaluate the anemia.

ANS: A This child has mild to moderate anemia and is showing a good response to the current dose of iron. Ferrous sulfate should be continued for at least 2 to 3 months to normalize hemoglobin, and then continue for 2 to 4 months to replace depleted iron stores. There is no need to increase the dose, since the child is responding appropriately to the current dose. Children with hemoglobin levels less than 4 g/dL should be referred.

11. A preschool-age child is seen in the clinic after waking up a temperature of 102.2°F, swelling and erythema of the upper lid of one eye, and moderate pain when looking from side to side. Which course of treatment is correct? a. Admit to the hospital for intravenous antibiotics. b. Obtain a lumbar puncture and blood culture. c. Order warm compresses 4 times daily for 5 days. d. Prescribe a 10- to 14-day course of oral antibiotics.

ANS: A This child has periorbital cellulitis and must be hospitalized because of having pain with movement of the eye, indicating orbital involvement. LP is performed on infants under 1 year of age. Warm compresses are used for mild cases. Oral antibiotics are not indicated.

13. A 14-year-old female has menometrorrhagia with moderate increase in menstrual flow and irregular periods. Her hemoglobin is 13.1 g/dL. How will this be managed? a. Iron supplementation and prostaglandin inhibitors b. One OCP twice daily for 3 to 4 days and then daily c. Progestin every day for 10 to 14 days d. Referral to a pediatric gynecologist for treatment

ANS: A This patient has mild AUB and may be managed by observation and reassurance along with iron to prevent anemia and prostaglandin inhibitors to reduce heavy bleeding. Patients with moderate AUB may be prescribed OCPs or progestins. Referral to a gynecologist is warranted with severe AUB.

7. A young adolescent female is observed to have mild unequal scapula prominences on gross examination while standing. In the Adams forward bending position, this inequality disappears. What will the primary care pediatric nurse practitioner do? a. Discuss posture and exercise and ask about backpacks and books. b. Obtain radiographic studies of the entire spine and neck. c. Reassure the child's parent that functional scoliosis will self-resolve. d. Refer to an orthopedic specialist for evaluation and possible bracing.

ANS: A Unequal scapulas noted on standing can denote scoliosis and deserves a more thorough physical assessment. Functional scoliosis can be diagnosed by assessing curves in the spine in the Adams forward bending position. Although it is relatively benign, functional scoliosis can progress to structural scoliosis if not treated, which can be done with physical therapy or other means, such as exercises or removing external forces (carrying heavy loads, heavy one-sided backpacks) that place unequal pressures on the spine, to prevent progression. Radiographs may be necessary if this worsens. This will not self-resolve but must be managed to prevent progression. Orthopedic referral is not necessary at this early, modifiable stage.

5. A child who has had a single non-febrile seizure has a normal neurologic exam. Which diagnostic test is indicated? a. Computerized tomography (CT) b. Electroencephalogram (EEG) c. Magnetic resonance imaging (MRI) d. Polysomnography

ANS: B An EEG is standard for all children after a first non-febrile seizure. CT is not routinely used because of radiation exposure. MRI is used if cognitive changes or postictal focal dysfunction persists, if the seizure lasts longer than 15 minutes, if the child is younger than 6 months of age, and if any new onset of focal neurologic deficit has occurred. Polysomnography is used to assess nocturnal seizures.

8. During a well child exam on a 5-year-old child, the primary care pediatric nurse practitioner assesses the child for school readiness. Which finding may be a factor in limiting school readiness for this child? a. Adherence to daily family routines and regular activities b. Having two older siblings who attend the same school c. Parental concerns about bullying in the school d. The child's ability to recognize four different colors

ANS: C Parental expectation is the strongest predictor of school success in children. Parents who are worried about what may happen in school can transmit this anxiety to the child. Children who have a secure family life with daily routines will do better in school. Having older siblings who attend school increases success. Children at this age are expected to know four colors, so this is an indication of school readiness.

12. During a routine well child exam on a 5-year-old child, the primary care pediatric nurse practitioner auscultates a grade II/VI, harsh, late systolic ejection murmur at the upper left sternal border that transmits to both lung fields. The child has normal growth and development. What will the nurse practitioner suspect? a. Aortic stenosis b. Patent ductus arteriosus c. Pulmonic stenosis d. Tricuspid atresia

ANS: C Pulmonic stenosis may be asymptomatic with a murmur as described above. Aortic stenosis is characterized by a louder, harsh systolic crescendo-decrescendo murmur at the upper right sternal border with radiation to the neck, LLSB, and apex. PDA has a machinery-like murmur. Tricuspid atresia is characterized by cyanosis.

1. The primary care pediatric nurse practitioner understands that a major child health outcome associated with worldwide climate change is : a. cost of living. b. education. c. nutrition. d. pollution.

ANS: C There is growing evidence that climate change is having a dramatic effect on food crops that leads to food distribution issues and food insecurity among families.

4. A 4-year-old child who has had extensive dental surgery to treat dental caries has white spot lesions on the primary teeth. How often should this child receive fluoride varnish applications? a. Annually b. Twice yearly c. Every 3 to 6 months d. Every month

ANS: C This child is high risk and should receive fluoride varnish treatments every 3 to 6 months. Annual or twice yearly applications are for lower risk children. It is not necessary to apply fluoride varnish every month.

6. The primary care pediatric nurse practitioner is examining a young child who was brought in by a grandmother for evaluation of a partial-thickness burn on one arm. The PNP suspects that this is an intentional injury, but the grandmother states that the parents are "just careless" and that the child is now living with her. What will the PNP do? a. Flag this as a concerning incident in the child's record. b. Reassure the grandmother that she is doing the right thing. c. Refer the child's parents to a parenting resource center. d. Report a suspicion of abuse to child protective services.

ANS: D All states have mandatory reporting laws that require health care professionals to report suspected or known abuse to appropriate agencies and provide both civil and criminal immunity to mandated reporters. The other options may be necessary once the case is investigated, but the priority is to report the suspicion of abuse.

10. A toddler exhibits exotropia of the right eye during a cover-uncover screen. The primary care pediatric nurse practitioner will refer to a pediatric ophthalmologist to initiate which treatment? a. Botulinum toxin injection b. Corrective lenses c. Occluding the affected eye for 6 hours per day d. Patching of the unaffected eye for 2 hours each day

ANS: D Deviations are initially treated by patching the unaffected eye for 2 hours each day to force the affected eye to move correctly. Botulinum toxin injection may be used with some deviations but is not a first-line therapy. Corrective lenses alone improve amblyopia in 27% of patients. The unaffected eye is patched; 2 hours per day is as effective as 6 hours per day.

6. The primary care pediatric nurse practitioner learns that the mother of a 3-year-old child has been treated for depression for over 5 years. Which aspect of this child's development will be of the most concern to the nurse practitioner? a. Fine motor b. Gross motor c. Social/emotional d. Speech and language

ANS: D Maternal depression in the first year of life has been associated with poorer language development at 3 years of age.

4. A healthy 14-year-old female has a dipstick urinalysis that is positive for 5-6 RBCs per hpf but otherwise normal. What is the first question the primary care pediatric nurse practitioner will ask this patient? a. "Are you sexually active?" b. "Are you taking any medications?" c. "Have you had a recent fever?" d. "When was your last menstrual period (LMP)?"

ANS: D Menstrual blood may appear in urine and is a common cause of urine with RBCs present, so this would be an appropriate first question of an adolescent. Asking about sexual activity or recent fevers may be part of the diagnostic reasoning if common causes are not present. Medications may discolor the urine but do not cause RBCs to be present.

18-month-old child through which assessment strategy? a. Asking the child to tell a story using dolls and other props b. Asking the child to draw a picture of him- or herself and other family members c. Interviewing the child separately from caretakers and parents d. Observation of the child with caretakers in structured and unstructured situations

ANS: D The PNP can observe infants and toddlers with caretakers in both structured and unstructured situations to determine strengths and limitations of each partner in the interaction. Asking children to tell stories is more appropriate for older toddlers and preschoolers. Preschoolers may be asked to draw pictures. Interviewing children separately from caretakers is more appropriate with school-age children and adolescents.

3. A 3-year-old child who attends day care has had a fever, nausea, and vomiting several weeks prior and now has darkened urine and constipation along with hepatomegaly and right upper quadrant tenderness. What treatment is warranted for this child? a. HAV vaccine b. Immunoglobulin G c. Interferon-alfa d. Supportive care

ANS: D The child has symptoms consistent with hepatitis A virus. HAV vaccine and IgG may be given within 2 weeks of exposure

1. 10. The primary care pediatric nurse practitioner is counseling the mother of a newborn about breastfeeding her infant. Which supplements will the nurse practitioner recommend? a. a. Fat-soluble vitamins b. b. Iron c. c. Multivitamins with iron d. d. Vitamin D

ANS: D The level of vitamin D in breast milk may not be adequate for breastfed infants, so it should be given as a supplement. Human milk has more than adequate amounts of other vitamins. Iron is not present, but this is generally added at 6 months of age, when solid foods are added to the diet.

7. The primary care pediatric nurse practitioner is assessing a special needs school-age child whose family has just moved to the area. What is a priority concern at this initial visit? a. Asking the parents to describe the child's illness, treatments, and unique needs b. Connecting the family to local support groups, school programs, and resources c. Gathering information about financial concerns related to the child's condition d. Providing expert information about the child's condition and its management

ANS: A Parents of children with special needs often become medical experts in their child's diagnosis, management, and idiosyncratic responses and expect to be treated seriously and respectfully in order to develop a trusting alliance with their health care providers. This family has experience caring for their child, and it is important to find out what that experience is and what works well for this child. The other options may be necessary after this collaborative relationship is established and when both parents and providers agree that these interventions are useful.

1. 1. According to the 2007 NHIS analysis of the use of CAM therapy, use was higher among : a. a. families whose parent or parents had a college education. b. b. Hispanic and African-American populations. c. c. persons who had not recently taken prescription medications. d. d. those living in households earning less than $65,000.

ANS: A The NHIS review showed that CAM use was higher in households with at least one college- educated parent. CAM use was also higher in non-Hispanic Caucasians, persons who had taken prescription drugs within the past 3 months, and those living in households with incomes higher than $65,000.

11. A 15-year-old female reports fainting at school in class on two occasions. The adolescent's orthostatic blood pressures are normal. The primary care pediatric nurse practitioner suspects a cardiac cause for these episodes and will order which tests before referring her to a pediatric cardiologist? a. 12-lead electrocardiogram b. Echocardiogram c. Tilt table testing d. Treadmill exercise testing

ANS: A A 12-lead ECG is useful for initial evaluation. Echocardiogram may be performed if the ECG shows potential cardiomyopathy. Tilt table testing is not recommended for use in primary care due to poor reliability. Treadmill exercise testing may be used in cases of exercise-related syncope.

1. 3. A toddler is brought to the clinic after grabbing the hot end of his mother's curling iron. An examination reveals a pale, yellow burned area to the palm of one hand. What is true about this burn? a. a. It may take up to 3 weeks to heal with scarring likely. b. b. Scarring is unlikely, with healing expected in 3 to 7 days. c. c. Surgical intervention and skin grafting are usually required. d. d. This type of burn usually heals without scarring in 7 to 14 days.

ANS: A A deep partial-thickness burn appears pale and yellow and scarring is more likely to occur, with complete healing taking up to 3 weeks. A superficial burn is erythematous without blisters and heals in 3 to 7 days without scarring. A full-thickness burn involves extensive destruction of underlying tissues and requires surgical intervention and skin grafts. A superficial partial- thickness burn is red, mottled, moist, and painful and may scar, with healing in 7 to 14 days.

1. 13. The primary care pediatric nurse practitioner is evaluating an 11-month-old infant who has had three viral respiratory illnesses causing bronchiolitis. The child's parents both have seasonal allergies and ask whether the infant may have asthma. What will the nurse practitioner tell the parents? a. a. "Although it is likely, based on family history, it is too soon to tell." b. b. "There is little reason to suspect that your infant has asthma." c. c. "With your infant's history of bronchiolitis, asthma is very likely." d. d. "Your infant has definitive symptoms consistent with a diagnosis of asthma."

ANS: A A genetic predisposition for the development of an IgE-mediated response to aeroallergens is the strongest identifiable predisposing risk factor for asthma, but asthma is rarely diagnosed before age 12 months due to the high rate of viral-induced bronchiolitis. The PNP should be cautious about diagnosing asthma until wheezing without an association to viral illnesses occurs. This infant has clear risk factors for asthma

1. The primary care pediatric nurse practitioner is performing an examination on a 2-year-old child who has been placed in emergency foster care with a grandparent after the child's mother has been arrested for drug use. The child has a history of asthma with frequent exacerbations because of parental smoking. What is a priority for the nurse practitioner at this visit? a. Evaluation of financial resources, medical insurance, and access to health care and medications b. Providing a list of websites and community-based support groups for grandparents parenting grandchildren c. Referral to a social worker to help the child deal with emotional conflict related to separation from the parent d. Teaching the grandparent about the need for consistency in routines and discipline for the child

ANS: A A high percentage of grandparents who parent grandchildren have financial difficulties and most cannot claim grandchildren as dependents for health care. This child has a chronic disease and will need medication and possibly hospitalization, so the PNP should assess resources and access to care. The other options are important but are not a priority in the initial visit.

11. A 3-year-old child with pressure-equalizing tubes (PET) in both ears has otalgia in one ear. The primary care pediatric nurse practitioner is able to visualize the tube and does not see exudate in the ear canal and obtains a type A tympanogram. What will the nurse practitioner do? a. Order ototopical antibiotic/corticosteroid drops. b. Prescribe a prophylactic antibiotic medication. c. Reassure the parent that this is a normal exam. d. Refer the child to an otolaryngologist for follow-up

ANS: A A normal, or type A, tympanogram in a child with PET may indicate a clogged tube. Ototopical antibiotic/corticosteroid drops can occasionally clear a clogged PET. Prophylactic antibiotics are not recommended to prevent otitis media. It is not necessary to refer unless the pain continues in spite of standard measures.

1. A 10-month-old infant who is new to the clinic has chronic hepatitis B infection. What will the primary care pediatric nurse practitioner do to manage this infant's disease? a. Consult a pediatric infectious disease specialist. b. Prescribe interferon-alfa. c. Provide supportive care. d. Consider use of lamivudine.

ANS: A A specialist in hepatitis B in children should be consulted for children with chronic hepatitis B infection because of the risk for developing hepatocellular carcinoma. Interferon-alfa and lamivudine are not used in infants. Supportive care only is not recommended.

16. The primary care pediatric nurse practitioner is examining a 2-week-old infant and auscultates a wide splitting of S2 during expiration. What condition may this finding represent? a. Atrial septal defect b. Coarctation of the aorta c. Patent ductus arteriosis d. Ventricular septal defect

ANS: A A wide splitting of S2 without becoming a single sound on expiration may indicate increased pulmonary flow, typical of atrial septal defect. Coarctation of the aorta may cause a systolic ejection murmur. A patent ductus arteriosus has a characteristic machinery-like murmur. A ventricular septal defect has a harsh, high-pitched, grade II to IV/VI holosystolic murmur.

2. Adolescent children are more likely to smoke cigarettes and drink alcohol if they live with a. cohabitating parents. b. grandparents. c. homosexual parents. d. single parents.

ANS: A Adolescents living in cohabitating households are more likely to smoke and drink in comparison with adolescents living in married or single parent households. Children living with homosexual parents and children living with grandparents do not have this increased risk.

10. The primary care pediatric nurse practitioner is performing a well child exam on a 17-year-old female whose mother is present during the history. The mother expresses concern that her daughter wishes to have an eyebrow piercing and states that she is opposed to the idea. What will the nurse practitioner do? a. Provide information about piercings and encourage continued discussion. b. Remind the adolescent that her mother is responsible for her health. c. State that piercings are relatively harmless and are an expression of individuality. d. Suggest that she wait until she is 18 years old and can make her own decisions.

ANS: A Adolescents who pierce their noses or have strange haircuts may be irritating to parents, but these are ways of expressing individuality and help them to achieve psychosocial milestones. The fact that the teen and her mother are discussing this is a good sign that the adolescent isn't in complete rebellion. The PNP should provide accurate health information and encourage continued dialogue. Although it is true that piercings are relatively harmless, the PNP shouldn't "side" with the teen during an open discussion or tell the teen that the mother is "in charge."

1. 9. The primary care pediatric nurse practitioner performs a well child assessment on a 6-month-old infant whose mother reports having less breast milk because of stressors associated with pumping and returning to work. The nurse practitioner will provide resources to promote pumping and a. a. discuss adding other foods to the baby's diet. b. b. encourage the mother to increase her fluid intake. c. c. prescribe a multivitamin containing iron. d. d. suggest offering only breast milk to the infant.

ANS: A After 6 months, infants should continue to breastfeed while taking other nutrients, at least up to 1 year of age. The mother should be encouraged to continue breastfeeding while adding other nutrients. The mother has successfully breastfed up to this point and increasing fluids is not the source of the problem.

1. The primary care pediatric nurse practitioner is examining a 15-year-old female who reports having her first period at age 13. She states that she has had five periods in the last year, with the last one 2 months prior. She participates in basketball at school. Which action is correct? a. Perform biometric screening to determine lean body mass. b. Prescribe oral contraceptives pills to regulate her periods. c. Reassure her that this is perfectly normal at her age. d. Refer her to an endocrinologist for hormonal evaluation.

ANS: A Although it can take 18 to 24 months for adolescents to establish regulatory cycles, periods can also be affected by athletic activity that decreases body fat. The PNP should assess the percentage of lean body mass, which should be 75% or less to maintain regular ovulatory cycles. OCPs are useful for regulating periods if this persists and other causes are ruled out. It is not necessary to refer her to an endocrinologist unless problems persist in spite of standard management.

1. 11. The primary care pediatric nurse practitioner sees a 3-year-old child whose parents report is a picky eater in spite of their continued efforts to provide nutritious meals. The parents ask whether a multivitamin is necessary. How will the nurse practitioner respond? a. a. Ask the parents to provide a 3-day food diary. b. b. Prescribe a daily multivitamin with iron. c. c. Reinforce the need to meet DRIs each day. d. d. Tell them that supplements are unnecessary

ANS: A Although most children who are "picky eaters" eat a balanced diet over time, it is worth assessing the situation using a 3-day diet history to determine whether key nutrients are being missed and if the child needs an MV supplement. MVs are not usually necessary and iron is not given unless there is a deficiency. It is not necessary to meet DRIs every day. Supplements may be necessary after data is collected.

21. A 2-month-old infant will receive initial immunizations, and the parent asks about giving medications to increase the infant's comfort and minimize fever. What will the primary care pediatric nurse practitioner recommend? a. Administering ibuprofen or acetaminophen as needed b. Avoiding antipyretics if possible to attain better immunity c. Giving ibuprofen and acetaminophen only after the vaccines d. Pre-treating the infant with both ibuprofen and acetaminophen

ANS: A Although some studies have detected lower antibody responses in infants and children who were given antipyretics before or after routine vaccinations, the lowered response did not influence persistent immunological memory and did not cause a decrease in vaccine protection. Parents may administer acetaminophen or ibuprofen before or after vaccines as needed. There is no evidence that a combination of acetaminophen and ibuprofen is more effective.

1. 1. The mother of a 15-year-old female expresses concerns that her daughter may be sexually active because she's had a steady boyfriend for over a year. The primary care pediatric nurse practitioner learns that the family is Catholic and that the mother had an abortion when she was 16 years old. What will the nurse practitioner do initially? a. a. Explore the mother's feelings about her own past experience. b. b. Offer to prescribe contraception to prevent pregnancy. c. c. Recommend that the mother discuss this with her daughter. d. d. Suggest that the mother talk to a priest about her daughter.

ANS: A An initial step when assisting families to manage ethical and behavioral issues is to assist parents and children in values clarification. Before offering other suggestions, the PNP should help the mother to clarify her own feelings. The other options may be necessary after values are clarified.

12. A child who has psoriasis, who has been using a moderate-potency topical steroid on thick plaques on the extremities and a high-potency topical steroid on more severe plaques on the elbows and knees, continues to have worsening of plaques. In consultation with a dermatologist, which treatment will be added? a. Anthralin ointment in high strength applied for 10 to 30 minutes daily b. Calcipotriol cream applied liberally each day to the entire body c. Oral steroids and methotrexate therapy until plaques resolve d. Wideband ultraviolet therapy for 15 minutes twice daily

ANS: A Anthralin ointment is useful for plaques that are resistant to steroids. Calcipotriol cream is effective for mild to moderate plaques, but when applied in excessive quantities over large areas can cause hypercalcemia. Oral steroids are not indicated and may worsen symptoms by causing pustular flare. Methotrexate is used for severe disease, and these symptoms indicate that this is moderate disease. If UV light is used, narrowband UVB light therapy is preferred in children for safety and efficacy.

1. 15. A 12-year-old child is brought to the clinic with joint pain, a 3-week history of low- grade fever, and a facial rash. The primary care pediatric nurse practitioner palpates an enlarged liver 2 cm below the subcostal margin along with diffuse lymphadenopathy. An ANA test is positive. Which test may be ordered to confirm a diagnosis of SLE? a. a. Anti-double-strand DNA antibodies b. b. Anti-La antibodies c. c. Anti-Ro antibodies d. d. Anti-Sm antibodies

ANS: A Anti-double-strand DNA antibodies are present in most people with SLE and are generally exclusively seen in cases of SLE and not other diseases. Anti-SM antibodies are diagnostic of SLE but are only seen in 30% of patients with SLE.

5. The mother of a 15-year-old adolescent female tells the primary care pediatric nurse practitioner that her daughter has extreme mood swings prior to her periods, which the adolescent vehemently denies. When asked if she notices anything different just before her periods, the adolescent points to her mother and says, "She gets really hard to live with." This demonstrates which characteristic of adolescent thinking? a. Apparent hypocrisy b. Imaginary audience c. Overthinking d. Personal fable

ANS: A Apparent hypocrisy is the notion that rules apply differently to adolescents than to others. The adolescent who chalks up the conflict with her mother related to her premenstrual mood swings does not see her own role in the conflict. Imaginary audience is the perception that everyone is thinking about them. Personal fable is the idea that they are special. Overthinking involves making things more complicated than they need to be.

6. The mother of a 6-month-old infant is distressed because the infant can say "dada" but not "mama" and asks the primary care pediatric nurse practitioner why this is when she is the one who spends more time with the infant. How will the nurse practitioner respond? a. "At this age, your baby does not understand the meaning of sounds." b. "Babies at this age cannot make the 'ma' sound." c. "Most sounds made by babies at this age are accidental." d. "This may mean that your baby doesn't hear well."

ANS: A At 6 months, infants delight in vocalizing sounds that they learn by imitation but do not ascribe meaning to the sounds they make. Infants can say "mama" but without meaning. Babies make sounds on purpose by imitating what they hear. A preference for one sound early in speech does not indicate a hearing deficit.

9. The parent of a 5-month-old is worried because the infant becomes fussy but doesn't always seem interested in nursing. What will the nurse practitioner tell this parent? a. The infant may be expressing a desire to play or to rest. b. The parent should give ibuprofen for teething pain before nursing. c. This is an indication that the infant is ready for solid foods. d. This may indicate gastrointestinal discomfort such as constipation.

ANS: A At this age, infants may cry when they are tired or need social interaction and not just when they are hungry. The PNP should teach parents about this change in social development so they can be responsive to their infant's needs. Solid foods are not added until age 6 months. Teething usually does not begin until at least 6 months. GI discomfort usually occurs after eating.

6. The primary care pediatric nurse practitioner is counseling a new parent about ways to reduce the risk of sudden infant death syndrome (SIDS). What will the nurse practitioner include when discussing SIDS? a. Bed-sharing with infants greatly increases the risk of SIDS. b. Breastfeeding does not appear to have any influence on SIDS risk. c. Infants who attend day care have a higher than usual incidence of SIDS. d. There is no difference in SIDS rates in immunized versus non-immunized infants.

ANS: A Bed-sharing with infants has been shown to have a five-fold increase in the incidence of SIDS, even with non-drug using and non-smoking parents

13. The primary care pediatric nurse practitioner is discussing fitness and exercise with the parents of a 5- year-old child who ask what kinds of activities are developmentally appropriate for their child. What will the nurse practitioner recommend? a. Bike riding b. Interactive play c. Martial arts d. Organized sports

ANS: A Bike riding away from traffic or with parents is a good activity for the preschool to early school-age child. Interactive play is recommended for toddlers. Martial arts and organized sports are recommended for school-age children.

11. A school-age child has had abdominal pain for 3 months that occurs once or twice weekly and is associated with a headache and occasional difficulty sleeping, often causing the child to stay home from school. The child does not have vomiting or diarrhea and is gaining weight normally. The physical exam is normal. According to Bishop, what is included in the initial diagnostic work-up for this child? a. CBC, ESR, amylase, lipase, UA, and abdominal ultrasound b. CBC, ESR, CRP, and fecal calprotectin c. CBC, ESR, CRP, UA, stool for ova, parasites, and culture d. Stool for H. pylori antigen and serum IgA, IgG, tTg

ANS: A Bishop suggests these labs as an initial approach in children suspected of having functional abdominal pain, along with a 3-day trial of a lactose-free diet. Fecal calprotectin is added if the child has changes in stool habits suggestive of inflammatory changes in the intestinal tract. Answer C is correct according to Rasquin's recommendations as is stool for H. pylori antigen.

7. What will the primary care pediatric nurse practitioner teach the parents of a child who has new pressure-equalizing tubes (PET) in both ears? a. Parents should notice improved hearing in their child. b. PET will help by reducing the number of ear infections the child has. c. The child should use earplugs when showering or bathing. d. The tubes will most likely remain in place for 3 to 4 years.

ANS: A By reducing middle ear fluid, the child with hearing loss from this condition should show improvement in hearing. Children may still have infections but without persistent effusion. Earplugs are not necessary unless the child's head is submerged. PETs usually fall out on their own; if they are still in place 2 to 3 years after placement, they should be removed by the otolaryngology surgeon.

8. Which lab value is most concerning in an infant with fever and a suspected bacterial infection? a. C-reactive protein of 11.5 mg/L b. Lymphocyte count of 8.7 c. Platelet count of 475 d. White blood cell count of 14

ANS: A CRP levels are non-specific acute phase indicators of inflammation with low diagnostic value except in predicting the likelihood of sepsis in infants, especially when the level is greater than 10 mg/L. Elevated lymphocyte, platelet, or WBC counts help with the differential diagnosis, but these values are not especially concerning.

9. The primary care pediatric nurse practitioner has a cohort of patients who have special health care needs. Which is an important role of the nurse practitioner when caring for these children? a. Care coordination and collaboration b. Developing protocols for parents to follow c. Monitoring individual education plans (IEPs) d. Providing lists of resources for families

ANS: A Care coordination is one of the key elements for children with special health care needs. PNPs are especially suited for this role and have the unique skills to function as care coordinators. Care for these children should involve shared decision making and individualized care and not "cookbook" approaches. The PNP may advocate for children's health care needs for the IEP but does not monitor these. The PNP should not just give parents lists of phone numbers but should assist them to make appointments.

7. A child is in the clinic because of symptoms of purulent, foul-smelling nasal discharge from the right nostril. Nasal visualization reveals something shiny in a mass of mucous in the nasal cavity. What will the primary care pediatric nurse practitioner do? a. Attempt to remove the mass gently using alligator forceps. b. Perform a saline nasal rinse using a water jet device. c. Refer the child to a pediatric otolaryngologist. d. Suction the mucoid mass using a bulb syringe.

ANS: A Children often insert foreign bodies into their nasal cavities and, if undetected for any period of time, will develop foul-smelling, unilateral, purulent nasal discharge. The foreign body may become embedded in granulation tissue or mucosa. If possible, the PNP should attempt removal if the FB is visible and can be easily removed without causing trauma. Saline nasal rinses with pressure may push the FB farther into the cavity. Referral to ENT may be necessary if attempts to remove the FB are not successful. Suction is not indicated.

1. 8. The parents of a toddler tell the primary care pediatric nurse practitioner that they get frustrated trying to get the child to eat any vegetables other than squash and carrots. What will the nurse practitioner recommend? a. a. Continue to offer a variety of foods without forcing the child to eat them. b. b. Offer snacks to make up for calories the child misses by not eating the vegetables. c. c. Prepare dishes the child likes to ensure that a vegetable is eaten at each meal. d. d. Require the child to take 1 to 2 bites of each food at each meal.

ANS: A Children reject new foods (food neophobia) for a variety of reasons and it sometimes takes as many as 15 to 20 exposures to a food before they become accustomed to it and enjoy eating it. Parents should offer the foods and remove them without comment if they are rejected in order to avoid food battles. If the child refuses foods, parents should not give snacks to make up for lost calories

7. The parent of a school-age child is concerned that the child is going to be short like both parents and worries that he will have difficulty in school if he can't participate in a variety of sports. What will the primary care pediatric nurse practitioner do to counsel this parent? a. Encourage the child to engage in regular physical activity. b. Overlook his or her own feelings about this physical characteristic. c. Point out the accomplishments of other short people. d. Steer the child into other activities at school.

ANS: A Children should be encouraged to participate in all activities and to explore their own abilities and limitations without attributing "good" or "bad" to any of these experiences to improve self-esteem. Parents should be encouraged to explore their own feelings about their child's appearance to avoid subliminal messages of low self-worth. Pointing out the characteristic puts an unnecessary focus on the trait. Steering the child to specific activities is limiting.

18. A 7-year-old child who has a history of a repaired congenital heart defect has many dental caries along with gingival erythema and irritation and a temperature of 102.5°F. What will the primary care pediatric nurse practitioner do next? a. Admit to the hospital with a pediatric cardiology consult. b. Obtain blood cultures and a CBC and consult a pediatric cardiologist. c. Refer the child to a pediatric dental surgeon immediately. d. Start prophylactic antibiotics such as penicillin twice daily for 2 weeks.

ANS: A Children who are suspected of having SBE should be admitted to the hospital and referred to pediatric cardiology. The child should begin treatment as soon as SBE is suspected, so getting labs and then consulting a cardiologist is not correct. The SBE is the priority treatment, not the gingivitis or caries. Treatment should include IV antibiotics in the hospital.

7. Because of their inability to ambulate, children with cerebral palsy should be evaluated for which nutrients? a. Calcium and vitamin D b. Fat-soluble vitamins c. Iron and zinc d. Sodium and potassium

ANS: A Children who do not place weight on their bones are at risk for osteopenia and should have vitamin D and calcium levels monitored and supplemented if indicated.

2. The primary care pediatric nurse practitioner performs a well child examination on a 9-month-old infant who has a history of prematurity at 28 weeks' gestation. The infant was treated for retinopathy of prematurity (ROP) and all symptoms have resolved. When will the infant need an ophthalmologic exam? a. At 12 months of age b. At 24 months of age c. At 48 months of age d. At 60 months of age

ANS: A Children who have a history of ROP requiring treatment, even if ROP has completely resolved, will need yearly ophthalmologic follow-up. Less frequent follow-up is required for children with ROP who did not require treatment.

4. The primary care pediatric nurse practitioner is performing a well child examination on a school-age child who had complete repair of a tetralogy of Fallot defect in infancy. What is important in this child's health maintenance regime? a. Cardiology clearance for sports participation b. Restriction of physical activity to avoid pulmonary complications c. Sub-acute bacterial endocarditis prophylaxis precautions d. Teaching about management of hypercyanotic episodes

ANS: A Children who have had TOF repair must be cleared by cardiology before participation in sports, but there is no need to restrict all physical activity. SBE prophylaxis is given prior to surgery and for 6 months afterward. Hypercyanotic episodes occur before repair.

5. The primary care pediatric nurse practitioner performs a well child examination on a 12-month-old child who had repair of a congenital heart defect at 8 months of age. The child has a normal exam. The parent reports that the child is not taking any medications. The nurse practitioner will contact the child's cardiologist to discuss whether the child needs which medication? a. Amoxicillin b. Capoten c. Digoxin d. Furosemide

ANS: A Children who have had complete repair of CHD should have SBE prophylaxis with amoxicillin for 6 months after the procedure. Capoten, an antihypertensive, digoxin, an inotropic medication, and furosemide, a diuretic, are given for specific symptoms as indicated.

14. A 14-year-old female comes to the clinic with amenorrhea for 3 months. A pregnancy test is negative. The adolescent's body weight is at 82% of expected for height and age. The mother reports that her daughter often throws up and refuses to eat most foods. Which condition does the primary care pediatric nurse practitioner suspect? a. Anorexia nervosa b. Bulimia nervosa c. Depression d. Substance abuse

ANS: A Children with anorexia nervosa are usually underweight. Refusal to maintain body weight at least 85% expected for age and height or failure to gain weight during growth periods so that weight drops below 85% expected is one of the diagnostic criteria of anorexia. Some may throw up frequently, but children with bulimia nervosa are generally average weight or overweight. Depression and substance abuse may be comorbidities, but these signs are consistent with anorexia nervosa.

6. The primary care pediatric nurse practitioner reviews hematology reports on a child with beta- thalassemia minor and notes an Hgb level of 8 g/dL. What will the nurse practitioner do? a. Evaluate serum ferritin. b. Order Hgb electrophoresis. c. Prescribe supplemental iron. d. Refer for RBC transfusions.

ANS: A Children with beta-thalassemia minor may have low hemoglobin without iron deficiency so, before prescribing iron, the PNP should measure serum iron levels or serum ferritin. Hgb electrophoresis is indicated in a child whose diagnosis is unknown to diagnose this disorder. Supplemental iron should only be ordered when there is documented iron deficiency. RBC transfusions are controversial and used only for more severe iron deficiency.

2. A child who has sustained a head injury after falling on the playground is brought to the clinic. The parents report that the child cried immediately and was able to walk around after falling. The primary care pediatric nurse practitioner notes slight slurring of the child's speech and the child has vomited twice in the exam room. Which course of action is warranted? a. Admit the child to the hospital for a neurology consult. b. Observe the child in the clinic for several hours. c. Order a head CT and observe the child at home. d. Send the child home with instructions for follow-up.

ANS: A Children with certain symptoms, such as vomiting or slurred speech after a head injury, should be admitted to the hospital for neurologic consultation. If the child had not exhibited these symptoms, any of the other options would be acceptable.

2. The parent of a 15-year-old male is concerned that he refuses to eat meals with the family and consumes only protein drinks. The adolescent is on the track team at school and spends much of his time training and working out. The primary care pediatric nurse practitioner notes that his weight and BMI have dropped from the 20th percentile to the 3rd percentile in the past year. This child most likely has a problem with a. body image. b. personal identity. c. role performance. d. self-esteem.

ANS: A Children with disturbed body image may have concerns related to appearance, body size, function, or potential. Possible behaviors include eating disorders and a preoccupation with the perfect body. Children with body image problems become overly concerned with appearance and compare themselves to others. This child is losing weight and working out too often to try to change his body. Children with personal identity issues internalize negative perceptions of others and manifest feelings of inferiority. Children with role performance problems feel incompetent and are hesitant to try new things or become perfectionists to overcompensate. Children with poor self-esteem seek attention, importance, and security.

23. A previously healthy school-age child develops herpes zoster on the lower back. What will the primary care pediatric nurse practitioner do to manage this condition? a. Order Burow solution and warm soothing baths as comfort measures. b. Prescribe oral acyclovir 30 mg/kg/day in 4 doses/day for 5 days. c. Recommend topical antihistamines to control itching. d. Stress the need to remain home from school until the lesions are gone.

ANS: A Children with herpes zoster should be treated with comfort measures (symptomatic treatment). Oral acyclovir is not recommended for all children but may be useful in children who are immunosuppressed or have more severe forms. Topical antihistamines are used with caution in children because of the risk of toxicity. If the lesions can be covered, children do not need to be kept home from school.

17. A 3-year-old child has just completed a 7-day course of amoxicillin for a second febrile urinary tract infection and currently has a negative urine culture. What is the next course of action? a. Obtain a renal and bladder ultrasound. b. Prescribe prophylactic antibiotics to prevent recurrence. c. Refer the child for a voiding cystourethrogram. d. Screen urine regularly for leukocyte esterase and nitrites.

ANS: A Children with recurrent UTI should have a renal and bladder US to assess for hydronephrosis, scarring, or other atypical findings. If the US is concerning, VCUG and/or DMSA screen may be performed. Screening regularly is not indicated.

11. The primary care pediatric nurse practitioner is examining a school-age child who complains of frequent stomach pain and headaches. The parent reports that the child misses several days of school each month. The child has a normal exam. Before proceeding with further diagnostic tests, what will the nurse practitioner initially ask the parent? a. About the timing of the symptoms each day and during the week b. How well the child performs in school and in extracurricular activities c. If the parent feels a strong need to protect the child from problems d. Whether there are any unusual stressors or circumstances at home

ANS: A Children with school refusal or school phobia often have symptoms that gradually improve as the day progresses and often disappear on weekends. The PNP should ask about the frequency and duration of the symptoms to evaluate this pattern. The other options are important questions when management of school phobia has begun as a way of understanding underlying causes for the reluctance to go to school.

9. A child has several shallow mucosal lesions on the buccal mucosa and tongue that are surrounded with an erythematous halo and covered by yellow plaques. What will the primary care pediatric nurse practitioner recommend? a. Chlorhexidine gluconate b. Diphenhydramine and Maalox c. Oral acyclovir d. Topical antiviral medication

ANS: A Chlorhexidine gluconate rinses are useful to treat aphthous ulcers. Diphenhydramine with Maalox and oral acyclovir are used to treat viral stomatitis. Topical antiviral medications are not indicated.

17. The parent of a school-age girl reports that the child has difficulty getting ready for school and is often late because of a need to check and recheck whether her teeth are clean and her room light has been turned off. What will the primary care pediatric nurse practitioner recommend to this parent? a. Cognitive-behavioral therapy b. Deferral of treatment until symptoms worsen c. Medication management with an SSRI d. Referral to a child psychiatrist

ANS: A Cognitive-behavioral therapy is used for mild to moderate symptoms of OCD. Children who have mild symptoms that do not interfere with their lives can defer treatment, but this is not the case in this situation. Medication and referral to a child psychiatrist are used for more severe symptoms.

6. A primary care pediatric nurse practitioner working in a community health center wishes to develop a program to assist impoverished children and families to have access to healthy foods. Which strategy will the pediatric nurse practitioner employ to ensure the success of such a program? a. Asking community members to assist in researching and implementing a program b. Designing a community garden approach that involves children and their parents c. Gaining support from the corporate community to provide needed resources d. Providing evidence-based information about the importance of a healthy diet

ANS: A Community collaboration can be fostered through community-based participatory research (CBPR), which is transformative research that bridges the gap between science and practice by actively engaging communities with formally trained researchers. In this type of research, community members formally participate in all aspects of the process, making the findings more relevant to the community it affects the most. Designing a community garden approach without first knowing whether the community needs or wants it does not ensure success. Gaining support from the corporate community without input from the affected community does not guarantee success. Giving evidence-based information does not involve the community members in research and does not increase success.

4. The parent of a 14-year-old child asks the primary care pediatric nurse practitioner how to help the child prevent injuries when basketball tryouts begin later in the school year. Which recommendation will be of most benefit? a. Preseason conditioning b. Proper footwear c. Protective knee braces d. Stretching before practices

ANS: A Conditioning in the preseason is one of the most important things children can do to build muscle strength, to prevent sports injuries, and to learn how to make twisting, jumping, and landing movements safely. Proper footwear is also recommended but is not the most important. Protective knee braces may be worn but do not prevent injury. Stretching should be done after warming up to maintain flexibility.

10. An adolescent exhibits mild depressive symptoms and tells the primary care pediatric nurse practitioner that he is most concerned about difficulty falling and staying asleep. The adolescent does not want to take medication to treat the depressive symptoms. What will the nurse practitioner recommend? a. A program of sleep hygiene and gradual sleep extension b. A sedative-narcotic will help both sleep and depression c. Cognitive therapy can help the adolescent to sleep better d. Using an antidepressant will improve sleep patterns

ANS: A Depression is linked to sleep problems and both predict and are predicted by a diagnostic cluster that includes ODD, GAD, and depression. One study found that gradual sleep extension combined with sleep extension advice had a beneficial effect on depressive symptoms of adolescents with chronic sleep reduction. Sedatives will not affect depression. Cognitive therapy is useful for insomnia related to anxiety. Antidepressants do not necessarily treat insomnia.

3. The primary care pediatric nurse practitioner sees a developmentally delayed toddler for an initial visit. The family has just moved to the area and asks the nurse practitioner about community services and resources for their child. What should the nurse practitioner do initially? a. Ask the parents if they have an individualized family service plan (IFSP). b. Consult with a physician to ensure the child gets appropriate care. c. Inform the family that services are provided when the child begins school. d. Refer the family to a social worker for assistance with referrals and services.

ANS: A Families with children who have developmental delays are eligible for early intervention services and should have IFSPs in place. This family may have one from their previous community, and it can be used as a starting point to determine needs. It is not necessary to consult with a physician to coordinate community resources. Early intervention is provided from birth, according to federal law. Until the specific referrals are known, the social worker is not consulted.

2. A 2-year-old child who has SCA comes to the clinic with a cough and a fever of 101.5°C. The child currently takes penicillin V prophylaxis 125 mg orally twice daily. What will the primary care pediatric nurse practitioner do? a. Admit the child to the hospital to evaluate for sepsis. b. Give intravenous fluids and antibiotics in clinic. c. Increase the penicillin V dose to 250 mg. d. Order a chest radiograph to rule out pneumonia.

ANS: A Fever and pulmonary symptoms are two conditions warranting referral or emergency admission to the hospital to rule out sepsis and acute chest syndrome. Increasing the dose of penicillin V or giving IV antibiotics is not indicated.

27. An adolescent who had cradle cap as an infant is in the clinic with thick crusts of yellow, greasy scales on the forehead and behind the ears. What will the primary care pediatric nurse practitioner recommend? a. Daily application of ketoconazole 2% topical cream b. High-potency topical corticosteroids applied daily c. Mineral oil and shampoo on the affected areas d. Selenium sulfide shampoo twice weekly to the face

ANS: A For facial dermatitis, daily ketoconazole 2% topical cream may be used. If steroids are prescribed, only low-dose steroids should be used on the face. Mineral oil and shampoo are recommended for cradle cap in infants. Selenium sulfide shampoo is used for scalp dermatitis.

15. The parent of a 3-month-old reports that the infant arches and gags while feeding and spits up undigested formula frequently. The infant's weight gain has dropped to the 5th percentile from the 12th percentile. What is the best course of treatment for this infant? a. Begin a trial of extensively hydrolyzed protein formula for 2 to 4 weeks. b. Institute an empiric trial of acid suppression with a proton pump inhibitor (PPI). c. Perform esophageal pH monitoring to determine the degree of reflux. d. Reassure the parent that these symptoms will likely resolve by 12 to 24 months.

ANS: A Formula-fed infants may be given a trial of a hydrolyzed protein formula to see if improvement occurs. An empiric trial of a PPI may be used in children and adolescents but is not recommended in infants. Esophageal pH monitoring may be performed in consultation with a specialist but not as first-line evaluation. The infant has warning signs of GERD that require further investigation and not just reassurance.

1. 12. The mother of a newborn infant asks the primary care pediatric nurse practitioner about pumping her breasts when she returns to work in 2 months. What will the nurse practitioner include in teaching this mother? a. a. Frozen breast milk may be stored up to 3 months in a 0° F freezer. b. b. Once she begins pumping the infant should drink only pumped breast milk. c. c. Pumped breast milk must be discarded after 3 days when stored in the refrigerator. d. d. Unused defrosted breast milk may be stored in the refrigerator for 48 hours.

ANS: A Frozen breast milk may be stored up to 3 months in a well-regulated freezer. It is not necessary to use only pumped breast milk once this begins. Refrigerated breast milk may be stored up to 8 days. Once frozen breast milk is thawed it should be used within 24 hours.

13. The parent of a school-age child reports that the child becomes frustrated when unable to perform tasks well and often has temper tantrums and difficulty sleeping. Which disorder may be considered in this child? a. Generalized anxiety disorder (GAD) b. Obsessive-compulsive disorder (OCD) c. Pediatric autoimmune neuropsychiatric disorder associated with streptococcal infection (PANDAS) d. Separation anxiety disorder (SAD)

ANS: A GAD is characterized by over-concern about competence, significant self-consciousness, irritability and tantrums, and poor sleep. OCD results in recurring thoughts, images, or impulses. Patients with PANDAS have OCD- and Tourette-like symptoms. SAD causes difficulties separating from caregivers and being away from home.

10. The parents of a special needs child tell the primary care pediatric nurse practitioner that they are planning a 3-month visit to their home country in Africa. The pediatric nurse practitioner assists the family to obtain a sufficient supply of medications and formula and to make sure that the child's equipment can be transported and used during the trip and at the destination. This is an example of a. global application. b. global awareness. c. system application. d. system awareness.

ANS: A Global application involves having a willingness and ability to adjust to the needs of clients, families, and communities both nationally and globally. Global awareness involves knowledge of diseases, political, and economic factors worldwide that affect health. System application involves assisting clients to overcome institutional barriers to effective interventions. System awareness is knowledge of these barriers.

18. A child has gross hematuria, abdominal pain, and arthralgia as well as a rash. What diagnosis is most likely? a. Henoch-Schönlein purpura b. Rhabdomyosarcoma c. Sickle cell disease d. Systemic lupus erythematosus

ANS: A HSP may presents with gross hematuria in the presence of abdominal pain with or without bloody stools, arthralgias, and a purpuric rash. Rhabdomyosarcoma is characterized by gross hematuria and voiding dysfunction. Sickle cell disease can cause gross hematuria but not always.

14. An 18-month-old child has horizontal, bright white lines along the upper gum line of the teeth. What is the most important question the primary care pediatric nurse practitioner will ask the child's parents? a. If the child is still drinking milk from a bottle b. If the child or the parents are brushing the teeth c. If they are brushing the child's teeth twice daily d. If they have taken the child to a dentist

ANS: A Horizontal, bright white lines along the gum line are commonly seen in children still drinking from bottles and are early caries lesions. The PNP should ask about weaning from the bottle. The other questions are an important aspect of dental health, but the issue of baby bottle tooth decay should be addressed first.

15. A school-age female has had vulvovaginitis for 2 months. All cultures and tests are negative, but the symptoms persist after treatment with both topical antibiotics and oral amoxicillin. What is the next course of action to treat this condition? a. Estrogen cream at bedtime for 2 to 3 weeks b. Referral to a pediatric gynecologist for further evaluation c. Trimethoprim-sulfamethoxazole daily for 1 to 2 months d. Workup for possible sexual abuse

ANS: A If antibiotics fail to treat prepubertal, nonspecific vulvovaginitis, the PNP should order estrogen cream to thicken the vulvar epithelium. If treatment fails, referral to a pediatric gynecologist should be made. TMP-SMX is used for recurrent vulvovaginitis. Sexual abuse is considered if signs of trauma are present or if STI testing is positive.

1. A child who has otitis externa has severe swelling of the external auditory canal that persists after 2 days of therapy with ototopical antibiotic/corticosteroid drops. What is the next step in treatment for this child? a. Insert a wick into the external auditory canal. b. Irrigate the external auditory canal with saline. c. Order systemic corticosteroids. d. Prescribe an oral antibiotic medication.

ANS: A If significant swelling is present, inserting a wick into the EAC is helpful and should be impregnated with antibiotics as long as it is in place. Irrigation is contraindicated during an acute infection. Systemic steroids and antibiotics are not indicated.

2. An adolescent female has heavy periods that are also irregular. The physical exam is normal. A complete blood count reveals a hemoglobin of 8.9 g/dL. What test will the primary care pediatric nurse practitioner order next? a. Coagulation studies b. C-reactive protein c. Thyroid function d. Ultrasound of pelvis

ANS: A If the patient's hemoglobin is low, coagulation studies should be ordered. CRP is ordered if infection is suspected. Thyroid function is indicated if systemic disease is suspected. A pelvic ultrasound is ordered if a mass is palpated, anomaly is suspected, bimanual exam cannot be completed, or if the condition is unresponsive to treatment.

9. The primary care pediatric nurse practitioner prescribes a twice daily inhaled corticosteroid for a 12- year-old child. At a well child visit, the child reports not using the medication on a regular basis. Which response by the pediatric nurse practitioner demonstrates an understanding of client-centered care? a. Asking the child to describe usual daily routines and schedules b. Referring the family to a social worker to help with medication compliance c. Reviewing the asthma action plan with the parent and the child d. Teaching the child how the medication will help to control asthma symptoms

ANS: A In a client-centered relationship, there is reciprocal communication and understanding. The PNP should be able to understand the client's perspective and unique situation. The first step is to evaluate possible reasons for nonadherence and not to make referrals or re-educate until potential barriers have been identified and negotiation with the client has occurred.

2. The primary care pediatric nurse practitioner is considering use of a relatively new drug for a 15- month-old child. The drug is metabolized by the liver, so the nurse practitioner will consult a pharmacologist to discuss giving the drug: a. less often or at a lower dose. b. more often or at a higher dose. c. via a parenteral route. d. via the oral route.

ANS: A Infants metabolize drugs more slowly than older children due to decreased levels of oxidases and conjugating enzymes produced in the immature liver, so they may need drugs given less often or at lower doses to avoid toxicity. The route does not necessarily play a role in this case.

16. The primary care pediatric nurse practitioner performs a well child examination on a 1-month-old. The infant was recently discharged from the neonatal intensive care unit after treatment with parenteral acyclovir for a neonatal herpetic infection and is currently taking oral acyclovir. What will the nurse practitioner do to manage this infant's care? a. Obtain regular absolute neutrophil counts. b. Perform routine skin cultures for herpes simplex virus. c. Reinforce the need to give acyclovir indefinitely. d. Stop the oral acyclovir at 2 months of age.

ANS: A Infants treated with parenteral acyclovir for neonatal herpetic infections should remain on oral acyclovir for 6 months. The PNP should obtain regular ANC levels and temporarily discontinue acyclovir if neutropenia occurs until the neutrophil count recovers. New lesions should be cultured, but routine skin cultures are not indicated. Oral acyclovir suppressive therapy for 6 months after parenteral treatment of any classification of acute neonatal disease has been shown to reduce the recurrences of mucocutaneous lesions and improve neuro-developmental outcomes.

3. During an assessment of a 4-week-old infant, the primary care pediatric nurse practitioner learns that a breastfed infant nurses every 2 hours during the day but is able to sleep for a 4-hour period during the night. The infant has gained 20 grams per day in the interval since last seen in the clinic. What will the nurse practitioner recommend? a. Continuing to nurse the infant using the current pattern b. Nursing the infant for longer periods every 4 hours c. Supplementing with formula at the last nighttime feeding d. Waking the infant every 2 hours to nurse during the night

ANS: A Infants who are encouraged to breastfeed every 2 to 3 hours may have one longer stretch of 4 hours at night. This infant is gaining between 0.5 and 1 gram per day, which is appropriate. It is not necessary to alter the pattern of nursing or to supplement with formula.

12. The primary care pediatric nurse practitioner performs a well baby examination on a 4-month-old infant who is exclusively breastfed and whose mother plans to introduce only small amounts of fruits and vegetables in addition to breastfeeding. To ensure that the infant gets adequate amounts of iron, what will the nurse practitioner recommend? a. Elemental iron supplementation of 1 mg/kg/day until cereals are added b. Elemental iron supplementation of 3 mg/kg/day for the duration of breastfeeding c. Monitoring the infant's hemoglobin and hematocrit at every well-baby checkup d. Offering iron-fortified formula to ensure adequate iron intake

ANS: A Infants who are exclusively breastfeeding or who receive more than half of their diet from breast milk should be given 1 mg/kg/day of supplemental iron until iron-containing foods are added to the diet. It is not necessary to monitor Hgb/Hct regularly unless the child has symptoms. Formula is not necessary for breastfeeding infants.

1. 1. The primary care pediatric nurse practitioner is examining a newborn who is breastfeeding and notes the presence of an ankyloglossia. What will the nurse practitioner do next? a. a. Ask the mother if the infant has any feeding difficulties. b. b. Refer the infant for a possible frenulectomy. c. c. Schedule an appointment with a lactation consultant. d. d. Suggest that the mother feed breast milk by bottle.

ANS: A Infants with ankyloglossia may have difficulty feeding if the tongue does not extend well. The PNP should first assess feeding difficulties and then may refer for a lactation consultant or consider a frenulectomy.

1. 18. The primary care pediatric nurse practitioner is performing a well-baby checkup on a 6-month-old infant and notes a candida diaper rash and oral thrush. The infant has had two ear infections in the past 2 months and is in the 3rd percentile for weight. What will the nurse practitioner do? a. a. Order a CBC with differential and platelets and quantitative immunoglobulins. b. b. Order candida and pneumococcal skin tests and lymphocyte surface markers. c. c. Refer the infant to an immunologist for evaluation of immunodeficiency. d. d. Refer the infant to an otolaryngologist to evaluate recurrent otitis media.

ANS: A Infants with warning signs of immunodeficiency, such as recurrent infections, skin infections, and oral thrush, should be evaluated. The initial step is to order a CBC with differential, platelets, and immunoglobulins. If this is not helpful, referral to an immunologist for further testing, such as candida and pneumococcal skin tests and lymphocyte surface markers, is warranted. Referral to an otolaryngologist is not indicated.

1. 1. The parent of a school-age child reports that the child usually has allergic rhinitis symptoms beginning each fall and that non-sedating antihistamines are only marginally effective, especially for nasal obstruction symptoms. What will the primary care pediatric nurse practitioner do? a. a. Order an intranasal corticosteroid to begin 1 to 2 weeks prior to pollen season. b. b. Prescribe a decongestant medication as adjunct therapy during pollen season. c. c. Recommend adding diphenhydramine to the child's regimen for additional relief. d. d. Suggest using an over-the-counter intranasal decongestant.

ANS: A Intranasal corticosteroids are a key component in long-term therapy to manage symptoms associated with AR. These should be begun 1 to 2 weeks prior to the beginning of pollen season. Decongestants are not recommended for long-term use because of side effects. Diphenhydramine causes daytime drowsiness.

5. The pediatric nurse practitioner provides primary care for a 30-month-old child who has sickle cell anemia who has had one dose of 23-valent pneumococcal vaccine. Which is an appropriate action for health maintenance in this child? a. Administer an initial meningococcal vaccine. b. Begin folic acid dietary supplementation. c. Decrease the dose of penicillin V prophylaxis. d. Give a second dose of 23-valent pneumococcal vaccine.

ANS: A Invasive bacterial infection is the leading cause of death in young children with SCA. Meningococcal vaccine should be given initially for all children over the age of 2 years and a booster dose given every 5 years after that. Folic acid supplementation is often used for adults but not for children unless there is a documented deficiency. Penicillin V prophylaxis is started at 2 months of age, with the dose increased at age 3 years. The 23-valent pneumococcal polysaccharide second dose is given 5 years after the first.

1. The primary care pediatric nurse practitioner is performing a well child exam on a 4-month-old infant who is nursing exclusively. The mother reports that the infant has had a marked decrease in the number of stools each day, from 3 to 5 stools each day to only one stool every other day. How will the nurse practitioner respond? a. Ask the mother to describe the color and consistency of the stools. b. Explain to the mother that breastfed infants should have daily stools. c. Recommend using a glycerin suppository as needed. d. Suggest to the mother that she increase her intake of fluids.

ANS: A It is common for older breastfed infants to stool less frequently. The PNP should assess the color and consistency of the stools to make sure that they are normal. As long as infants are happy, thriving, and free from clinical signs of GI distress, parents can be reassured that this is normal. It is not necessary to use a glycerin suppository or ask the mother to increase fluids in the absence of clinical pathology.

1. 9. The primary care pediatric nurse practitioner is evaluating a 2-year-old with a documented speech delay. Screenings to assess motor skills and cognition are normal, and the child passed a recent hearing test. What will the pediatric nurse practitioner do next? a. a. Ask the child's parents whether they read to the child. b. b. Give parents educational materials to encourage speech. c. c. Refer the child to an early intervention program. d. d. Suggest that they purchase age-appropriate music videos.

ANS: A Language development requires oral-motor ability, auditory perception, and cognitive ability, which this child has been shown to have, as well as the psychosocial-cultural environment to motivate the child to engage in language use. The PCPNP's initial step should be to determine whether the parents provide such an environment. Educational materials may be used after it is determined that these are useful. Early intervention may be used if the speech delay persists. Music videos do not necessarily engage the child in expression of speech.

12. The parent of a 3-year-old child tells the primary care pediatric nurse practitioner that the child has never been able to fall asleep without a parent in the room. The child has a new sibling and the parent is concerned that the toddler's cries will awaken the infant. What will the nurse practitioner counsel the parent? a. Leaving the room as the child is falling asleep and returning at intervals to check on the child b. Offering a reward for each night the child falls asleep without the parent in the room c. Putting the child to bed at the same time every night and ignoring all sleep interfering behaviors d. Taking away a favorite activity or video for each night the child fusses about the parent not being in the room

ANS: A Leaving the room as the child becomes drowsy and checking on the child at intervals is called graduated extinction and allows parents to ensure safety while helping the child to initiate and maintain sleep independently. The other measures may result in the toddler becoming upset and crying, which would awaken the baby. Rewards and punishments are not necessarily successful.

9. During a well child examination, a 15-year-old female tells the primary care pediatric nurse practitioner that some of her friends have begun having sex. She has a boyfriend but denies engaging in sex with him. What will the nurse practitioner do initially? a. Ask her for her definitions of "sex." b. Discuss the risks of sexually transmitted diseases. c. Find out if she is considering sexual relations. d. Give her information about contraception.

ANS: A Many adolescents do not equate oral or anal intercourse with sex, so it is important to find out how this patient defines sex. The other options also may be considered depending on the situation, however, clarity about the words used in the discussion are most important initially for the nurse practitioner to focus the subsequent conversation appropriately.

1. 4. During a well child examination of a school-age child from a family who recently immigrated from Africa, the primary care pediatric nurse practitioner learns that the child has been involved in many arguments at school. The parents are concerned that their child will never fit in with classmates. How will the nurse practitioner address this situation? a. a. Assess the conditions in the country of origin prior to immigration. b. b. Recommend counseling to determine underlying causes of this behavior. c. c. Stress that this may be a normal response to feeling different at school. d. d. Suggest that the child may be responding to being bullied by others.

ANS: A Many children who exhibit signs of spiritual distress can express this by being angry or withdrawn. Trauma or violence to the self or to others can contribute to spiritual distress. The PNP should evaluate conditions in the country of origin since it is likely that the family may have escaped war or persecution prior to immigration. Once underlying conditions are known, the PNP can recommend options or try to explain the behavior to parents.

5. The primary care pediatric nurse practitioner is considering using a drug for an "off-label" use in a child. The nurse practitioner has used the drug in a similar situation previously, has consulted a pharmacology resource and the FDA website, and has determined that there are no significant contraindications and warnings for this child. What else must the nurse practitioner do when prescribing this drug? a. Discuss recommendations with the parents and document their consent. b. Document anecdotal reports of previous use of the drug by other providers. c. Follow up daily with the parents to determine safe administration of the drug. d. Report this use to the FDA Medwatch website for tracking purposes.

ANS: A Many prescriptions are written for "off-label" uses for children because the drug doesn't have enough substantial evidence for FDA approval. The PNP should make sure to discuss the drug and this use with the family and document the decision-making process and their consent for this use. It is not enough to base a decision solely on what someone else has done. Unless the drug is experimental or has many serious adverse effects, close daily monitoring is not necessary. The PNP is not required to report off- label drug use to the FDA.

1. 8. A 4-month-old infant has a history of reddened, dry, itchy skin. The primary care pediatric nurse practitioner notes fine papules on the extensor aspect of the infant's arms, anterior thighs, and lateral aspects of the cheeks. What is the initial treatment? a. a. Moisturizers b. b. Oral antihistamines c. c. Topical corticosteroids d. d. Wet wrap therapy

ANS: A Moisturization is the first-line therapy to interrupt the itch-scratch-itch cycle. Oral antihistamines are used mostly to allow sleep during nighttime pruritus. Topical corticosteroids are used if moisturization is not effective. Wet wrap therapy is used to treat flares with recalcitrant disease.

15. The primary care pediatric nurse practitioner is discussing newborn care with a mother who is pregnant with triplets. When counseling the mother about feeding issues, the nurse practitioner will recommend a. developing a plan to rotate breastfeeding for her infants. b. making sure that the triplets are on the same feeding schedule. c. pumping her breasts so she can feed breastmilk to all three. d. supplementing with formula to ensure adequate nutrition.

ANS: A Mothers of multiples can breastfeed their infants. With more than two infants, the mother should develop a rotation schedule. While it is recommended to attempt to get the babies on the same schedules for feeding and sleeping, this is not always possible and not necessary. Infant suckling is the best way to increase milk supply. Supplementation is not recommended unless her milk supply is inadequate.

8. The primary care pediatric nurse practitioner is counseling an obese 16-year-old client about weight management. The adolescent says, "I know I need to lose weight, but I don't want to give up all my favorite foods." When using motivational interviewing techniques, how will the nurse practitioner respond? a. "Do you think there are any foods you could limit or do without for a while?" b. "I hear you telling me that you really don't have a desire to lose weight." c. "If you can't give up these foods, you won't see the benefits of weight loss." d. "In the long run, the sacrifices you make today will improve your health."

ANS: A Motivational interviewing (MI) uses persuasion, rather than coercion, drawing out the individual's motivation for change in order to build a collaborative partnership between the client and the counselor. This adolescent has indicated an understanding of the need to lose weight but doesn't want to give up favorite foods. The PNP asks if there are any foods, allowing the client to choose possible options. In answer B, the PNP does not acknowledge the client's motivations. In the other two answers, the PNP doesn't allow choices.

22. A child will need an occlusive dressing to treat lichen simplex chronicus. What will the primary care pediatric nurse practitioner tell the parents about applying this treatment? a. Apply ointment before the dressing. b. Plastic wrap should not be used. c. The dressing should be applied to dry skin. d. Change the dressing twice daily.

ANS: A Occlusive dressings are placed over creams and ointments to enhance hydration and absorption of topical medications. Plastic wrap is often used. The medications and dressings should be applied to damp skin. The dressing should not be left on more than 8 hours.

7. An adolescent has 2+ proteinuria in a random dipstick urinalysis. A subsequent first-morning voided specimen is negative. What will the primary care pediatric nurse practitioner do to manage this condition? a. Monitor for proteinuria at each annual well child examination. b. Order a 24-hour timed urine collection for creatinine and protein excretion. c. Reassure the parents that this is a benign condition with no follow-up needed. d. Refer the child to a pediatric nephrologist for further evaluation.

ANS: A Orthostatic proteinuria, demonstrated by proteinuria of greater than 1+ with activity and low-protein to normal urine on a first-morning void, is common in adolescents. If the first-morning void is negative, the adolescent should be monitored annually. A 24-hour urine collection is not indicated unless the first- morning void is elevated. Although the orthostatic proteinuria is mostly benign, annual monitoring is recommended and patient education should stress the importance of follow-up to evaluate the cause of proteinuria. Children with mild asymptomatic proteinuria who have a normal first-morning specimen do not require extensive testing for kidney disease but should be monitored annually. Unless proteinuria is severe or persistent, referral to a nephrologist is not indicated.

23. The primary care pediatric nurse practitioner is reviewing medical records for a newborn that is new to the clinic. The toddler's mother was found to be HIV positive during her pregnancy with this child and received antiretroviral therapy during pregnancy. The child was born by cesarean section, begun on anti-retroviral prophylaxis, and did not breastfeed. What is the correct management for this child? a. Consult with a pediatric HIV specialist. b. Discontinue cART after 4 weeks of age. c. Obtain a CD4+ cell count and HIV RNA levels. d. Reinforce the need to give cART for life.

ANS: A PNPs may manage infants exposed in utero to HIV but should do so in consultation with a pediatric HIV specialist. cART should be given for 6 weeks. Lab work is ordered according to protocol at the direction of the specialist. Many children who are treated according to the protocol do not become HIV positive.

5. The primary care pediatric nurse practitioner performs a physical examination on a 12-year-old child and notes poor hygiene and inappropriate clothes for the weather. The child's mother appears clean and well dressed. The child reports getting 6 to 7 hours of sleep each night because of texting with friends late each evening. What action by the nurse practitioner will help promote healthy practices? a. Discuss setting clear expectations about self-care with the mother b. Give the child information about sleep and self-care c. Reassure the mother that this "non-compliance" is temporary d. Tell the mother that experimenting with self-care behaviors is normal

ANS: A Parents of school-age children should be advised to set clear limits for their children for cleanliness, healthy exercise, hours of sleep, and other health promotion behaviors to encourage the development of responsibility for these things. Giving the child information can be done along with setting expectations, but, at this age, the parent should still be supervising. While "non-compliance" is a part of this process, and is a means of asserting independence, parents need to discuss this with children to resolve the issue.

4. The primary care pediatric nurse practitioner is counseling the parents of a 13-year-old female who has Down syndrome about sexual maturation. What will the nurse practitioner tell these parents? a. It is important to discuss and support healthy sexuality. b. Providing too much information about sexuality may be confusing given the child's cognitive level of understanding. c. Suppressing periods with contraceptives will lessen their daughter's distress. d. They should give her information about periods but not about sexuality.

ANS: A Persons with disabilities have the same desires to make decisions and foster fulfilling relationships with others as other people have. Unless healthy sexuality is taught and supported, unhealthy and abusive sexuality is more likely to occur. Parents should give information when it is desired and delivered in a manner appropriate to the child's level of understanding. Suppressing periods only ignores the issue but does not change the increased feelings that accompany puberty.

9. The parent of a school-age child reports that the child doesn't like being alone in rooms because of a fear of aliens hiding in closets. What will the primary care pediatric nurse practitioner tell the parent? a. "Fear of imaginary creatures does not usually occur at this age." b. "I may need to refer your child to a pediatric mental health specialist." c. "Your child is expressing normal fears for a school-age child." d. "Your child may be watching too much violence on television."

ANS: A Preschoolers tend to fear imaginary creatures but not school-age children. The PNP does not have enough information about the severity of the fear or its effects on behavior to refer the child to a mental health specialist. The PNP should evaluate the situation before attributing the fear to a cause such as television.

10. The primary care pediatric nurse practitioner is performing a well child examination on a high school age adolescent who plays football who has hypercalciuria. Which dietary supplement will the nurse practitioner question the adolescent about? a. Protein supplements b. Salt tablets c. Sports drinks d. Vitamin C

ANS: A Protein supplements can cause hypercalciuria with calcium loss and dehydration if protein intake is too high. Salt tablets can cause hypernatremia and delayed gastric emptying. Sports drinks are high in sugar and electrolytes, which will not affect the calcium content of the urine. It is not necessary to take vitamin C.

10. A single mother of an infant worries that living in a household with only one parent will cause her child to be maladjusted. To help address the mother's concerns, the primary care pediatric nurse practitioner will suggest : a. developing consistent daily routines for the child. b. exposing her child to extended family members when possible. c. not working outside the home during the first few years. d. taking her child to regular play date activities with other children.

ANS: A Providers can teach parents that providing predictable, consistent, and loving care helps an infant to learn trust and help influence positive brain development. Involving extended family members and going to play dates are good ways to socialize children but are not essential to learning trust. It may not be possible for her to be a stay-at-home mother.

2. The primary care pediatric nurse practitioner is examining a child whose parents recently emigrated from a war-torn country in the Middle East. Which is a priority assessment when performing the patient history? a. Asking about physical, psychological, and emotional trauma b. Determining the parents' English language competency and literacy level c. Learning about cultural preferences and complementary medicine practices d. Reviewing the child's previous health and illness records

ANS: A Recent history that includes trauma, loss, and refugee camp experience may exacerbate difficulties adjusting to life in the U.S. and can lead to acute and chronic physical and mental health concerns. All of the other parts of the history will be necessary, but this should be a priority, since the family has escaped a war-torn country.

11. The parent of a newborn has quit smoking cigarettes within the past month and reports feeling fidgety. Using a "reframing" technique, how will the primary care pediatric nurse practitioner respond? a. Explore ways that the parent can use this extra energy to do things for the baby. b. Remind the parent that this is a normal, temporary part of nicotine withdrawal. c. Suggest that the parent take up exercise to enjoy the benefits of not smoking. d. Tell the parent that, over time, these symptoms of withdrawal will subside.

ANS: A Reframing is a counseling strategy that uses the context of an experience to give it a new meaning, creating a frame of reference that focuses on a desired outcome instead of the current problem. The withdrawal symptoms associated with tobacco cessation are uncomfortable, but the PNP can suggest channeling this nervous energy into positive action for the baby. Telling the parent that the symptoms of withdrawal are temporary or normal does not reframe the perception. Suggesting exercise may be beneficial, but does not reframe the situation.

12. The primary care pediatric nurse practitioner is discussing lifestyle changes with an adolescent who has hypertension. What will the nurse practitioner recommend about exercise for this client? a. Regular to vigorous activity initially with a combination of resistance and aerobic exercise to maintain lower blood pressure b. Moderate daily exercise such as walking for 20 minutes daily with increasing intensity as blood pressure drops c. Vigorous aerobic exercise combined with maximal strength training to lower blood pressure d. Vigorous aerobic exercise only to reduce blood pressure and then to maintain lowered blood pressure

ANS: A Regular to vigorous physical activity for 30 minute 3 days per week helps to lower blood pressure. Resistance training may be combined with aerobic exercise after blood pressure is lowered to help maintain lowered blood pressure. Strength training is contraindicated in children with hypertension.

5. An unimmunized school-age child whose mother is in her first trimester of pregnancy is diagnosed with rubella after a local outbreak. What will the primary care pediatric nurse practitioner recommend? a. Assessment of maternal rubella titers b. Intravenous immunoglobulin for the child c. MMR vaccine for the mother and child d. Possible termination of the pregnancy

ANS: A Reinfection or revaccination with rubella for pregnant women rarely results in congenital rubella syndrome, and these are not a reason for pregnancy termination. Maternal rubella antibody titers should be assessed. MMR vaccine is not given during pregnancy. IVIG is not indicated

10. During a well child exam of a school-age child, the primary care pediatric nurse practitioner learns that the child has been having angry episodes at school. The nurse practitioner observes the child to appear withdrawn and sad. Which action is appropriate? a. Ask the child and the parent about stressors at home b. Make a referral to a child behavioral specialist c. Provide information about anger management d. Suggest consideration of a different classroom

ANS: A School-age children are learning to manage emotions and need help to manage their feelings in acceptable ways. A variety of stressors, including parental divorce, substance abuse, bullying in school, and early responsibilities, can cause anxiety in the child, who may not manage these feelings well. Until the underlying cause is better understood, management options cannot be determined, so referrals to specialists, information about anger management, or moving to a different classroom may not be indicated.

7. A child is diagnosed with tinea versicolor. What is the correct management of this disorder? a. Application of selenium sulfide 2.5% lotion twice weekly for 2 to 4 weeks b. Oral antifungal treatment with fluconazole once weekly for 2 to 3 weeks c. Sun exposure for up to an hour every day for 2 to 4 weeks d. Using ketoconazole 2% shampoo on lesions twice daily for 2 to 4 weeks

ANS: A Selenium sulfide lotion or 1% shampoo is first-line treatment for children and younger adolescents. Oral antifungal medications are used in resistant cases in older adolescents. Sun exposure only intensifies lesions. Ketoconazole shampoo is used on older adolescents.

7. The parent of an adolescent reports noting cutting marks on the teen's arms and asks the primary care pediatric nurse practitioner what it means. What will the nurse practitioner tell this parent? a. Cutting is a way of dealing with emotional distress. b. It is a method of fitting in with other adolescents. c. The behavior is common and will usually stop. d. This type of behavior is a type of suicide attempt.

ANS: A Self-injurious behavior (SIB) is used as a coping strategy to relieve distress, anger, and stress. It is not commonly done among adolescents and is not a way of fitting in with a peer group. Because it indicates underlying distress, adolescents must get help identifying these causes. Many have a history of physical, sexual, or emotional abuse. Although individuals who engage in SIB are more likely to attempt suicide in the future, the act itself is not a suicide attempt.

2. The parent of a 4-year-old child reports that the child seems to be having trouble adjusting to a new day care and reportedly is always engaging in solitary play when the parent arrives to pick up the child. What will the primary care pediatric nurse practitioner do? a. Ask the parent if the child is slow to warm up to other new situations. b. Reassure the parent that parallel play is common among preschool-age children. c. Recommend that the parent spend time encouraging the child to play with others. d. Suggest that the day care center may be neglecting the child.

ANS: A Shyness is a pattern of social inhibition with unfamiliar people, novel objects, or novel situations. If the child shows this pattern in other situations, shyness is most likely the cause of this behavior. Parallel play is common among toddlers and not preschoolers. Parent should be supportive and not push children to interact. Because this shyness is a common pattern in this age child, it is not necessary to suggest that neglect is occurring, unless there are other signs.

7. An adolescent female reports facial pain and frequent popping of her jaw. An exam reveals unilateral tender facial muscles and a deviation of the mandible to the affected side with opening of the mouth. What will the primary care pediatric nurse practitioner do? a. Recommend ice packs, NSAIDs, and a soft diet. b. Refer to a pediatric mental health specialist. c. Refer to an orthodontist for a surgical intervention. d. Suggest obtaining Botox injection treatments.

ANS: A TMJ is a biopsychosocial problem. Conservative treatment should always be the first course of treatment, so ice, NSAIDs, and dietary changes should be recommended to minimize discomfort. If conservative measures are not effective, children should be referred to a team of dentists and psychologists to manage the problem. Surgical intervention is not recommended. Botox injections are being studied but currently are not FDA approved.

5. The primary care pediatric nurse practitioner is evaluating health literacy in the mother of a new preschool-age child. How will the nurse practitioner assess this? a. Ask the child how many books he has at home. b. Ask the mother about her highest grade in school. c. Ask the mother to determine the correct dose of a drug from a label. d. Ask the mother to read a health information handout aloud.

ANS: A The "newest vital sign," or health literacy, can be determined quickly by asking the parent how many children's books are in the home. Greater than 10 books in the home is an independent positive predictor of adequate parent health literacy. The other questions may determine a specific level of literacy in general but are not as efficient.

2. The primary care pediatric nurse practitioner counseling the parent of an overweight school-age child about improving overall fitness. What will the nurse practitioner include? a. Encourage the child to begin by engaging in swimming or cycling. b. Exercise will help lower total cholesterol and low-density lipoproteins. c. School-age children need 60 minutes of moderate exercise daily. d. Strength training exercises are not safe for school-age children.

ANS: A The AAP suggests that overweight children initially participate in activities that place less stress on weight-bearing joints, such as swimming or cycling. Exercise helps raise HDL levels but does not reduce total cholesterol or LDL levels. School-age children need 60 minutes of physical activity but not necessarily exercise each day. Strength training exercises are safe, but powerlifting and maximal weight training are not, because of effects on developing bones.

1. 3. The primary care pediatric nurse practitioner is performing a medication history on a child and learns that the child's parents use various complementary treatments and remedies for the child. According to the American Academy of Pediatrics standard for providers, what will the nurse practitioner do? a. a. Evaluate the safety and efficacy of each product and monitor use. b. b. Incorporate these therapies into standard care practices. c. c. Recommend not using the products until the child is older. d. d. Suggest that most of these treatments are not safe for children.

ANS: A The AAP suggests that providers be nonjudgmental and sensitive to cultural and ethnic practices of patients and that they keep an open mind regarding the use of complementary and alternative therapies as long as safety and efficacy are evaluated and monitored. Products that are not safe should be discouraged and not just incorporated into standard care. Telling parents not to use the products and suggesting that they are not safe may be perceived as judgmental and will likely prevent collaboration with the parent.

10. What will the primary care pediatric nurse practitioner elicit when obtaining a positive Barlow maneuver when screening for developmental dysplasia of the hip? a. Dislocation of an unstable hip b. Dropping of the iliac crest with a raised leg c. Reduction of a dislocated hip d. Unequal knee heights in a supine child

ANS: A The Barlow maneuver dislocates an unstable or dislocatable hip posteriorly. Having the child raise a leg and watching for dropping of the iliac crest on that side is the Trendelenberg sign. The Ortolani maneuver reduces a posteriorly dislocated hip. Unequal knee heights in a supine child occur with a positive Galeazzi maneuver.

12. The primary care pediatric nurse practitioner is performing a well child assessment on an adolescent and is concerned about possible alcohol and tobacco use. Which assessment tool will the nurse practitioner use? a. CRAFFT b. HEEADSSS c. PHQ-2 d. RAAPS

ANS: A The CRAFFT tool is a six-question tool used to screen for adolescent substance abuse. The HEEADSSS is used as a psychosocial screening tool. The PHQ-2 is a rapid screen for depression. The RAAPS is used to assess risk behaviors that contribute to most morbidity, mortality, and social problems in teens.

4. When formulating developmental diagnoses for pediatric patients, the primary care pediatric nurse practitioner may use which resource? a. DC: 0-3R b. ICD-10-CM c. ICSD-3 d. NANDA International

ANS: A The DC: 0-3R refers to the Diagnostic Classification of Mental Health and Developmental Disorders of Infancy and Early Childhood and is useful for developmental problem diagnosis. The ICD-10-CM is the International Classification of Diseases-Tenth Revision, Clinical Modification and is useful for identifying physiologic diseases. The ICSD-3 is the International Classification of Sleep Disorders - 3rd edition. NANDA International is used to label problems in the functional health domain.

1. A 15-year-old female has a positive pregnancy test and asks the primary care pediatric nurse practitioner not to tell her parents. She is tearful and says she isn't sure she wants to keep the baby. What will the nurse practitioner do first? a. Determine the state-mandated reporting laws. b. Encourage the adolescent to talk to her parents. c. Obtain a social work consult to discuss adoption options. d. Refer her to a prenatal care specialist for follow-up.

ANS: A The PNP should first determine what the state's reporting laws are in case there are mandatory provisions for reporting statutory rape. The other options may be correct, depending on the laws and on the decisions of the adolescent.

14. A school-age child is seen in the clinic after a fragment from a glass bottle flew into the eye. What will the primary care pediatric nurse practitioner do? a. Refer immediately to an ophthalmologist. b. Attempt to visualize the glass fragment. c. Irrigate the eye with sterile saline. d. Instill a topical anesthetic.

ANS: A The PNP should never attempt to remove an intraocular foreign body or any projectile object but should refer immediately to an ophthalmologist. Visualizing the object, irrigating the eye, or instilling drops may further injure the eye.

2. The pediatric nurse practitioner provides primary care for a special needs infant whose parent takes an active role in the infant's care. The parent has a high school diploma and asks many questions about her infant's treatments. Which approach will the nurse practitioner take to ensure health literacy for this parent? a. Ask the parent to read back all information given. b. Encourage the parent to ask questions when confused. c. Provide written materials presented at an 8th grade level. d. Reinforce written information with verbal instructions.

ANS: A The PNP should take a precautionary approach and assume that all clients will have health literacy limitations. Assessment of health literacy can be done by asking clients to "read back" the information the provider gives. While encouraging questions is important, parents may not want to admit that they are confused and may not ask questions. Written materials should be given at a 5th grade level. Parents who do not understand medical terms will not understand written or verbal information.

9. The primary care pediatric nurse practitioner is evaluating recurrent stomach pain in a school-age child. The child's exam is normal. The nurse practitioner learns that the child reports pain most evenings after school and refuses to participate in sports but does not have nausea or vomiting. The child's grandmother recently had gallbladder surgery. Which action is correct? a. Encourage the child to keep a log of pain, stool patterns, and dietary intake b. Order radiologic studies and laboratory tests to rule out systemic causes c. Reassure the child and encourage resuming sports when symptoms subside d. Refer the child to a counselor to discuss anxiety about health problems

ANS: A The PNP suspects a somatic disorder after a normal exam and should encourage the child to keep a food or pain diary to help manage symptoms. The PNP should not "medicalize" the problem with tests. The child should be encouraged to resume sports and participate in normal activities. If the symptoms persist, referral for counseling is warranted.

4. A pharmaceutical company has developed a new drug that was tested only on adults. The FDA has declared this drug to have potential benefits for ill children. According to the Pediatric Research Equity Act (PREA), what may the pharmaceutical company be required to do? a. Conduct pediatric drug studies to determine whether the drug is safe and effective in children. b. Provide labeling stating that the safety and efficacy of the drug is not established for children. c. Receive a patent extension for conducting pediatric studies to determine use in children. d. Survey existing data about the drug to determine potential use in the pediatric population.

ANS: A The PREA gives the FDA more leverage over the types of new drugs developed for children and can require pharmaceutical companies to conduct pediatric drug trials if the FDA declares a drug as possibly useful to ill children or one that might be used by a substantial number of children. The Food and Drug Administration Modernization Act (FDAMA) allowed labeling that "safety and effectiveness in pediatric patients have not been established" on drugs with insufficient evidence to support pediatric indications. The Best Pharmaceuticals of Children Act (BPCA) grants a patent extension when a drug company voluntarily studies a known or new drug in children. The FDAMA also requires pharmaceutical companies to survey existing data and determine potential drug use and indications in pediatric populations.

10. A 9-month-old infant has developed two teeth since the 6-month checkup. The local water supply contains fluoride. What will the primary care pediatric nurse practitioner do to promote healthy dentition at this visit? a. Apply sodium fluoride varnish to the infant's teeth. b. Encourage the parents to make an initial dental appointment. c. Prescribe oral fluoride supplementation. d. Teach the parents how to brush the infant's teeth with fluoride toothpaste.

ANS: A The U.S. Preventive Task Force has issued two recommendations for preventing caries in children, including a recommendation that primary providers apply sodium fluoride varnish to the primary teeth of all infants and children beginning at the onset of the first tooth. The American Academy of Pediatric Dentistry (AAPD) recommends establishment of a dental home by age 12 months, or 6 months after the eruption of the first primary tooth. Oral fluoride supplementation is given when local water supplies are fluoride deficient. Fluoride toothpaste is not recommended in infancy unless the risk for caries is high.

13. An adolescent has localized bleeding of the gums when brushing the teeth. An exam of the mouth reveals the presence of plaque and calculus on the teeth, which are not loose. What will the primary care pediatric nurse practitioner recommend? a. Consistently brushing and flossing the teeth twice daily b. Referral to an oral surgeon for treatment c. Rinsing the mouth daily with chlorhexidine gluconate d. Using a xylitol-containing gum after meals

ANS: A The adolescent has gingivitis from poor dental hygiene, which is reversible with good hygiene. It is not necessary to refer to an oral surgeon. Chlorhexidine gluconate is not indicated. A xylitol-containing gum can prevent caries but does not improve hygiene.

5. The primary care pediatric nurse practitioner cares for children from a Native American family and learns that they used many herbs to treat and prevent illness. Which approach will the pediatric nurse practitioner use to promote optimum health in the children? a. Ask about the types of practices used and when they are applied. b. Provide a list of harmful herbs and ask the family to avoid those. c. Suggest that the family avoid using these remedies in their children. d. Tell the parents to use the herbs in conjunction with modern medications.

ANS: A The challenge, when working with families from different cultural backgrounds, who use alternative or complementary medicines, is to find ways to achieve a mutual understanding of the differences and to negotiate an acceptable plan of care. The first step is to begin a discussion about these practices. Providing a list of harmful herbs, suggesting that the family avoid certain herbs, and suggesting that the herbs are only an adjunct to "modern medicine" will sound disparaging and will convey a sense of mistrust.

8. The primary care pediatric nurse practitioner evaluates a 4-year-old girl whose parent reports frequent urination in the evenings on weekdays, incontinence after voiding. The parent reports that the child has soft formed stools 5 or 6 times weekly. Which assessment will the nurse practitioner make initially? a. Examination for labial adhesions b. Palpation for abdominal masses c. Screening for potential child abuse d. Urine culture and sensitivity

ANS: A The child exhibits incontinence after voiding, or vaginal voiding, which may indicate labial adhesions. Examination for this may be completed easily during the physical assessment. Since the parent reports normal stools, it is less likely that chronic constipation is causing dysfunctional voiding. Screening for child abuse may be necessary if physiologic causes are ruled out. If a UTI is suspected, the first test will be a urinalysis, not a culture.

4. A newly divorced mother of a toddler reports that the child began having difficulty sleeping and nightmares along with exhibiting angry outbursts and tantrums 2 months prior. The primary care pediatric nurse practitioner learns that the child refuses to play with usual playmates and often spends time sitting quietly. What will the nurse practitioner do initially? a. Ask the mother about the child's relationship with the father. b. Consult with a child psychiatrist to prescribe medications. c. Recommend cognitive behavioral or psychodynamic therapy. d. Refer the family to a child behavioral specialist for counseling.

ANS: A The child exhibits signs of PTSD. Because the parents are newly divorced, the PNP should evaluate the child's previous interactions with the father to determine whether violence occurred. If PTSD is likely, referral to social service agencies may be warranted. Pediatric mental health specialists may be involved once a diagnosis is established and may order medications.

9. The primary care pediatric nurse practitioner is discussing toileting issues with the parent of a 3-year- old toddler who reports that the child has been toilet trained for several months but has recently been refusing to have bowel movements and is becoming constipated. What will the nurse practitioner do? a. Ask the parent about bathroom facilities in the child's day care. b. Refer the child to a gastroenterologist for evaluation of pathology. c. Suggest putting the child in diapers and resuming toilet training in a few weeks. d. Tell the parent that this represents a developmental delay.

ANS: A The child has bowel dysfunction that may be related to restricted access to bathroom facilities, causing the child to actively try to prevent bowel movements. The fact that the child previously had control indicates a behavioral cause. It is not necessary to refer to a specialist. Putting the child in diapers and resuming toilet training later is useful for the child who is unable to toilet train. Without further evaluation, the PNP cannot determine that there is a developmental delay.

3. The primary care pediatric nurse practitioner evaluates a 5-year-old child who presents with pallor and obtains labs revealing a hemoglobin of 8.5 g/dL and a hematocrit of 31%. How will the nurse practitioner manage this patient? a. Prescribe elemental iron and recheck labs in 1 month. b. Reassure the parent that this represents mild anemia. c. Recommend a diet high in iron-rich foods. d. Refer to a hematologist for further evaluation.

ANS: A The child has mild to moderate iron-deficiency anemia and will need iron supplementation. The hemoglobin, hematocrit, and reticulocytes should be reevaluated in 4 weeks after initiation of treatment. The child needs iron supplementation, so reassurance alone is not indicated. It is difficult to get iron from foods, so supplementation will be needed. Children with hemoglobin levels less than 4 g/dL and some children with hemoglobin levels less than 7 g/dL must be referred.

14. A 10-year-old child has had abdominal pain for 2 days, which began in the periumbilical area and then localized to the right lower quadrant. The child vomited once today and then experienced relief from pain followed by an increased fever. What is the likely diagnosis? a. Appendicitis with perforation b. Gastroenteritis c. Pelvic inflammatory disease (PID) d. Urinary tract infection (UTI)

ANS: A The child has the progression of symptoms typical of appendicitis with perforation - pain before vomiting that localizes to the RLQ and then relief of pain with onset of fever upon perforation. With gastroenteritis, vomiting precedes pain. PID symptomology includes increasing pain over time. The symptoms of an UTI include fever, chills, and urinary symptoms.

1. 12. A child who has been diagnosed with asthma for several years has been using a short-acting B2-agonist (SABA) to control symptoms. The primary care pediatric nurse practitioner learns that the child has recently begun using the SABA two or three times each week to treat wheezing and shortness of breath. The child currently has clear breath sounds and an FEV1 of 75% of personal best. What will the nurse practitioner do next? a. a. Add a daily inhaled corticosteroid. b. b. Administer 3 SABA treatments. c. c. Continue the current treatment. d. d. Order an oral corticosteroid.

ANS: A The child is showing a need to step up treatment based on the frequency of symptoms, greater than twice each week. The PNP should order an inhaled corticosteroid maintenance medication to control symptoms and reduce the need for a SABA. The child is not having an acute exacerbation, so does not need 3 SABA treatments. Oral corticosteroids are given for moderate obstruction, <70%.

1. 6. The primary care pediatric nurse practitioner is counseling the parents of a toddler about appropriate discipline. The parents report that the child is very active and curious, and they are worried about the potential for injury. What will the pediatric nurse practitioner recommend? a. a. Allow the child to explore and experiment while providing appropriate limits. b. b. Be present while the child plays to continually teach the child what is appropriate. c. c. Let the child experiment at will and to make mistakes in order to learn. d. d. Say "no" whenever the child does something that is not acceptable.

ANS: A The child who is securely attached uses the parents as a base from which to safely explore the world. Toddlers learn by doing and need to experiment to gain mastery over the environment. It is important that parents are present for safety, but parents should not be ever-present and controlling. Parents should be close by and should intervene if the child is at risk for injury. Continual criticism and the use of the word "no" can make the toddler feel powerless.

28. A 5-year-old child who has a history of pertussis infection as an infant is in the clinic for immunizations prior to kindergarten. Which vaccine will be given? a. DTaP b. DTP c. Td d. Tdap

ANS: A The duration of immunity after infection with pertussis in unknown and is thought to be short, so immunization for pertussis must still be provided. The DTP, with whole-cell pertussis vaccine is given in other countries but not in the U.S. Td does not contain pertussis. Tdap is given after age 7 years.

4. The primary care pediatric nurse practitioner is managing a 6-year-old child who has chronic constipation and encopresis. The nurse practitioner has ruled out neurogenic etiology. The parents report that the child was difficult to toilet train as a toddler. What is key to managing this child's condition? a. Encouraging use of maintenance medications for at least 2 months after resolution of constipation b. Referral to a mental health consultant to manage problems in the parent-child dyad c. Spending time with the parents to uncover their feelings about their child's condition d. Teaching the parents that the symptom of stool retention is often voluntary for the child

ANS: A The emphasis of management of encopresis in the absence of pathology is to establish a regular bowel routine after bowel evacuation. Parents should be taught to use the maintenance laxative for at least 2 months after resolution of the constipation. Referral to a mental health consultant should be reserved for situations where the child and family may have underlying issues contributing to an ongoing problem. It is not necessary to spend time attributing cause or blame unless the problem continues to be chronic following initial management strategies that include maintenance medications. Parents should be taught what the dynamics of encopresis, including retention, are and that they are involuntary.

5. During a well baby examination of a newborn, the primary care pediatric nurse practitioner notes adduction of the right forefoot, with normal position of the mid- and hind-foot, along with a convex- shaped lateral border of the foot. What will the nurse practitioner do to evaluate this deformity? a. Grasp the heel with one hand and abduct the forefoot with the other hand. b. Observe both legs for medial and lateral rotations. c. Order anterior-posterior and lateral radiographs of both feet. d. Refer the infant immediately to a pediatric orthopedic specialist.

ANS: A The foot position is characteristic of metatarsus adductus. The PNP should evaluate whether the deformity is rigid, requiring treatment by an orthopedist, or not. If the forefoot can be brought past the midline with the heel held in place, it is not considered to be a rigid deformity. Assessing legs for medial and lateral rotations is part of the exam for in-toeing. Radiographs are not routinely performed. Referral to an orthopedic specialist, if this is found to be rigid deformity, is not urgent.

3. A child who plays soccer is in the clinic reporting pain and swelling in both knees. A physical examination reveals swelling and focal tenderness at the tibial tuberosities, with pain worsening when asked to extend the knees against resistance. What is the treatment for this condition? a. Apply ice packs to both knees and avoid activities that cause pain. b. Begin quadriceps-stretching exercises now to prevent further injury. c. Obtain radiographic studies to rule out fractures or ligament tears. d. Refer to a pediatric orthopedic specialist to evaluate the need for surgery.

ANS: A The history of sports involving kicking a ball and the location and type of pain and swelling are consistent with Osgood-Schlatter disease (OSD). Management involves rest and ice and sometimes NSAIDs. Quadriceps-stretching exercises are not encouraged until the acute symptoms pass. Radiographic studies are not necessary. Referral is not indicated for OSD.

30. A 3-year-old child whose immunizations are up-to-date has been exposed to measles because of a localized outbreak among unvaccinated children. The parent reports that contact with infected children occurred within the last 2 days at a birthday party. What is the best course of action? a. Administer the MMR vaccine to help prevent disease. b. Give antiviral medications at the first sign of symptoms. c. Give the child a dose of immune globulin to mitigate the response. d. Reassure the parent that most exposed children will not get measles.

ANS: A The measles vaccine can be given to those exposed if given within 72 hours of exposure. IG may be given in those without prior measles vaccine. Antiviral medications are not effective. Nine of 10 children who are unimmunized or under-immunized will contract the disease if exposed.

4. An adolescent female tells the primary care pediatric nurse practitioner that she had unprotected sexual intercourse 4 days prior and is worried she might become pregnant. What will the nurse practitioner do? a. Prescribe ulipristal acetate (Ella). b. Recommend levonorgestrel (Plan B One Step). c. Start a combination OCP at regular doses. d. Suggest using the less expensive After Pill preparation.

ANS: A The prescription medication ulipristal can be used up to 5 days after unprotected intercourse. Levonorgestrel, which is the active ingredient in Plan B and the After Pill, should be taken within 72 hours of unprotected intercourse. The After Pill must be purchased online. A combination OCP may be used, but it is an alternative approach.

24. A child is brought to the clinic with a fever, headache, malaise, and a red, annular macule surrounded by an area of clearing and a larger, erythematous annular ring. The child complains of itching at the site. What will the primary care pediatric nurse practitioner do to determine the diagnosis? a. Ask about recent tick bites b. Obtain a skin culture c. Order blood cultures d. Perform serologic testing

ANS: A The presence of an erythema migrans rash with a positive history is diagnostic for Lyme disease, and no further testing is necessary. Because Borrelia burgdorferi is transmitted to humans through ticks, asking about recent tick bites is paramount to making this diagnosis. Skin and blood cultures are not indicated. Serology testing for IgG and IgM antibodies may be performed if the child is symptomatic without the characteristic EM rash.

2. The mother of a newborn tells the primary care pediatric nurse practitioner that she is worried that her child will develop allergies and asthma. Which tool will the nurse practitioner use to evaluate this risk? a. Three-generation pedigree b. Review of systems c. Genogram d. Ecomap

ANS: A The three-generation pedigree is used to map out risks for genetic diseases in families, as well as conditions with modifiable risk factors. The review of systems is used to evaluate the history of the child's body systems. The genogram is an approach to developing a family database to provide a graphic representation of family structure, roles, and problems of recurring significance in a family. The ecomap is used to identify relationships in the family and community that are supportive or harmful.

7. The primary care pediatric nurse practitioner is prescribing contraception for an adolescent who has not used birth control previously. The adolescent has a normal exam and has no family history of cardiovascular and peripheral vascular disease or diabetes. Which preparation is used initially? a. A combination oral contraceptive pills (OCP) with 30 to 35 mcg of estrogen and low progestin b. A combination OCP with low androgenic potency, such as Ortho-Cyclen c. A progestin-only mini-pill oral contraceptive d. A subdermal implant contraception, such as Implanon or Nexplanon

ANS: A The usual initial OCP is a combination with 30 to 35 mcg of estrogen and low progestin potency. The combination OCP with low androgenic potency is used for adolescents with hirsutism or PCOS. The progestin-only mini-pill is used in patients for whom progestin is contraindicated, such as for lactating women, and is not generally used in adolescents because of irregular bleeding and higher failure rates. The subdermal implants are used in older adolescents who are committed to long-term contraception.

5. A 17-year-old sexually active female who began having periods at age 14 reports having moderate to severe dull lower abdominal pain associated predominantly with periods but that occurs at other times as well. The history reveals a recent onset of these symptoms. A pregnancy test is negative. Which course of action is most important? a. Perform a full diagnostic workup to evaluate potential causes. b. Prescribe a prostaglandin synthetase inhibitor. c. Start a 3- to 6-month trial of oral contraceptive pills. d. Suggest using transcutaneous electrical nerve stimulation.

ANS: A This adolescent has symptoms consistent with secondary dysmenorrhea and should have a full diagnostic workup to evaluate potential causes. Prostaglandin synthetase inhibitors, OCPs, and TENS are useful to treat discomfort, but determining the cause is more important.

1. 4. A child is brought to the clinic immediately after being stung by a wasp while playing in the yard. The physical examination reveals localized redness and edema at the site, along with abdominal tenderness, watery eyes, and generalized hives. What is the initial treatment? a. a. Administer intramuscular epinephrine. b. b. Apply a topical glucocorticoid cream. c. c. Give oral diphenhydramine. d. d. Order a bronchodilator treatment.

ANS: A This child has signs of a severe reaction to the sting and should receive epinephrine first, followed by oral diphenhydramine and bronchodilators if wheezing. Topical glucocorticoids are used for mild, localized reactions.

8. A toddler who presents with anemia and reticulocytopenia has a history of a gradual decrease in energy and increase in pallor beginning after a recent viral infection. How will the primary care pediatric nurse practitioner treat this child? a. Closely observe the child's symptoms and lab values. b. Consult with a pediatric hematologist. c. Prescribe supplemental iron for 4 to 6 months. d. Refer for transfusions to correct the anemia.

ANS: A This child has symptoms and a history consistent with transient erythroblastopenia of childhood (TEC), which is usually self-limited. The PNP should monitor the child closely without treatment unless the anemia gets worse. Any of the other options may be necessary if the child's condition worsens.

4. The primary care pediatric nurse practitioner is examining a 5-year-old child who has had recurrent fevers, bone pain, and a recent loss of weight. The physical exam reveals scattered petechiae, lymphadenopathy, and bruising. A complete blood count shows thrombocytopenia, anemia, and an elevated white cell blood count. The nurse practitioner will refer this child to a specialist for a. bone marrow biopsy. b. corticosteroids and IVIG. c. hemoglobin electrophoresis. d. immunoglobulin testing.

ANS: A This child has symptoms and initial lab tests consistent with leukemia and should be referred to a pediatric hematologist-oncologist for a bone marrow biopsy for a definitive diagnosis. Corticosteroids and IVIG are given for severe ITP. Hgb electrophoresis is used to diagnose SCA. Immunoglobulins are evaluated when immune deficiency syndromes are suspected.

4. A child who has been taking antibiotics is brought to the clinic with a rash. The parent reports that the child had a fever associated with what looked like sunburn and now has "blisters" all over. A physical examination shows coalescent target lesions and widespread bullae and areas of peeled skin revealing moist, red surfaces. What will the primary care pediatric nurse practitioner do? a. Consult with a pediatric intensivist for admission to a pediatric intensive care unit. b. Order oral acyclovir 20 mg/kg/day in two doses for 6 to 12 months. c. Prescribe systemic antihistamines and antimicrobial medications as prophylaxis. d. Recommend analgesics, cool compresses, and oral antihistamines for comfort.

ANS: A This child has symptoms consistent with toxic epidermal necrolysis, which is potentially life-threatening. Children with symptoms should be admitted to the PICU for management. The other options are treatments for erythema multiforme, a more benign, viral-induced rash. Oral acyclovir is given when herpes simplex infection is possible.

3. A school-age child has a history of chronic otitis media and is seen in the clinic with vertigo. The primary care pediatric nurse practitioner notes profuse purulent otorrhea from both pressure- equalizing tubes and a pearly-white lesion on one tympanic membrane. Which condition is most likely? a. Cholesteatoma b. Mastoiditis c. Otitis externa d. Otitis media with effusion

ANS: A This child has symptoms of cholesteatoma, especially with a pearly white lesion on the TM. Mastoiditis involves the mastoid bone behind the ear.

14. A school-age child has had nasal discharge and daytime cough but no fever for 12 days without improvement in symptoms. The child has not had antibiotics recently and there is no significant antibiotic resistance in the local community. What is the appropriate treatment for this child? a. Amoxicillin 45 mg/kg/day b. Amoxicillin 80-90 mg/kg/day c. Amoxicillin-clavulanate 80-90 mg/kg/day d. Saline irrigation for symptomatic relief

ANS: A This child meets criteria for treatment of acute rhinosinusitis (ARS) based on duration of symptoms without clinical improvement. The initial treatment is amoxicillin 45 mg/kg/day. The higher dose is used to treat ARS in communities with resistant S. pneumonia. If antibiotics have been used previously, amoxicillin-clavulanate is used. The use of buffered isotonic saline into the nasal cavity by squeeze bottle or neti pot (in late childhood and adolescence) may be helpful, but the clinical guidelines do not support or negate the use of saline.

3. The primary care pediatric nurse practitioner is performing a well child check-up on a 20-month-old child. The child was 4 weeks premature and, according to a parent-completed developmental questionnaire, has achieved milestones for a 15-month-old infant. Which action is correct? a. Perform an in-depth developmental assessment screen at this visit to evaluate this child. b. Reassure the parent that the child will catch up to normal development by age 2 years. c. Re-evaluate this child's development and milestone achievements at the 2-year visit. d. Refer the child to a specialty clinic for evaluation and treatment of developmental delay.

ANS: A This child should be at a 19-month adjusted age for prematurity so, according to the parent screen, is 4 months behind. The PNP should perform a more in-depth screen to evaluate this delay. Waiting to see if the child will "catch up" or assuring the parent that this will happen will cause the delays to become more severe. A referral to a specialty clinic should not be made solely on the basis of the parent-completed questionnaire but only after further evaluation of possible delays.

5. A 9-year-old child exhibits school refusal and a reluctance to attend sleepovers with classmates. The parent is concerned because the child has recently begun sleeping in the parents' bed. Which initial action by the primary care pediatric nurse practitioner is appropriate? a. Assess for environmental stress, parental dysfunction, and maternal depression. b. Ask about recent traumatic events that may have precipitated this behavior. c. Consider a possible pediatric autoimmune neuropsychiatric disorder cause. d. Recommend firm insistence on school and activity attendance.

ANS: A This child shows symptoms of separation anxiety disorder. Environmental stress, parental dysfunction, and maternal depression are risk factors for this disorder. Assessing for traumatic events is necessary if PTSD is suspected. Pediatric autoimmune neuropsychiatric disorder manifests with OCD- and Tourette- like symptoms. If the child does have separation anxiety disorder (SAD), treatment and not discipline is warranted.

11. A 14-year-old boy who is overweight develops a unilateral limp with pain in the hip and knee on the affected side. An exam reveals external rotation of the hip when flexed and pain associated with attempts to internally rotate the hip. What is most important initially when managing this child's condition? a. Place the child on crutches or in a wheelchair to prevent weight-bearing. b. Provide information about weight loss to minimize further injury. c. Recommend seeing an orthopedic specialist as soon as possible. d. Refer the child to physical therapy to improve range of motion.

ANS: A This child's age, history, and symptoms are consistent with slipped capital femoral epiphysis. The child should be placed on crutches or in a wheelchair to prevent weight bearing. Obesity is often part of the history and should be managed, but the immediate need is to prevent further damage to the hip. Referral to orthopedics should immediately follow prevention of weight bearing. Physical therapy may be part of treatment after the epiphysis is stabilized.

5. The primary care pediatric nurse practitioner examines an infant whose weight is below the 3rd percentile and whose mother does not comply with the feeding regimen. When attempting to enlist the help of the infant's grandmother, the grandmother says, "My daughter was like this when she was a baby and she turned out all right." Which approach will the nurse practitioner take to improve the outcome for this infant? a. Ask the grandmother about her daughter's health during childhood. b. Explain that the condition is potentially serious if not treated. c. Give the grandmother and mother information about normal growth. d. Refer the family to a social worker to investigate possible neglect.

ANS: A This family has an altered perception of health and health behaviors based on their own experience with the condition. The grandmother considers this infant's weight similar to her own daughter's and assumes that things will resolve. The PNP should evaluate the mother's health to see if any problems occurred. Giving more information or trying to alarm the family will not be effective if their perception is that everything is normal. It is not necessary to refer to a social worker unless the condition persists.

1. The primary care pediatric nurse practitioner is preparing to conduct a well child assessment of an 8- year-old child. How will the nurse practitioner begin the exam? a. Ask the child about school, friends, home activities, and sports b. Discuss the purpose of the visit and explain the procedures that will be performed c. Offer age-appropriate information about usual developmental tasks d. Provide information about healthy nutrition and physical activities

ANS: A To build rapport with the child and parent, the PNP will begin by asking direct questions to the child, encouraging the child to share information about daily routines. The other answers list aspects of the well child visit that can be introduced after the initial conversation.

3. The primary care pediatric nurse practitioner is evaluating a 4-year-old female child for enuresis. The parents reports that the child has never been dry at night and has recently begun having daytime incontinence, usually when at preschool. The nurse practitioner learns that the child does not appear to have an abnormal urine stream. What will the nurse practitioner do next? a. Examine the urethral meatus and labia and obtain a dipstick clean catch urinalysis. b. Reassure the parent that the child probably gets distracted and puts off voiding until it is urgent. c. Refer the child to a pediatric urologist for evaluation of possible vesicoureteral reflux. d. Suggest a bladder retraining program and use of a nighttime bedwetting alarm.

ANS: A Urinary tract infection and labial adhesions are both in the differential diagnosis of dysfunctional voiding and can easily be ruled out or confirmed in the clinic setting with an exam and a UA. Although distractibility can contribute to daytime incontinence, it is important to rule out other causes. The child does not exhibit signs such as abnormal urine streams or recurrent UTI that warrant referral to a pediatric urologist. Until the causes are known, it is premature to recommend treatment.

18. A 12-year-old child who plays soccer is diagnosed with vocal cord dysfunction. What will the primary care nurse practitioner say when the child's parents ask about continued sports participation? a. The child may continue to participate in soccer. b. The child should limit activity to non-aerobic sports. c. This condition is a contraindication for all sports. d. This condition predisposes the child to sudden cardiac death.

ANS: A Vocal cord dysfunction causes shortness of breath and must be managed but does not prevent children from participation in sports. It does not indicate underlying cardiac problems and does not mean children should avoid any sport that may increase heart or respiratory rates.

2. A previously healthy school-age child develops a cough and a low-grade fever. The primary care pediatric nurse practitioner auscultates wheezes in all lung fields. Which diagnosis will the nurse practitioner suspect? a. Atypical pneumonia b. Bacterial pneumonia c. Bronchiolitis d. Bronchitis

ANS: A Wheezing in a child over 5 years of age without a history of wheezing may point to an atypical pneumonia. Bacterial pneumonia is characterized by diminished breath sounds or crackles along with high fever. Bronchiolitis causes coarse wheezing. Bronchitis is characterized by cough without adventitious lung sounds.

1. 10. The primary care pediatric nurse practitioner is examining a toddler who is below the 3rd percentile for weight even though the parents claim that the child eats "constantly." What will the nurse practitioner do initially? a. a. Evaluate the child's feeding and elimination behaviors and ask the family to describe mealtime routines. b. b. Recommend giving a multivitamin and offering high-calorie foods, such as ice cream. c. c. Refer the child to a feeding evaluation clinic for a swallow study and evaluation of possible GERD. d. d. Suggest that the parents supplement the child's food intake with a high-calorie formula.

ANS: A When a child doesn't thrive, it is first necessary to evaluate feeding and elimination behaviors and mealtime routines to determine what the child's actual intake is along with the types of foods that are eaten. Only after the underlying causes are discovered will the PNP prescribe treatment options.

10. The primary care pediatric nurse practitioner is examining a 12-month-old infant who was 6 weeks premature and observes that the infant uses a raking motion to pick up small objects. The PEDS questionnaire completed by the parent did not show significant developmental delays. What will the nurse practitioner do first? a. Perform an in-depth developmental assessment. b. Reassure the parent that this is normal for a premature infant. c. Refer the infant to a developmental specialist. d. Suggest activities to improve fine motor skills.

ANS: A When developmental screening indicates an infant is not progressing at the expected rate, additional testing to determine the degree of delay is necessary. A referral may be needed if a delay is determined. This is not normal for this degree of prematurity

8. The primary care pediatric nurse practitioner is counseling an adolescent who was recently hospitalized for an asthma exacerbation and learns that the child usually forgets to use twice- daily inhaled corticosteroid medications that are supposed to be given at 0800 and 2000 each day. Which strategy may be useful in this case to improve adherence? a. Ask the adolescent to identify two times each day that may work better. b. Consider having the school nurse supervise medication administration. c. Prescribing a daily oral corticosteroid medication instead. d. Suggest that the parent enforce the medication regimen each day.

ANS: A When working with adolescents who take medication, it is important to allow the adolescent to have input into dosing schedules and what works for them. Having the school nurse supervise does not allow autonomy and creates continued dependency. Daily oral corticosteroids are not used for maintenance. The PNP should assist the family with transitioning the adolescent from parent to teen administration and not suggest that parents enforce medication rules.

1. In a respiratory disorder causing a check-valve obstruction, which symptoms will be present? a. Air entry on inspiration with expiratory occlusion b. Complete obstruction on inspiration and expiration c. Narrowing of the lumen with increased air flow resistance d. Obstruction of air entry with unimpeded expiratory air flow

ANS: A With check-valve or ball-valve obstruction, air entry is possible, but the lumen is completely occluded during expiration so that air escape is impossible. A complete obstruction does not allow inspiratory or expiratory air flow. A bypass valve obstruction allows air flow but with increased resistance. There is no obstruction that impedes inspiration while allowing expiratory air flow.

1. The primary care pediatric nurse practitioner is performing a pre-participation sports physical examination on a 14-year-old male who will be on the wrestling team at school. What will the nurse practitioner include when discussing healthy practices with this adolescent? a. Risks associated with repeatedly losing and gaining weight b. The need for an electrocardiogram or echocardiogram prior to participation c. The need to consume 20 to 30 grams of protein after exercise d. To consume water with CHO prior to activity lasting up to an hour

ANS: A Wrestlers often try to lose weight rapidly prior to wrestling matches to put themselves into a lower weight category. It is important to teach young athletes about the risks associated with repeated weight loss and gain. ECG and echocardiograms are not recommended as a requirement for all pre-participation physical exams unless there is an indication for doing so, such as with syncope or murmurs. Athletes do not need to consume 10 to 20 grams of protein after exercise

14. A child who has nephrotic syndrome is on a steroids and a salt-restricted diet for a relapse of symptoms. A dipstick urinalysis shows 1+ protein, down from 3+ at the beginning of the episode. In consultation with the child's nephrologist, what is the correct course of treatment considering this finding? a. Begin a taper of the steroid medication while continuing salt restrictions. b. Continue with steroids and salt restrictions until the urine is negative for protein. c. Discontinue the steroids and salt restrictions now that improvement has occurred. d. Relax salt restrictions and continue administration of steroids until proteinuria is gone.

ANS: B Steroid medications and salt restrictions are continued until proteinuria resolves

6. A 16-year-old female reports breast tenderness and a "lump." The primary care pediatric nurse practitioner palpates a small fluid-filled mass in her right breast. A pregnancy test is negative. Which action is correct? a. Obtain a CBC to rule out infection. b. Order an ultrasound of the mass. c. Prescribe NSAIDs to treat her discomfort. d. Reassure her that the findings are normal.

ANS: B A cyst is usually a fluid-filled mass and should be evaluated by ultrasound to confirm this. The mass is not warm or consistent with mastitis, so a CBC is not indicated. NSAIDs may be offered once the diagnosis is confirmed. If the US reveals a cyst, reassurance can be given.

9. The parent of a school-age child tells the primary care pediatric nurse practitioner that the child is restless most nights and complains often that bugs are in the bed. After consultation with a sleep disorder specialist and subsequent evaluation of a ferritin level of 30, the nurse practitioner may expect to treat this child with a. clonazepam. b. ferrous sulfate. c. gabapentin. d. sertraline.

ANS: B A ferritin level of less than 50 is associated with periodic limb movements. The treatment for this is ferrous sulfate 3 mg/kg per day. Clonazepam and gabapentin may be ordered if ferritin levels are normal and other organic causes of PLM have been ruled out. Sertraline may make PLM worse.

10. A 2-month-old infant cries up to 4 hours each day and, according to the parents, is inconsolable during crying episodes with fists and legs noted to be tense and stiff. The infant is breastfeeding frequently but is often fussy during feedings. The physical exam is normal and the infant is gaining weight normally. What will the primary care pediatric nurse practitioner recommend? a. A complete work-up, including laboratory and radiologic tests b. Eliminating certain foods from the mother's diet c. Empiric treatment with a proton pump inhibitor medication d. Stopping breastfeeding and beginning a hydrolyzed formula

ANS: B A first step in a breastfeeding infant with symptoms of colic should be to try eliminating certain foods such as cow's milk products, nuts, eggs, and fish from the mother's diet to see if improvement occurs. It is not necessary to stop breastfeeding; hydrolyzed formulas may be tried in formula-fed infants. An infant with a normal exam and normal weight gain does not need further diagnostic studies. PPIs are not indicated as first-line therapy.

9. A 6-month-old infant has a retractile testis that was noted at the 2-month well baby exam. What will the primary care pediatric nurse practitioner do to manage this condition? a. Reassure the parent that the testis will most likely descend into place on its own. b. Refer the infant to a pediatric urologist or surgeon for possible orchiopexy. c. Teach the parent to manipulate the testis into the scrotum during diaper changes. d. Tell the parent that hormonal therapy may be needed to correct the condition.

ANS: B A retractile testis that does not retain scrotal residence should be referred to a pediatric urologist or surgeon by 6 months of age. By that age, the child should be seen by a specialist to rule out orchiopexy, in which the testis does not descend on its own (parental reassurance would not be appropriate). Teaching the parent to manipulate the testis is not indicated. Hormonal therapy has not demonstrated efficacy in stimulating testicular descent.

10. A 9-month-old infant is brought to the clinic with scrotal swelling and fussiness. The primary care pediatric nurse practitioner notes a tender mass in the affected scrotum that is difficult to reduce. What is the correct action? a. Obtain an abdominal radiograph. b. Refer immediately to a pediatric surgeon. c. Schedule an appointment with a pediatric urologist. d. Teach the parents signs of incarceration.

ANS: B A scrotal mass that is difficult to reduce or is painful is likely to be a hernia. Immediate referral is indicated to rule out incarceration, which is a medical emergency with potentially severe consequences if not promptly treated. The PNP may order radiographs to distinguish a hernia from a hydrocele, but not when these symptoms occur. The referral must be immediate, since surgery is required. A child with a non-tender, reducible hernia will require referral, but parents can be taught signs of incarceration until an appointment can be scheduled and the specialist seen.

1. A child is in the clinic after swallowing a metal bead. A radiograph of the GI tract shows a 6 mm cylindrical object in the child's stomach. The child is able to swallow without difficulty and is not experiencing pain. What is the correct course of treatment? a. Administer ipecac to induce vomiting. b. Have the parents watch for the object in the child's stool. c. Insert a nasogastric tube to flush out the object. d. Refer the child for endoscopic removal of the object.

ANS: B A small foreign body that is not corrosive or sharp and that has reached the stomach is most likely to continue to pass through the GI tract and no intervention is necessary. Inducing vomiting increases the risk of aspiration of the FB. NG tube removal and endoscopy are not indicated unless the object has the potential to damage the GI tract; most objects that are not sharp or corrosive that have reached the abdomen will pass through without causing damage.

6. A 2-month-old infant has a staccato cough and fever. Which aspect of the history is most important in determining the diagnosis? a. Day care attendance b. Immunization history c. Medication history d. Past medical history

ANS: B A staccato cough may be present with pertussis, which is a vaccine-preventable disease. Careful assessment of immunization history is important when this is suspected. Day care attendance is an important aspect of determining exposure and may be considered, but it is not the most important part of the history in this case. Medication and past medical history are probably not relevant in this case since it is less likely that a 2-month-old infant has been taking medications or has a chronic or recurrent illness.

11. The primary care pediatric nurse practitioner is performing a well baby examination on a newborn whose mother is 17 years old. The mother states that she is living with her parents and plans to finish high school. The maternal grandmother will care for the infant while she is in school. What will the nurse practitioner discuss with this mother at this visit? a. Early child intervention programs b. Her needs for socialization with peers c. Immunizations and well child visits d. Referral to a community health nurse

ANS: B Adolescent mothers face problems inherent when this major role is assumed before they are developmentally ready themselves. Their developmental needs may sometimes be in conflict with their children. Although this mother has support from her family, the PNP should assess her desires to socialize with her peers to evaluate her developmental needs. Early child intervention may be necessary when the child is preschool age and immunizations and well child visits should be discussed, but these do not take precedence over the need to evaluate a potential role conflict. Referrals to community health may be necessary if problems arise.

1. 21. The primary care pediatric nurse practitioner is managing care for a child who has JIA who has a positive ANA. Which specialty referral is critical for this child? a. a. Cardiology b. b. Ophthalmology c. c. Orthopedics d. d. Pain management

ANS: B An ophthalmology consultation is critical for children with JIA who have a positive ANA. Uveitis occurs in up to 35% of children with JIA who have a positive ANA. Other specialists may be consulted for specific symptoms.

34. A 7-year-old child whose immunizations are up-to-date has a fever, headache, stiff neck, and photophobia. What course of treatment is indicated? a. Empiric treatment with oral antibiotics or intramuscular ceftriaxone b. Hospitalization for diagnosis and treatment with antibiotics c. Immediate vaccination with meningococcal vaccine d. Outpatient lab work, including a CBC and blood and CSF cultures

ANS: B Any child suspected of having meningococcal meningitis should be hospitalized immediately with IV antibiotics started pending cultures. Empiric treatment is not indicated. Vaccination is not helpful once the disease has started. Outpatient lab work is not indicated.

6. The parents of a formula-fed newborn report that they get their drinking water from a well. What will the primary care pediatric nurse practitioner recommend to provide adequate fluoride for this infant? a. Giving the infant a fluoride supplement b. Testing the fluoride level of their water source c. Using bottled water to prepare the infant's formula d. Using powdered formula with added fluoride

ANS: B Before adding any supplemental fluoride to the infant's formula, the water source should be tested for fluoride. Too much fluoride can cause fluorosis, and giving fluoride when the water supply of it is adequate can cause this.

17. A 10-year-old is hit in the head with a baseball during practice and is diagnosed with concussion, even though no loss of consciousness occurred. The primary care pediatric nurse practitioner is evaluating the child 2 weeks after the injury and learns that the child is still experiencing some sleepiness every day. The neurological exam is normal. The child and the parent are adamant that the child be allowed to return to play baseball. What will the nurse practitioner recommend? a. Continuation of cognitive rest only b. Continuation of physical and cognitive rest c. Continuation of physical rest only d. Returning to play

ANS: B Both physical and cognitive rest is indicated after diagnosis of concussion in youth, particularly if symptoms continue following injury. Cognitive recovery may lag behind physical recovery and is a key factor in return-to-play decisions. Only after all symptoms resolve may athletes progress through steps to gradually return to play.

5. The mother of a 3-month-old child tells the primary care pediatric nurse practitioner that it is "so much fun" now that her infant coos and smiles and wants to play. What is important for the nurse practitioner to teach this mother? a. Appropriate ways to stimulate and entertain the infant b. How to read the infant's cues for overstimulation c. The importance of scheduling "play dates" with other infants d. To provide musical toys to engage the infant

ANS: B By 3 months, infants demonstrate a social smile and will become more active, alert, and responsive. Parents may mistakenly assume that the infant can handle more activity and stimulation when this occurs, and the PNP should teach caregivers how to recognize infant cues for the need to rest or to have decreased stimulation.

4. During a well-baby assessment on a 1-week-old infant who had a normal exam when discharged from the newborn nursery 2 days prior, the primary care pediatric nurse practitioner notes moderate eyelid swelling, bulbar conjunctival injections, and moderate amounts of thick, purulent discharge. What is the likely diagnosis? a. Chemical-induced conjunctivitis b. Chlamydia trachomatis conjunctivitis c. Herpes simplex virus (HSV) conjunctivitis d. Neisseria gonorrhea conjunctivitis

ANS: B C. trachomatis conjunctivitis usually begins between 5 to 14 days of life and causes moderate eyelid swelling, palpebral or bulbar conjunctivitis, and moderate, thick, purulent discharge. Chemical-induced conjunctivitis manifests as nonpurulent discharge. HSV is characterized by serosanguinous discharge. N. gonorrhea causes acute conjunctival inflammation and excessive purulent discharge.

3. A school-age child has begun refusing all cooked vegetables. What will the primary care pediatric nurse practitioner recommend to the parent? a. Allow the child to make food choices since this is usually a phase b. Ensure that the child has three nutritious meals and two nutritious snacks each day c. Prepare vegetables separately for the child to encourage adequate intake d. Teach the child how important it is to eat healthy fruits and vegetables

ANS: B Children have food jags that are generally self-limited. The parent's responsibility is to provide three nutritious meals and two nutritious snacks each day so that all available choices are acceptable. Allowing food choices may result in an overabundance of non-nutritious foods selected. It is not necessary to prepare separate dishes for a child who is going through a temporary phase. Teaching the child about nutrition is important but will not likely have much impact during this phase.

6. The primary care pediatric nurse practitioner is performing a well child exam on a 24-month-old child. The parent tells the nurse practitioner that the child is being toilet trained and expresses frustration that on some days the child uses the toilet every time and on other days not at all. What will the nurse practitioner do? a. Advise the parent to make the child get clean clothes after an accident. b. Ask the parent about the child's toilet habits and understanding of toilet training. c. Recommend using an awards system to encourage toilet use. d. Suggest that the parent place the child on the toilet at predictable intervals.

ANS: B Children often will not toilet train easily if the process is started too early. The PNP should assess toilet training readiness to see if the child is ready. If the child is not ready, toilet training can be very stressful to both the child and the parent(s), so the PNP should not make recommendations that add to this stress.

12. A middle-school-age child is skipping school frequently and getting poor grades since the child's father was killed while deployed in the military. How will the primary care pediatric nurse practitioner manage this situation? a. Prescribe short-term antidepressants for this situational depression. b. Refer the child to a mental health specialist for evaluation and treatment. c. Schedule extended appointments for counseling and mental health interventio. d. Suggest that the child have close follow-up by a school counselor.

ANS: B Children who are experiencing enduring problems, such as the loss of a parent, should be treated either in consultation with or by referral to a pediatric mental health specialist. Antidepressants and other pharmacotherapeutic agents should never be used without a thorough mental health evaluation. The PNP is not qualified to manage this in a primary care setting without consultation or referral, nor is a school counselor, although both may be part of the team of professionals who help to manage this child.

1. A parent who encourages competitiveness in a child who excels at a single sport but not in others may also encourage a sense of a. competence. b. insecurity. c. significance. d. worthiness.

ANS: B Children who gain praise for external measures, such as performance of a sport, may end up unduly comparing themselves with others and feel insecure, inferior, and inadequate, even as they continue to excel in this sport. Competence comes from feeling capable and confident and able to approach new tasks. Significance comes from having a sense of belonging and being accepted unconditionally

1. 1. The primary care pediatric nurse practitioner is evaluating a school-age child who, after removal of a pituitary tumor, has altered hypothalamic control over hunger and satiety. The child is morbidly obese and expresses feeling depressed because of the obesity. What will the nurse practitioner recommend? a. a. Developing a system to reward compliance with a dietary regimen b. b. Restricting all access to food in the house and at school c. c. Suggesting an after-school exercise program to help with weight loss d. d. Using a food diary to track all calories and food intake

ANS: B Children with brain dysfunction affecting hypothalamic control may need rigid restrictions of access to all foods in refrigerators, cupboards, and even garbage cans

15. The parent of a toddler diagnosed with grade V vesicoureteral reflux asks the primary care pediatric nurse practitioner how the disease will be treated. What will the nurse practitioner tell this parent? a. That long-term antibiotic prophylaxis will prevent scarring b. That surgery to correct the condition is possible c. that the child will most likely require kidney transplant d. that the condition will probably resolve spontaneously

ANS: B Children with grade V VUR generally do not experience spontaneous resolution and will likely have to have surgery to correct the condition. Antibiotic prophylaxis will not necessarily prevent scarring. Unless scarring occurs and is severe, kidney transplantation is not likely.

7. The parent of a child who has asthma asks the primary care pediatric nurse practitioner about whether the child may engage in strenuous exercise. What will the nurse practitioner tell the parent? a. Children with asthma should be excluded from vigorous exercise and most strenuous sports. b. Children with asthma show improved aerobic and anaerobic fitness with moderate to vigorous/physical activity. c. Physical activity has been shown to improve overall pulmonary function in children with asthma. d. Vigorous exercise helps improve symptoms in children with poorly controlled asthma.

ANS: B Children with mild or well-controlled asthma may participate in moderate to vigorous sports and show benefits to aerobic and anaerobic fitness, which helps lung function and overall health outcomes. It is not necessary to exclude children with asthma from sports as long as symptoms are well controlled. Overall pulmonary function does not substantially improve with exercise. Children with poor control should not engage in sports until symptoms are under control.

10. The parents of a 4-year-old boy are concerned because he has begun twisting and pulling out his hair, especially when he is tired or stressed. What will the primary care pediatric nurse practitioner recommend as part of an initial approach to treat this behavior? a. Consultation with a pediatric behavioral specialist b. Cutting his hair so that it is too short to pull c. Long-term anti-streptococcal prophylaxis d. Medication with risperidol or clonidine

ANS: B Children with mild to moderate tic disorders can be managed with cognitive-behavioral therapy

8. A toddler has begun hitting and biting other children at a day care center and is exhibiting temper tantrums and bad language at home. The parent reports that these behaviors began shortly after a sibling was born. What will the primary care pediatric nurse practitioner do? a. Advise the parent that the child is exhibiting early symptoms of ADHD. b. Engage the parent in positive parenting strategies to facilitate appropriate child coping. c. Recommend evaluating the child for conduct or oppositional defiant disorder. d. Suggest putting the child in another day care center to ameliorate the problems.

ANS: B Children with social aggression may exhibit the behaviors described above. When social aggression is a response to acute stress, such as the birth of a sibling, the problem usually resolves if parents use positive parenting strategies and facilitate developmentally appropriate child coping efforts. These are not symptoms of ADHD. Conduct disorder (CD) symptoms usually manifest in the preschool age. Oppositional defiant disorder (ODD) is characterized by disobedience rather than aggressiveness. Moving the child to another day care or school does not solve the problem.

17. A 5-year-old child has an elevated blood pressure during a well child exam. The primary care pediatric nurse practitioner notes mottling and pallor of the child's feet and lower legs and auscultates a systolic ejection murmur in the left infraclavicular region radiating to the child's back. The nurse practitioner will suspect which condition? a. Aortic stenosis b. Coarctation of the aorta c. Patent ductus arteriosus d. Pulmonic stenosis

ANS: B Coarctation of the aorta may not have symptoms until later childhood and may present as high blood pressure in the upper extremities and poor perfusion in the lower extremities. A systolic ejection murmur at the left infraclavicular region with transmission to the back is characteristic. Aortic stenosis is characterized by a louder, harsh systolic crescendo-decrescendo murmur at the upper right sternal border with radiation to the neck, LLSB, and apex. PDA has a machinery-like murmur. Pulmonic stenosis may be asymptomatic or may have a harsh, late systolic ejection murmur at the upper left sternal border that transmits to both lung fields.

11. An adolescent is diagnosed with major depression, and the mental health specialist has prescribed fluoxetine. What other treatment is important to protect against suicide risk? a. Addition of risperidone therapy b. Cognitive-behavioral therapy c. Family therapy d. Hospitalization

ANS: B Cognitive-behavioral therapy appears to have a protective effect against suicide and the best treatment responses come from combinations of cognitive-behavioral therapy and SSRIs. Risperidone and other antipsychotics are used if psychosis is present to control those symptoms. Family therapy is useful but does not add protection from suicide. Hospitalization is not the first-line treatment and is used for severe exacerbations or suicide attempts.

15. The primary care pediatric nurse practitioner is examining a 17-year-old male who is on his high school swim team. The adolescent is concerned about "lumps" on his chest. The nurse practitioner notes a marked increase in weight since the last visit along with worsening of the adolescent's acne. Given this set of symptoms, which performance-enhancing substance will the nurse practitioner be most concerned about and ask about? a. Creatine b. Dehydroepiandrosterone (DHEA) c. Ephedra d. Growth hormone

ANS: B DHEA is a prohormone that is converted to either testosterone or estrone and will cause adverse changes similar to anabolic steroids, such as increased weight, gynecomastia, and acne. Creatine is taken because athletes believe it enhances endurance. Side effects include weight gain but not androgenic effects such as gynecomastia or acne. Ephedra is similar to amphetamine, with most side effects related to the heart, such as tachycardia and arrhythmias. Growth hormone will cause increased weight and has side effects associated with diabetes, cardiomyopathy, hepatitis, and renal failure.

3. A child with cerebral palsy receives all nutrition via gastrostomy tube. What will the primary care pediatric nurse practitioner recommend to promote dental health in this child? a. Applying topical iodine every month b. Daily chlorhexidine gluconate rinses c. Ordering medications to prevent drooling d. Prescribing prophylactic antibiotics

ANS: B Daily chlorhexidine gluconate rinses are recommended for children with special health care needs to promote adequate oral hygiene and prevent caries. Topical iodine may be useful but is applied every 4 to 6 months. Medications that dry up salivary secretions make the problem worse. Prophylactic antibiotics are not indicated.

11. The primary care pediatric nurse practitioner performs a developmental assessment on a 3-year-old child and notes normal cognitive, fine-motor, and gross-motor abilities. The child responds appropriately to verbal commands during the assessment but refuses to speak when asked questions. The parent tells the nurse practitioner that the child talks at home and that most other adults can understand what the child says. The nurse practitioner will : a. ask the parent to consider a possible speech delay and report any concerns. b. continue to evaluate the child's speech at subsequent visits. c. refer the child for a speech and hearing evaluation. d. tell the parent to spend more time in interactive conversations with the child.

ANS: B Development should be monitored over time and within the context of the child's overall well-being, rather than at an isolated testing session. The child has normal development in observed measures and appears to hear and understand well. By parental report, the child is able to speak. The PNP should continue to evaluate speech over time, since this refusal to speak may be associated with shyness or intimidation in the clinic. It is not necessary to tell the parent that the child has a possible speech delay. Unless an actual speech delay is observed, a referral is not indicated, nor is it necessary to implement a home therapy.

5. Which recommendation will a primary care pediatric nurse practitioner make when parents ask about ways to discipline their 3-year-old child who draws on the walls with crayons? a. Give the child washable markers so the drawings can be removed easily. b. Provide a roll of paper for drawing and teach the child to use this. c. Put the child in "timeout" each time the child draws on the walls. d. Take the crayons away from the child to prevent the behavior.

ANS: B Discipline involves training or education that molds appropriate behavior and is used to teach the child what is permitted and encouraged. Providing an appropriate outlet for drawing helps to teach the child where to use the crayons. Using washable markers allows the parents to clean the walls but does not teach the child appropriate behaviors. Timeout and taking away the crayons are forms of punishment, or a loss of privileges, that are administered as a form of retribution.

5. An adolescent has suspected infectious mononucleosis after exposure to the virus in the past week. The primary care pediatric nurse practitioner examines the adolescent and notes exudate on the tonsils, soft palate petechiae, and diffuse adenopathy. Which test will the primary care pediatric nurse practitioner perform to confirm the diagnosis? a. Complete blood count b. EBV-specific antibody testing c. Heterophile antibody testing d. Throat culture

ANS: B EBV-specific IgG antibody testing is the specific serologic test for EBV infection. Heterophile antibody testing can be helpful in school-age children and adolescents after the first week of infection. A CBC can identify lymphocytosis with atypical lymphocytes but is non-specific. A throat culture is performed to identify bacterial causes; however, in this case of known exposure to EBV this would not be the appropriate confirmatory test.

29. A toddler is receiving long-term antibiotics to treat osteomyelitis. Which laboratory test will the primary care pediatric nurse practitioner order to monitor response to therapy in this child? a. Blood cultures b. Erythrocyte sedimentation rate (ESR) c. Serum procalcitonin (Pro-CT) d. White blood count (WBC)

ANS: B ESR is a late measure of inflammation and is useful in helping monitor response to therapy, especially when long-term antibiotics are used. Blood cultures will most likely be negative during antibiotic treatment. Pro-CT is used to differentiate viral from bacterial infections. The WBC will most likely decrease over time with a treated infection and is not useful for monitoring response to therapy.

1. 2. The parent of a newborn reports using echinacea for family members to help treat viral illnesses and feels that it is usually effective. What will the primary care pediatric nurse practitioner tell this parent? a. a. That echinacea has no known therapeutic effects and should not be given b. b. That the supplement should not be given to children under 2 years of age c. c. To give half the recommended adult dose until the child is 5 years old d. d. To wait until the infant is at least 1 month old before giving this product

ANS: B Echinacea use should be restricted to children older than 2 years. Telling a parent who is convinced that a product works that it is not effective will prevent open communication between the parent and practitioner. Children under 2 years should not receive the supplement.

14. The primary care pediatric nurse practitioner is examining an infant who has otitis media and learns that the mother and child are homeless. Besides assisting the mother to obtain medication to treat this illness, what is a priority during this visit? a. Assisting the mother to obtain transportation for health care needs b. Determining well child examination history and immunization status c. Making sure the family has access to WIC and food stamps resources d. Obtaining a tuberculosis skin test and scheduling a return office visit

ANS: B Even if a health care visit is in response to a crisis that cannot be denied, assessment of homeless patients should include well child care and immunizations on the operating principle that every child should receive the maximum health care possible. The other options may be part of ongoing assistance and evaluation of homeless families, but the initial response should be evaluation of well child needs.

1. 3. The primary care pediatric nurse is performing a well child examination on an adolescent who was adopted as a toddler. The parent reports that the child had been removed from an abusive home at age 3 years. What will the nurse practitioner evaluate in light of possible long-term effects of this early situation? a. a. Cognitive and psychosocial development b. b. Mental health and suicide risk c. c. Moral development and conscience formation d. d. Spirituality, faith, and religious affiliation

ANS: B Findings of early research suggest that epigenetic changes may mediate the effect of prenatal and infant environment on mental health and disease in older children and adults. Adult suicide victims who were abused as children show decreased levels of a genetic marker of a stress reduction gene, suggesting that child abuse may have an epigenetic effect leading to prolonged stress and mental disorders.

3. The parents of a pre-pubertal female who is on the local swim team tell the primary care pediatric nurse practitioner that their daughter wants to begin a strength training program to help improve her swimming ability. What will the nurse practitioner recommend? a. Avoiding strength training programs until after puberty to minimize the risk for injury b. Enrolling their daughter in a program that uses fixed weight machines or resistance bands c. Having their daughter participate in weight training 4 or 5 times each week for maximum effect d. Making sure that their daughter begins with the greatest weight tolerable using lower repetitions

ANS: B Fixed weights or resistance bands are recommended for pre-pubertal youth to help prevent injury. Strength training prior to menarche helps to strengthen long bones and is considered beneficial. Weight training should be 2 to 3 times weekly with a day in between sessions. Initially, youth should begin with a low number of sets and low intensity.

11. A sexually active adolescent female tests positive for N. gonorrhoeae and C. trachomatis. She tells the primary care pediatric nurse practitioner that she wants to be treated today since she is moving out of town the next day. What will the nurse practitioner order? a. Azithromycin 1 g PO in a single dose b. Ceftriaxone 250 mg IM and azithromycin 1 g PO one time each c. Doxycycline 100 mg PO bid for 7 days d. Erythromycin base 500 mg PO qid for 7 days

ANS: B Follow-up cultures for gonorrhea are required unless ceftriaxone is used. Azithromycin is used to treat both gonorrhea and chlamydia. The other options involve twice daily and four times daily dosing, which present difficulties with compliance.

1. 9. A child is brought to the clinic after falling from a swing and scraping both knees and hands. An examination reveals abraded skin with oozing serous fluid and blood, along with dirt and grime from the playground surface. What will the primary care pediatric nurse practitioner do to minimize the risk of infection? a. a. Apply povidone-iodine to all areas. b. b. Irrigate gently with normal saline. c. c. Rinse with hydrogen peroxide. d. d. Scrub the abraded areas with alcohol.

ANS: B Gentle irrigation with water or normal saline is the preferred method for cleaning a wound. Povidone-iodine, alcohol, and hydrogen peroxide should not be used on open wounds.

6. During a well baby exam on a 9-month-old infant, the parent reports that the baby always uses the left hand to pick up objects and asks if the baby will be left-handed. What will the primary care pediatric nurse practitioner do? a. Explain that it is too soon to tell which hand the infant will prefer later. b. Perform a careful assessment of fine and gross motor skills. c. Teach the parent to encourage the infant to use both hands. d. Tell the parent that a hand preference usually develops between 6 and 12 months.

ANS: B Hand preference before 1 year of age is usually suspect for cerebral palsy and may indicate a lack of motor skills in the other hand. The PNP should perform a careful assessment of fine and gross motor skills. Infants should not exhibit a hand preference until after 1 year of age, so the correct response is to assess further.

1. 14. An 8-year-old boy has a recent history of an upper respiratory infection and comes to the clinic with a maculopapular rash on his lower extremities and swelling and tenderness in both ankles. The pediatric nurse practitioner performs a UA, which shows proteinuria and hematuria and diagnoses HSP. What ongoing evaluation will the nurse practitioner perform during the course of this disease? a. a. ANA titers b. b. Blood pressure measurement c. c. Chest radiographs d. d. Liver function studies

ANS: B Hypertension is a serious risk of HSP, so repeated BP measurement is indicated. ANA titers are not measured with HSP. Chest radiographs are performed only if indicated. LFTs are not indicated

12. A child with a history of a pustular rash at the site of a cat scratch on one arm now has warm, tender, swollen axillary lymph nodes on the affected side. The primary care pediatric nurse practitioner notes induration and erythema of these nodes. What will the nurse practitioner do? a. Obtain a complete blood count and C-reactive protein. b. Order an immunofluorescent assay (IFA) for serum antibodies. c. Perform a needle aspiration of the affected lymph nodes. d. Prescribe a 5-day course of azithromycin.

ANS: B IFA shows a good correlation with cat-scratch fever disease and is useful for a more definitive diagnosis. A complete blood count and C-reactive protein are non-specific indicators of disease. Needle aspiration is only necessary to determine whether local lymph nodes are infected. Antibiotics are not given unless nodes are infected.

10. When performing a neurologic exam to assess for meningeal signs in an infant, the primary care pediatric nurse practitioner will attempt to elicit the Kernig sign by a. bending the infant at the waist to touch fingers to toes. b. extending the leg at the knee with the infant supine. c. flexing the infant's neck to touch chin to chest. d. turning the infant's head from side to side.

ANS: B In an infant, the Kernig sign is elicited by extending the leg at the knee with the infant in a supine position while observing for facial grimacing. Older children can bend at the waist to touch the toes to elicit the Kernig sign. The Brudzinski sign is elicited by passively flexing the neck to cause the patient to spontaneously flex the hip and knees. Turning the infant's head from side to side is not done to elicit either sign.

11. A child is diagnosed with nephrotic syndrome, and the pediatric nurse practitioner provides primary care in consultation with a pediatric nephrologist. The child was treated with steroids and responded well to this treatment. What will the nurse practitioner tell the child's parents about this disease? a. "Future episodes are likely to have worse outcomes." b. "Steroids will be used when relapses occur." c. "This represents a cure from this disease." d. "Your child will need to take steroids indefinitely."

ANS: B In situations in which a child responds well to steroids, this shows promise of a good prognosis, indicating that the child may be treated successfully with steroids during future anticipated relapses. The fact that a child is a "steroid responder" indicates that future episodes of treatment will be successful and have positive outcomes. This disease is chronic and not curable. Steroid use with children who respond positively is intermittent during episodes of relapse. Steroids are not given continuously and are not seen as prophylactic.

7. The primary care pediatric nurse practitioner provides patient teaching for children newly diagnosed with irritable bowel syndrome (IBS). At which stage of development will children be able to understand the link between stress and the symptoms of the disease? a. Concrete-operational stage b. Formal-operational stage c. Pre-conceptual stage d. Sensorimotor stage

ANS: B In the formal-operational stage, children can understand how the body works and how illness may be related to the body and the environment. It is late in this stage when children can conceptualize the mind and body interactions of illness. In the concrete-operational stage, children do not yet distinguish between the body and the mind. The pre-conceptual stage is characterized by a perception that people must be near the cause of an illness. The sensorimotor stage, causes are not considered.

1. The primary care pediatric nurse practitioner sees a 12-month-old infant who is being fed goat's milk and a vegetarian diet. The child is pale and has a beefy-red, sore tongue and oral mucous membranes. Which tests will the nurse practitioner order to evaluate this child's condition? a. Hemoglobin electrophoresis b. RBC folate, iron, and B12 levels c. Reticulocyte levels d. Serum lead levels

ANS: B Infants and children who are fed goat's milk or who are on a strict vegetarian diet are at risk for folic acid and vitamin B12 deficiency. These should be evaluated, along with iron, to rule out IDA. Hemoglobin electrophoresis is used to evaluate diseases associated with altered hemoglobin, such as beta-thalassemia and sickle cell anemia, neither of which is indicated by this child's history. Reticulocyte levels are evaluated to evaluate transient erythroblastopenia of childhood, a condition that frequently follows a viral infection. Serum lead levels are not indicated based on this history.

1. 6. The primary care pediatric nurse practitioner provides anticipatory guidance for a 6-month-old infant who is breastfed who takes 400 IU of vitamin D daily. The parent reports that the infant has begun taking cereals, fruits, and vegetables in addition to nursing. What will the nurse practitioner recommend to promote healthy nutrition? a. a. Begin supplementing with iron. b. b. Continue to nurse as long as desired. c. c. Discontinue the vitamin D supplement. d. d. Stop breastfeeding at 1 year of age.

ANS: B Infants who are breastfed should exclusively nurse until age 6 months and then may continue breastfeeding, supplemented with appropriate foods for 1 year or longer as long as desired by both infant and mother. It is not necessary to supplement with iron unless there is a documented iron deficiency. Vitamin D supplements should continue for all breastfed infants until 1 year old.

8. The primary care pediatric nurse practitioner provides primary care for a 4-month-old infant who has a ventricular septal defect. The infant has been breastfeeding well but in the past month has dropped from the 20th percentile to the 5th for weight. What will the nurse practitioner recommend? a. Adding solid foods to the infant's diet to increase caloric intake b. Fortifying breast milk to increase the number of calories per ounce c. Stopping breastfeeding and giving 30 kcal/ounce formula d. Supplementing breastfeeding with 24 kcal/ounce formula

ANS: B Infants with heart defects who have CHF may need modification of formula or breast milk to increase calories. This infant is nursing well, so fortifying the breast milk to increase calories is the first and best option. Adding solids does not significantly increase caloric intake.

15. The primary care pediatric nurse practitioner manages care in conjunction with a pediatric pulmonologist for a child with cystic fibrosis. Which medication regimen is used to facilitate airway clearance for this child? a. Ibuprofen and azithromycin b. Inhaled dornase alfa c. Ivacaftor d. Prophylactic clindamycin

ANS: B Inhaled dornase alfa is given to promote airway clearance by reducing mucus viscosity. Ibuprofen and azithromycin is given to reduce chronic airway inflammation. Ivacaftor is given to patients with specific gene mutations. Antibiotic therapy is based on regular sputum cultures.

8. A child is diagnosed with Crohn disease. What are likely complications for this child? a. Cancer of the colon and possible colectomy b. Intestinal obstruction with scarring and strictures c. Intestinal perforation and hemorrhage d. Liver disease and sepsis

ANS: B Intestinal obstruction with scarring and strictures are the major complications of CD. The other answers describe complications of ulcerative colitis.

11. A complete blood count on a 12-month-old infant reveals microcytic, hypochromic anemia with a hemoglobin of 9.5 g/dL. The infant has mild pallor with no hepatosplenomegaly. The primary care pediatric nurse practitioner suspects a. hereditary spherocytosis. b. iron-deficiency anemia. c. lead intoxication. d. sickle-cell anemia.

ANS: B Iron-deficiency anemia is the most common type of anemia in infants and children, accounting for approximately 90% of cases. It is characterized by decreased hemoglobin, with microcytic, hypochromic RBCs. Hereditary spherocytosis is characterized by pallor and jaundice with splenomegaly. Lead intoxication is accompanied by neurobehavioral problems. Sickle-cell anemia involves the presence of HgbS.

4. A 2-year-old child has an acute diarrheal illness. The child is afebrile and, with oral rehydration measures, has remained well hydrated. The parent asks what can be done to help shorten the course of this illness. What will the primary care pediatric nurse practitioner recommend? a. Clear liquids only b. Lactobacillus c. Loperamide d. Peppermint oil

ANS: B Lactobacillus, given early in a viral diarrheal illness, can decrease the duration of diarrhea by about 25 hours and is safe to use in children. Parents should begin refeeding early to stimulate enterocyte growth and help facilitate mucosal repair. Loperamide may be given to children over the age of 3 years. Peppermint oil may help reduce cramping, but its efficacy is not certain.

10. The primary care pediatric nurse practitioner is performing a well child examination on a school-age child who has a history of cancer treated with cranial irradiation. What will the nurse practitioner monitor in this child? a. Cardiomyopathy and arrhythmias b. Leukoencephalopathy c. Obesity and gonadal dysfunction d. Peripheral neuropathy and hearing loss

ANS: B Leukoencephalopathy is a late effect of cancer treatment associated with cranial irradiation. Cardiomyopathy and arrhythmias are related to anthracycline use. Obesity and gonadal dysfunction result from neuroendocrine effects of chemotherapeutic agents. Peripheral neuropathy and hearing loss occur after cisplatin use.

12. The primary care pediatric nurse practitioner is examining a young child who has cerebral palsy. Which part of the family history raises concerns about potential child maltreatment? a. Child attends day care b. Limited financial resources c. Mother works outside the home d. No membership in a church

ANS: B Limited financial resources can put a strain on caring for a child with special needs whose medical needs are expensive. The fact that the mother works outside the home and the child attends day care may actually provide some respite from the strain of caring for a special needs child. Families may have strong spiritual values whether they attend church or not.

1. 6. A child has wheat allergies and continues to have problems in spite of consuming a diet without breads and cereals. What will the primary care pediatric nurse practitioner caution the parents to look for on product labels that may indicate wheat products are present in foods? a. a. Casein b. b. Malt c. c. Miso d. d. Whey

ANS: B Malt in a product may signal a hyperallergenic food for a child with wheat allergy. Miso contains soy. Casein and whey are dairy products.

1. 2. The primary care pediatric nurse practitioner sees a 6-year-old child after a hospitalization for injuries sustained in a motor vehicle accident (MVA) in which the child's grandfather was killed. The parent states that it is difficult to get the child to stop talking about the accident and is worried that the child will have permanent emotional scars. What will the nurse practitioner suggest? a. a. Assure the child that he is safe and this won't happen again. b. b. Encourage the child to express and examine feelings. c. c. Reassure the child that his grandfather is in heaven. d. d. Redirect these conversations to happier topics.

ANS: B Management goals when health crises occur should focus on helping children and families make sense of the events and to better understand what is happening to help regain control. Helping the child to express and examine feelings will help him to put what happened in perspective. Assuring the child that accidents won't happen again is false and can lead to even more distress in the future. Telling the child that the grandparent is in heaven doesn't address all his fears about the accident. Redirecting the conversation to happier topics minimizes the child's concerns.

26. The parent of an infant asks why some vaccines, such as MMR, are not given along with the other series of immunizations at 2, 4, and 6 months of age. What will the primary care pediatric nurse practitioner tell this parent? a. Febrile seizures are more likely in younger infants with some vaccines. b. Maternal antibodies neutralize some vaccines and are delayed until 12 months. c. The risk of adverse effects is lower for some vaccines after the first year. d. Too many vaccines at once can overwhelm the infant's immune system.

ANS: B Maternal antibodies may neutralize certain vaccines, so some are delayed until the child is 1 year old. Febrile seizures and adverse reactions are not more likely in younger infants. There is no evidence that a large number of vaccines can overwhelm the infant's immune system.

8. An adolescent female has periods every 30 days that are consistently heavy and last from 5 to 8 days. What is her diagnosis? a. Menometrorrhagia b. Menorrhagia c. Metrorrhagia d. Polymenorrhea

ANS: B Menorrhagia is characterized by normal period intervals with excessive flow or duration of menses. Menometrorrhagia involves both excessive bleeding and irregular cycles. Metrorrhagia is irregular frequency with bleeding between cycles. Polymenorrhea is when there are fewer than 21 days between cycles.

1. 9. The primary care pediatric nurse practitioner is performing a well child examination on a 15-year-old girl who consumes a vegan diet. Based on this assessment, which nutrients may this adolescent need to supplement? a. a. Calcium, vitamin C, and vitamin A b. b. Iron, folic acid, and B12 c. c. Magnesium, vitamin E, and zinc d. d. Vitamin D, vitamin C, and phosphorus

ANS: B Menstruating females are at risk for iron deficiency. Women of childbearing age should take folic acid supplements. Children who eat a vegan diet will need B12 supplements. The other options represent nutrients that do not have implications specific to menstruating females or vegans.

7. The primary care pediatric nurse practitioner is performing a well baby examination on a 2-week-old infant. The parent is concerned that the infant sleeps too much. The nurse practitioner asks the parent to keep a sleep log and will teach the parent that which amount of sleep per day is optimal for this infant? a. 10 to 12 hours b. 12 to 15 hours c. 15 to 18 hours d. 18 to 20 hours

ANS: B Newborns sleep a total of 15 to 18 hours per day.

11. The primary care pediatric nurse practitioner is evaluating a heart murmur during a pre-participation examination of a high school athlete. Which finding would be a concern requiring referral to a cardiologist? a. A murmur that is louder when squatting and softer when standing b. A murmur that is quieter when squatting and louder with a Valsalva maneuver c. A murmur with narrow and variable splitting of S2 d. A systolic murmur that is grade 1 or 2

ANS: B Normally, squatting will increase venous return to the heart and cause murmurs to be louder, while standing or performing a Valsalva maneuver will cause murmurs to be quieter. If the reverse is true, then hypertrophic cardiomyopathy or mitral valve prolapse must be ruled out. A murmur with a wide or fixed splitting of S2 must be evaluated. A split S2 that is variable, particularly in synchrony with respirations, is common, and a narrow S2 split is of less concern but should be monitored over time. Systolic murmurs of grade 3 or greater must be evaluated by specialists

4. Which is true about the health status of children in the United States? a. Globalism has relatively little impact on child health measures in the U.S. b. Obesity rates among 2- to 5-year-olds have shown a recent significant decrease. c. The rate of household poverty is lower than in other economically developed nations. d. Young children who attend preschool or day care have higher food insecurity.

ANS: B Obesity rates are a major concern for child health in the U.S. but recently have stabilized in the rate of increase and have declined among 2- to 5-year-olds between 2004 and 2013. Globalism has an increasing effect on child health in the U.S. The rate of household poverty in the U.S. is higher than in other economically developed nations. Young children who attend preschool or day care have lower food insecurity.

1. 11. The primary care pediatric nurse practitioner examines a child who has had stiffness and warmth in the right knee and left ankle for 7 or 8 months but no back pain. The nurse practitioner will refer the child to a rheumatology specialist to evaluate for a. a. enthesitis-related JIA. b. b. oligoarticular JIA. c. c. polyarticular JIA. d. d. systemic JIA.

ANS: B Oligoarticular JIA is characterized by mild, painless asymmetric joint involvement without systemic symptoms. Enthesitis-related JIA involves arthritis of the lower limbs, especially the hips, intertarsal joints, and sacroiliac joints, with swelling, tenderness, and warmth. Polyarticular JIA involves 5 or more joints. Systemic JIA presents with systemic symptoms, such as fever.

5. The primary care pediatric nurse practitioner is performing a well child exam on a 12-month-old infant. The parent tells the nurse practitioner that the infant has predictable bowel and bladder habits and asks about toilet training. What will the nurse practitioner tell this parent? a. It is too early to begin introducing the child to the toilet, and the parent should wait until the child is at least 2 years old. b. Placing the child on a "potty" chair helps the child associate elimination cues with the toilet. c. Predictability of elimination patterns indicates readiness for toilet training, and the parent can begin this process. d. The parent should wait until other signs of toilet training readiness occur before introducing the child to the toilet.

ANS: B Once elimination patterns are predictable, the parent may place the child on a potty chair to help the child associate elimination with the toilet, as long as the parent understands that this is not actual potty training. It is not too early to begin this association exercise. Predictability of elimination patterns does not indicate cognitive readiness for toilet training.

1. 7. The mother of a 15-month-old infant tells the primary care pediatric nurse practitioner that she wishes to continue nursing her child for another year, if possible. What will the nurse practitioner recommend? a. a. Breastfeed only at bedtime to establish meal patterns. b. b. Clean the toddler's teeth each time after breastfeeding. c. c. Offer the breast just prior to meals to maintain milk supply. d. d. The toddler should continue to be breastfed "on demand."

ANS: B One drawback to breastfeeding toddlers is the effect of prolonged contact with lactose on the teeth. Mothers should be cautioned to consult with a dentist and to clean the toddler's teeth. The mother may choose when and how often to breastfeed but should not allow "on demand" feedings. The breast should be offered after meals.

9. A child with a history of otitis externa asks about ways to prevent this condition. What will the primary care pediatric nurse practitioner recommend? a. Cleaning ear canals well after swimming b. Drying the ear canal with a hair dryer c. Swimming only in chlorinated pools d. Using cerumenolytic agents daily

ANS: B Otitis externa is most frequently caused by retained moisture in the ear canal after swimming and when the protective barriers on the skin break down. Drying the ear canals with a hair dryer on a low setting helps to remove the moisture. Cleaning the ear canals, swimming in chlorinated water, and using a cerumenolytic remove the wax that protects the ear canal from superficial infection.

13. A child complains of itching in both ears and is having trouble hearing. The primary care pediatric nurse practitioner notes periauricular edema and marked swelling of the external auditory canal and elicits severe pain when manipulating the external ear structures. Which is an appropriate intervention? a. Obtain a culture of the external auditory canal. b. Order ototopical antibiotic/corticosteroid drops. c. Prescribe oral amoxicillin-clavulanate. d. Refer the child to an otolaryngologist.

ANS: B Ototopical antibiotic/corticosteroid drops are the mainstay of therapy for OE. It is not necessary to obtain a culture unless the infection does not respond to treatment. Oral antibiotics are not indicated unless impetigo occurs and is severe. A referral to a specialist is not recommended.

13. An 18-month-old child has bronchopulmonary dysplasia. To help prevent pneumococcal disease, which vaccine will be ordered? a. PCV7 b. PCV13 c. PCV23 d. PCV33

ANS: B PCV13 is recommended for all children under age 5 years. PCV7 was replaced by PCV13 in 2010. PCV23 is used in children over age 2 years who are at higher risk of pneumococcal disease

13. An 18-month-old child has bronchopulmonary dysplasia. To help prevent pneumococcal disease, which vaccine will be ordered? a. PCV7 b. PCV13 c. PCV23 d. PCV33

ANS: B PCV13 is recommended for all children under age 5 years. PCV7 was replaced by PCV13 in 2010. PCV23 is used in children over age 2 years who are at higher risk of pneumococcal disease. this child will be eligible for the PCV23 at age 2. children younger than 2 years have shown poor immunogenicity to PCV23. PCV33 does not exist.

3. During a well child examination, the primary care pediatric nurse practitioner learns that a 5-year-old child has had several episodes of walking out of the bedroom after falling asleep, looking dazed, with open eyes, and saying things that don't make sense. What will the nurse practitioner recommend? a. Establishing a graduated extinction program and good sleep hygiene b. Making sure that stairs are blocked and doors are locked c. Referral to a sleep disorder clinic for evaluation of a parasomnia d. To awaken the child when these occur and asking about nightmares

ANS: B Parents of children with sleep walking should be assured that this is relatively benign but should make sure the house is secure so the child will not cause self-harm. Graduated extinction and sleep hygiene are used for children who have difficulty initiating or maintaining sleep. Referral to a sleep disorder clinic may be warranted if the child has an episode of leaving the house or some other dangerous activity. The child should be guided back to bed without awakening.

1. 9. An 8-year-old child is diagnosed with systemic lupus erythematosus (SLE), and the child's parent asks if there is a cure. What will the primary care pediatric nurse practitioner tell the parent? a. a. Complete remission occurs in some children at the age of puberty. b. b. Periods of remission may occur but there is no permanent cure. c. c. SLE can be cured with effective medication and treatment. d. d. The disease is always progressive with no cure and no remissions.

ANS: B Periods of remission do occur in some children with SLE for unknown reasons, but there is no permanent remission or cure. For some children with Juvenile Idiopathic Arthritis (JIA), complete remission occurs at puberty.

16. A 9-month-old infant has vesiculopustular lesions on the palms and soles, on the face and neck, and in skin folds of the extremities. The primary care pediatric nurse practitioner notes linear and S-shaped burrow lesions on the parent's hands and wrists. What is the treatment for this rash for this infant? a. Ivermectin 200 mcg/kg for 7 to 14 days, along with symptomatic treatment for itching b. Permethrin 5% cream applied to face, neck, and body and rinsed off in 8 to 14 hours c. Treatment of all family members except the infant with permethrin 5% cream and ivermectin d. Treatment with permethrin 5% cream for 7 days in conjunction with ivermectin 200 mcg/kg

ANS: B Permethrin 5% cream is the drug of choice for treating scabies and is intended for use in infants as young as 2 months of age. Infants will get lesions on the face and neck, and permethrin may be applied to the face, avoiding the eyes. Ivermectin is not recommended for children under 5 years old. Treatment must include the infant as well as all family members whether symptomatic or not.

1. A 3-year-old child has head lice. What will the initial treatment recommendation be to treat this child? a. Lindane b. Permethrin c. Pyrethrin d. Spinosad

ANS: B Permethrin is the treatment of choice for head lice because of its safety and efficacy. Pyrethrin has more treatment failures and is not the first-line treatment. Lindane has neurotoxic side effects and is only recommended when treatment failure occurs. Spinosad is used in children 4 years and older.

1. 2. A school-age child steps on a nail while wearing tennis shoes and develops cellulitis in that foot. The child's immunizations are up-to-date. What antibiotic will the pediatric nurse practitioner empirically prescribe? a. a. Amoxicillin-clavulanate b. b. Ciprofloxacin c. c. Clindamycin d. d. Trimethoprim-sulfamethoxazole

ANS: B Plantar puncture wounds, particularly those wounds that occur following puncture of sneakers/shoes, require ciprofloxacin to cover potential Pseudomonas infection and to protect against an osteomyelitis. Amoxicillin-clavulanate is used in other puncture wounds with signs of infection. Clindamycin is used for similarly wounded children allergic to penicillins. TMP-SMX is used if MRSA is cultured.

22. According to recent research, which populations may have higher rates of under-immunization than others? a. Those with higher rates of Asians b. Those with higher rates of graduate degrees c. Those with lower rates of poverty d. Those with lower rates of primary providers

ANS: B Rates of under-immunization were increased in populations with an increased percentage of graduate degrees in a study of geographic clusters of under-immunized communities in northern California. Populations having a higher percentage of Asians have increased immunization rates. Higher levels of poverty are associated with under-immunization. The study did not look at the effect of the number of primary providers.

5. A dipstick urinalysis is positive for leukocyte esterase and nitrites in a school-age child with dysuria and foul-smelling urine but no fever who has not had previous urinary tract infections. A culture is pending. What will the pediatric nurse practitioner do to treat this child? a. Order ciprofloxacin ER once daily for 3 days if the culture is positive. b. Prescribe trimethoprim-sulfamethoxazole (TMP) twice daily for 3 to 5 days. c. Reassure the child's parents that this is likely an asymptomatic bacteriuria. d. Wait for urine culture results to determine the correct course of treatment.

ANS: B Short-term antibiotics of 3 to 5 days may be as effective for treating UTI in non-febrile bladder infections and TMP is generally a first-line drug in children without history of UTI. Ciprofloxacin is used in adolescents older than 18 years and this child is symptomatic with positive leukocyte esterase and nitrites and will need treatment. Asymptomatic bacteriuria occurs when bacteria are in the urine of a child who is asymptomatic (without symptoms).

1. The primary care pediatric nurse practitioner is performing a well child exam on an 8-year-old girl and notes the presence of breast buds. What will the nurse practitioner include when initiating anticipatory guidance for this patient? a. A discussion about the risks of pregnancy and sexually transmitted diseases b. Information about sexual maturity and menstrual periods c. Material about the human papillomavirus vaccine d. Sexual orientation and the nature of sexual relationships

ANS: B Since this child is 8 years old, it is early to discuss sexual behavior and reproduction given the level of the child's cognition and understanding. However, with these early changes in telearche marking the onset of puberty, it is wise to discuss menstruation in an age-appropriate manner before it occurs so that the child can be prepared. Since this child is showing signs of early puberty, this information can be included in anticipatory guidance.

14. The primary care pediatric nurse practitioner notes a small, round object in a child's external auditory canal, near the tympanic membrane. The child's parent thinks it is probably a dried pea. What will the nurse practitioner do to remove this object? a. Irrigate the external auditory canal to flush out the object. b. Refer the child to an otolaryngologist for removal. c. Remove the object with a wire loop curette. d. Use a bayonet forceps to grasp and remove the object.

ANS: B Spherical objects are the most difficult to remove and should be referred. Irrigation is not recommended for objects made of organic material and also increases the risk of pushing the object farther down.

1. 22. The parent of a school-age child who is diagnosed with oligoarticular JIA asks the primary care pediatric nurse practitioner what exercises the child may do to help reduce symptoms. What will the nurse practitioner recommend? a. a. Running b. b. Swimming c. c. Weights d. d. Yoga

ANS: B Swimming is an excellent exercise for children with JIA because water therapy and the use of heat or cold reduce pain and stiffness, unless they have severe anemia or cardiac involvement.

7. The primary care pediatric nurse practitioner performs a Hirschberg test to evaluate a. color vision. b. ocular alignment. c. peripheral vision. d. visual acuity.

ANS: B The Hirschberg test, or corneal light reflex, assesses ocular mobility and alignment by looking for symmetry of reflected light. Color vision testing is performed with Richmond pseudo-isochromatic plates. Peripheral vision is tested by watching the child's response to objects as they are moved in and out of the visual fields. Visual acuity is performed using eye charts or visual-evoked potential readings.

10. A parent is concerned about vaccine adverse reactions. Based on an Institute of Medicine report, what will the primary care pediatric nurse practitioner tell the parent? a. Administering multiple vaccines may trigger the development of type 1 diabetes. b. The MMR may be linked to febrile seizures in immunocompromised children. c. There is some risk of CNS disorders associated with the hepatitis B vaccine. d. Vaccines containing thimerosol are linked to pervasive developmental disorders.

ANS: B The IOM report found that febrile seizures and measles inclusion body encephalitis can occur in immunocompromised children. The IOM found no substantiated evidence that multiple vaccines trigger type 1 diabetes, hepatitis B vaccine is associated with increased risk of CNS disorders, or thimerosol- containing vaccines are linked to pervasive developmental disorders.

8. The primary care pediatric nurse practitioner is performing a well child examination on a 9-month-old infant whose hearing is normal but who responds to verbal cues with only single syllable vocalizations. What will the nurse practitioner recommend to the parents to improve speech and language skills in this infant? a. Provide educational videos that focus on language. b. Read simple board books to the infant at bedtime. c. Sing to the child and play lullabies in the baby's room. d. Turn the television to Sesame Street during the day.

ANS: B The best way to improve language skills is to read to children. As long as the reading includes positive interactions with the baby and the reader, the baby is learning language. Educational videos, music, and television are all passive media and do not involve this interaction.

11. A child is brought to clinic with several bright red lesions on the buttocks. The primary care pediatric nurse practitioner examines the lesions and notes sharp margins and an "orange peel" look and feel. The child is afebrile and does not appear toxic. What is the course of treatment for these lesions? a. Hospitalize the child for intravenous antibiotics and possible I&D of the lesions. b. Initiate empiric antibiotic therapy and follow up in 24 hours to assess response. c. Obtain blood cultures prior to beginning antibiotic treatment. d. Perform gram stain and culture of the lesions before initiating antibiotics.

ANS: B The child has clinical signs of erysipelas, which is a superficial variant of cellulitis. Because the child is afebrile and doesn't appear toxic, outpatient antibiotics with 24-hour follow-up can be initiated. If the child does not respond or becomes toxic, hospitalization and IV antibiotics are indicated. Blood cultures rarely are positive. Gram stain and cultures are performed if unusual organisms are suspected or if pus is present.

6. An adolescent who is overweight expresses a desire to lose weight in order to participate in sports but tells the primary care pediatric nurse practitioner that he doesn't want to give up sweets and soft drinks because he enjoys them too much. Which stage of change does this represent? a. Action b. Contemplation c. Precontemplation d. Preparation

ANS: B The contemplation stage occurs when the individual is aware that a problem exists and struggles with the costs and energy required for change. This client knows what to do but doesn't want to give up the enjoyment of certain treats. The action phase occurs when behaviors to eliminate the problem occur. The precontemplation stage occurs when the individual wishes to change but is resistant to change without clear reason for being so. The preparation stage occurs when the individual makes small behavior changes, such as giving up sweetened soda first, in preparation for commitment to the actual plan.

18. A child who is immunocompromised has a fever and a rash consisting of macules, papules, and pustules. What will the primary care pediatric nurse practitioner do? a. Administer varicella immune globulin (VariZIG). b. Hospitalize the child for intravenous acyclovir. c. Order intravenous immunoglobulin as an outpatient. d. Prescribe oral acyclovir for the duration of the illness.

ANS: B The description of the rash the immunocompromised child has been exposed to is that of varicella. Intravenous acyclovir should be given to immunocompromised individuals. Immune globulin is not effective after the disease has progressed. Oral acyclovir is expensive and not routinely recommended for most children.

2. An adolescent has right-sided flank pain without fever. A dipstick urinalysis reveals gross hematuria without signs of infection or bacteriuria, and the primary care pediatric nurse practitioner diagnoses possible nephrolithiasis. What is the initial treatment for this condition? a. Extracorporeal shockwave lithotripsy (ESWL) b. Increasing fluid intake up to 2 L daily c. Percutaneous removal of renal calculi d. Referral to a pediatric nephrologist

ANS: B The first line of therapy for all stone types is increasing fluids. ESWL may be indicated if symptoms worsen and stones are not passed. Percutaneous removal of renal calculi and referral to nephrology may be indicated with worsening symptoms.

3. The primary care pediatric nurse practitioner is counseling an obese adolescent whose parents both have type 2 diabetes mellitus. Which health behavior prediction model is useful when the nurse practitioner discusses lifestyle changes with this client? a. Behavioral change model b. Health belief model c. Health promotion model d. Transtheoretical model

ANS: B The health belief model explains behavior used to prevent disease rather than to promote health. Clients need to believe that they are vulnerable to the disease, will have negative consequences if they are affected, and that taking action will reduce the risk. The adolescent who believes that there is a risk of diabetes may be willing to undertake lifestyle changes if taught that these can reduce the risk. The behavioral change model is useful for changing behaviors to promote health. The health promotion model has a focus on health promotion and not disease promotion. The transtheoretical model identifies the stages of change.

10. The primary care pediatric nurse practitioner is performing a focused problem assessment on a child who has asthma and learns that one of the child's parents smokes around the child in spite of being advised against this. The nurse practitioner recognizes this as a possible alteration in which functional health pattern? a. Cognitive-perceptual b. Health perception c. Role-relationship d. Values-beliefs

ANS: B The health perception-health management pattern describes client perceptions of personal health and health behaviors and includes the belief that there is a relationship between health status and health practices. The cognitive-perceptual pattern describes sensory-perceptual and cognition patterns. The role- relationship pattern describes patterns of roles and responsibilities of the client and other family members. The values-beliefs pattern identifies the beliefs that influence daily living, decision making, and meaning of life.

8. The primary care pediatric nurse practitioner performs a well baby assessment of a 5-day-old infant and notes mild conjunctivitis, corneal opacity, and serosanguinous discharge in the right eye. Which course of action is correct? a. Administer intramuscular ceftriaxone 50 mg/kg. b. Admit the infant to the hospital immediately. c. Give oral erythromycin 30 to 50 mg/kg/day for 2 weeks. d. Teach the parent how to perform tear duct massage.

ANS: B The infant has symptoms consistent with HPV conjunctivitis and requires hospitalization for topical and systemic antiviral medications to prevent spread to the central nervous system, mouth, and skin. IM ceftriaxone is given for gonococcal conjunctivitis. Oral erythromycin is given for chlamydial conjunctivitis. Tear duct massage is performed for lacrimal duct obstruction.

13. The primary care pediatric nurse practitioner notes velvety, brown thickening of skin in the axillae, groin, and neck folds of an adolescent Hispanic female who is overweight. What is the initial step in managing this condition? a. Consultation with a pediatric dermatologist b. Performing metabolic laboratory tests c. Prescribing topical retinoic acid cream d. Referral to a pediatric endocrinologist

ANS: B The initial step is to determine whether metabolic syndrome is the underlying cause for these lesions, which, according to the other physical findings, is most likely. If hyperinsulinemia is present, referral to a pediatric endocrinologist is the next step. A dermatology referral is not indicated. Unless the lesions are thick or cause discomfort, prescribing retinoic acid is not necessary.

8. A Somalian immigrant mother is concerned that her 8-year-old child is underweight. The primary care pediatric nurse practitioner notes that the child's weight is at the 25th percentile. After realizing that the mother is comparing her child to a group of American-born children who are overweight, the pediatric nurse practitioner is able to convince the mother that this is a normal weight. Which domain of cultural competence does this represent? a. Global b. Interpersonal c. Intrapersonal d. Organizational

ANS: B The interpersonal domain of cultural competence refers to how cultural competence is manifested between and among individuals and includes all relationships within the health care setting. The PNP becomes aware of cultural norms in body weight and uses this knowledge to discuss healthy weights with the parent. The global domain recognizes a movement toward integration and interconnection of the world population in economic, political, technological, and sociocultural terms. The intrapersonal domain refers to an understanding of the self to understand one's own cultural background. The organizational domain is knowledge of institutional culture and how it affects health care.

10. The primary care pediatric nurse practitioner needs to assess a potential hymenal tear in a prepubertal female who is apprehensive about the exam. Which approach will the nurse practitioner use? a. Have the child sit frog-legged on the parent's lap. b. Place the child in the knee-chest position on the exam table. c. Put the child supine on the exam table with her feet in the stirrups. d. Refer the child for a speculum exam under sedation.

ANS: B The knee-chest position is the best position for noninvasive, internal examination of the vulva and vagina. The frog-leg position does not allow for internal visualization. Putting the child on the exam table with her feet in the stirrups will increase her anxiety. If an exam cannot be performed in the office because of the child's anxiety, sedation may be indicated, but it is not the first choice.

1. 5. The primary care pediatric nurse practitioner is prescribing ibuprofen for a 25 kg child with JIA who has oligoarthitis. If the child will take 4 doses per day, what is the maximum amount the child will receive per dose? a. a. 200 mg b. b. 250 mg c. c. 400 mg d. d. 450 mg

ANS: B The maximum dose is 40 mg/kg/day divided into 3 to 4 doses. 25 kg × 40 mg = 1000/4 = 250 mg.

4. The primary care pediatric nurse practitioner is performing a well child assessment on a 13-year-old female whose mother asks when her daughter's periods may start. Which information will the nurse practitioner use to help estimate the onset of periods? a. The age of the mother's menarche b. The patient's age at thelarche c. When adrenarche occurred d. Whether linear growth has stopped

ANS: B Thelarche, or the development of breast budding, generally precedes menarche by 2.5 years, so this should be determined when attempting to predict this milestone. The age of the mother at menarche is not a reliable indicator. Adrenarche is related to adrenal and not gonadal development and is less valid than other secondary sex characteristics in assessing sexual maturation. Rapid linear growth usually begins after thelarche and peaks about a year later but is not used to predict menarche.

19. A preschool-age child with no previous history has mild flank pain and fever but no abdominal pain or vomiting. A urinalysis is positive for leukocyte esterase and nitrites. A culture is pending. Which is the correct course of treatment for this child? a. Hospitalize for intravenous antibiotics. b. Order amoxicillin clavulanate. c. Prescribe trimethoprim-sulfamethoxazole. d. Refer for a voiding cystourethrogram.

ANS: B These symptoms suggest this young child may have pyelonephritis. Amoxicillin clavulanate may be given to young children with uncomplicated pyelonephritis who are well hydrated with no abdominal pain or vomiting. Hospitalization is not necessary for uncomplicated pyelonephritis in this age child. TMP is not a first-line drug for pyelonephritis. Voiding cystourethrogram is not indicated for a first febrile UTI.

6. The primary care pediatric nurse practitioner sees a 10-year-old child whose parent describes as a "class clown." The child denies having problems at school, but acknowledges poor grades by saying, "I'm not very smart, I guess." When counseling the parent about helping this child deal with this self- perception issue, the nurse practitioner will recommend which strategy? a. Empower the child to make decisions and assume more responsibilities. b. Help the child identify skills and activities that he is good at. c. Spend time each evening helping the child with homework to improve grades. d. Work with the teacher to set appropriate limits on school behavior.

ANS: B This child exhibits problems with personal identity and copes by clowning around to avoid dealing with problems of inferiority about school performance. The parent should work with the child to find areas of strength and help the child become accomplished in those things to improve self-esteem.

6. An 18-month-old child with no previous history of otitis media awoke during the night with right ear pain. The primary care pediatric nurse practitioner notes an axillary temperature of 100.5°F and an erythematous, bulging tympanic membrane. A tympanogram reveals of peak of +150 mm H2O. What is the recommended treatment for this child? a. Amoxicillin 80 to 90 mg/kg/day in two divided doses b. An analgesic medication and watchful waiting c. Ceftriaxone 50 to 75 mg/kg/dose IM given once d. Ototopical antibiotic drops twice daily for 5 days

ANS: B This child has no previous history and only has a mild fever and can be managed by watchful waiting, with parents given instructions about when and why to notify the provider. Analgesia is essential so that the child can be comfortable. If antibiotics are indicated as a result of no improvement after 48 to 72 hours, amoxicillin is the first-line drug. Ceftriaxone is given if the child is vomiting. Topical antibiotics are given when there is a perforation in the tympanic membrane.

5. The primary care pediatric nurse practitioner diagnoses acute otitis media in a 2-year-old child who has a history of three ear infections in the first 6 months of life. The child's tympanic membrane is intact and the child has a temperature of 101.5°F. What will the nurse practitioner prescribe for this child? a. Amoxicillin twice daily for 10 days b. An analgesic medication and watchful waiting c. Antibiotic ear drops and ibuprofen d. Ceftriaxone given once intramuscularly

ANS: B This child has no recent history, is over 24 months, and has relatively mild symptoms, so can be treated by watchful waiting with adequate follow-up and analgesic medication. Antibiotics are not indicated unless the child worsens or does not improve in 48 to 72 hours.

1. 4. A child has a fever and arthralgia. The primary care pediatric nurse practitioner learns that the child had a sore throat 3 weeks prior and auscultates a murmur in the clinic. Which test will the nurse practitioner order? a. a. Anti-DNase B test b. b. ASO titer c. c. Rapid strep test d. d. Throat culture

ANS: B This child has symptoms and a history consistent with ARF. The ASO titer peaks in 3 to 6 weeks and will confirm a recent strep infection. The anti-DNase B test will also confirm a recent strep infection, but this doesn't peak until 6 to 8 weeks after the initial infection. A rapid strep test and throat culture do not differentiate the carrier state from a true infection.

5. A school-age child has recurrent diarrhea with foul-smelling stools, excessive flatus, abdominal distension, and failure-to-thrive. A 2-week lactose-free trial failed to reduce symptoms. What is the next step in diagnosing this condition? a. Lactose hydrogen breath test b. Serologic testing for celiac disease c. Stool for ova and parasites d. Sweat chloride test for cystic fibrosis

ANS: B This child has symptoms consistent with celiac disease, especially FTT and foul-smelling stools. Since the lactose-free trial did not reduce symptoms, the likelihood of lactose intolerance is less and thus testing is not likely to be helpful. The symptoms are recurrent, so giardiasis is less likely. CF is still possible, but most children with CF are diagnosed as infants and have accompanying respiratory symptoms of some type.

1. The parents of an 18-month-old child bring the child to the clinic after observing a brief seizure of less than 2 minutes in their child. In the clinic, the child has a temperature of 103.1°F, and the primary care pediatric nurse practitioner notes a left otitis media. The child is alert and responding normally. What will the nurse practitioner do? a. Order a lumbar puncture, complete blood count, and urinalysis. b. Prescribe an antibiotic for the ear infection and reassure the parents. c. Refer to a pediatric neurologist for anticonvulsant and antipyretic prophylaxis. d. Send the child to the emergency department for EEG and possible MRI.

ANS: B This child has symptoms of a simple febrile seizure with a focal site of infection and an otherwise normal exam. While this is very frightening to the family, the PNP should treat the infection and provide reassurance to the parents. Lumbar puncture may be performed in infants younger than 12 months. Prophylactic medications aren't indicated for febrile seizures. Antipyretics aren't useful, since most seizures occur when the temperature is either rising or falling. EEG and MRI are not indicated when focal neurological signs are not present.

33. A 5-year-old child who received VariZIG after exposure to varicella while immunocompromised during chemotherapy is in the clinic 5 months after stopping chemotherapy for kindergarten vaccines. What will the primary care pediatric nurse practitioner order for this child? a. MMR and Tdap b. MMR, Varivax, Tdap c. Tdap only d. Varivax and Tdap

ANS: B This child is eligible for all three vaccines. Varivax should be given 5 months after VariZIG, unless varicella disease occurred despite VariZIG administration.

4. A 3-year-old child is brought to the clinic by a parent who reports that the child refuses to use the right arm after being swung by both arms while playing. The child is sitting with the right arm held slightly flexed and close to the body. There is no swelling or ecchymosis present. What will the primary care pediatric nurse practitioner do? a. Consider maltreatment as a possible cause of injury. b. Gently attempt a supination and flexion technique. c. Immobilize the arm with a sling and refer to orthopedics. d. Obtain a radiograph of the child's right arm and elbow.

ANS: B This is most likely an annular ligament displacement injury, or "nursemaid's elbow." The primary provider can attempt to reduce the elbow using either a supination/flexion technique or a pronation technique. Consider maltreatment if recurrent dislocations or other symptoms or signs are present. If this fails after three attempts, immobilization and referral are indicated. Radiologic studies are rarely necessary.

21. A 9-month-old infant has a grade III/VI, harsh, rumbling, continuous murmur in the left infraclavicular fossa and pulmonic area. A chest radiograph reveals cardiac enlargement. The primary care pediatric nurse practitioner will refer the infant to a pediatric cardiologist and prepare the parents for which intervention to repair this defect? a. Cardiopulmonary bypass surgery b. Coil insertion in the catheterization laboratory c. Indomethacin administration d. Observation for spontaneous closure

ANS: B This murmur is characteristic of a PDA and, because of cardiac enlargement, represents a larger shunt, requiring repair. Infants older than 8 months of age may have a coil or plug inserted into the shunt in the cardiac catheterization laboratory. Cardiopulmonary bypass surgery is not indicated, even with ligation of the shunt. Indomethacin is administered to premature infants in the early post-natal period and is not useful in term or older infants. Because this infant is symptomatic, observation for spontaneous closure is not recommended.

6. A 12-month-old infant exhibits poor weight gain after previously normal growth patterns. There is no history of vomiting, diarrhea, or irregular bowel movements, and the physical exam is normal. What is the next step in evaluating these findings? a. Complete blood count and electrolytes b. Feeding and stooling history and 3-day diet history c. Stool cultures for ova and parasites d. Swallow study with videofluoroscopy

ANS: B Vomiting, diarrhea, and bowel irregularities are more predictive of organic causes of FTT, which are not present in this infant. A careful history and physical examination and limited laboratory evaluation are the first steps unless there is reason to think that an organic cause is present. The fact that the infant was previously gaining weight appropriately makes a swallowing disorder less likely.

9. The primary care pediatric nurse practitioner is working with a 12-year-old female who has poor diabetes control. The child tells the nurse practitioner that the parent forgets to remind her to check her blood sugars. Which action is correct? a. Assess the parent's knowledge about diabetes management. b. Help the child develop a strategy to remember without parental prompts. c. Refer to a social worker to help the family overcome obstacles to care. d. Remind the child's parent about the importance of good diabetes control.

ANS: B When adherence is a concern, the PNP should work with the family or the child to develop intervention strategies appropriate to the skills, needs, and desires of the parents and children. A 12-year-old child is old enough to take responsibility for checking blood glucose, so the PNP can work with her. The other options may be used if the situation does not improve.

3. The parent of an adolescent female tells the primary care pediatric nurse practitioner that the child may be the victim of cyber-bullying at school but won't talk about it with her parents. What is the nurse practitioner's initial response? a. Ask about the adolescent's school performance and friends. b. Interview the adolescent separately from the parent. c. Reassure the parent that suicide is a rare response to bullying. d. Suggest that the parent discuss this with the school counselor.

ANS: B When bullying is suspected, it is ideal to interview the youth separately from the parent, even though it is necessary to get the history from both, since children are often reluctant to discuss bullying with their parents. Asking about school performance and friends is part of the history but not a priority. Reassuring the parent that suicide is rare minimizes the parent's concerns. Suggesting that the parent discuss this with the school shows an unwillingness to consider the problem as part of the child's overall health.

14. A pre-school age child has honey-crusted lesions on erythematous, eroded skin around the nose and mouth, with satellite lesions on the arms and legs. The child's parent has several similar lesions and reports that other children in the day care have a similar rash. How will this be treated? a. Amoxicillin 40 to 5 mg/kg/day for 7 to 10 days b. Amoxicillin-clavulanate 90 mg/kg/day for 10 days c. Bacitracin cream applied to lesions for 10 to 14 days d. Mupirocin ointment applied to lesions until clear

ANS: B When children have multiple impetigo lesions or non-bullous impetigo with infection in multiple family members or child care groups, oral antibiotics are indicated. Amoxicillin-clavulanate is a first-line drug for this indication. Amoxicillin is not used for skin infections. Bacitracin is bacteriostatic and may be used when only a few lesions are present and if bacterial resistance is not an issue. Mupirocin is used for mild impetigo when the case is isolated.

3. During a well child examination on a 4-month-old infant, the primary care pediatric nurse practitioner evaluates mental health issues. Which statement by the parent indicates a potential problem with the parent-infant relationship? a. "I can sense a difference in my baby's cries." b. "I let my baby cry a while to learn to be patient." c. "My baby prefers to nurse in a darkened room." d. "My baby seems very sensitive to loud noises."

ANS: B When parents respond promptly to their baby's needs, infants develop the ability to wait for care and typically provide less intense distress signals. Parents who are attentive to their babies' signals, preferences, and sensitivities are more in tune with their infants' needs.

15. The primary care pediatric nurse practitioner is teaching a parent of a child with dry skin about hydrating the skin with bathing. What will the nurse practitioner include in teaching? a. Apply lubricating agents at least 1 hour after the bath. b. Have the child soak in a lukewarm water bath. c. Keep the child in the bath until the skin begins to "prune." d. Soaping should be done at the beginning of the bath.

ANS: B When using bathing to hydrate dry skin, lukewarm water should be used. Lubricating agents should be applied immediately after patting the skin dry. The bath should last long enough to allow the skin to become moisturized without becoming supersaturated or "pruned." Soaping and shampooing should be performed at the end of the bath followed by thorough rinsing.

3. The primary care pediatric nurse practitioner is performing a well child exam on a 12-year-old female who has achieved early sexual maturation. The mother reports that she spends more time with her older sister's friends instead of her own classmates. What will the nurse practitioner tell this parent? a. Early-maturing girls need to identify with older adolescents to feel a sense of belonging. b. Girls who join an older group of peers may become sexually active at an earlier age. c. Spending time with older adolescents indicates a healthy adjustment to her maturing body. d. The association with older adolescents will help her daughter to gain social maturity.

ANS: B While it is true that early maturing females may join an older group of peers to feel that they fit in, the ones who do put themselves at risk for risky behaviors, including sexual activity. Although many teens feel awkward when they mature at different rates than their peers, joining a group of older peers demonstrates a poor adjustment and does not promote social maturity.

9. The primary care pediatric nurse practitioner is counseling a parent about bicycle helmet use. The parent reports having a helmet used a year previously by an older child and wonders about using it for a younger child since they are so expensive. What will the nurse practitioner tell the parent? a. "As long as the helmet does not have cracks, you may use it." b. "If the helmet is free from marks, you may use it." c. "You may continue to use a helmet up to 10 years." d. "You should always purchase a new helmet for each child."

ANS: B While parents should be taught not to purchase a secondhand helmet, using a fairly new, undamaged helmet from an older child is acceptable. Any helmet that has marks should be discarded, even if not cracked. Helmets should be replaced every 5 years or sooner, depending on the manufacturer's recommendations. It is not necessary to purchase a new helmet for each child, especially if money is an issue.

2. A school-age child has a fractured wrist with a Salter-Harris Type II fracture, according to the radiologist. What is true about this type of fracture? a. Growth disturbance of the long bones of the arm is likely. b. There is a metaphyseal fragment on the compression side of fracture. c. There is usually a compression or crushing injury to the physis. d. This will require anatomic reduction using an open approach.

ANS: B With a Salter-Harris Type II fracture, a metaphyseal is present on the compression side of the fracture. Types I and II Salter-Harris fractures rarely show growth disturbances. Type V fractures have a compression or crushing injury to the physis. Anatomic reduction with an open approach is usually necessary for a Type III fracture.

1. 4. The parents of a 3-year-old child are concerned that the child has begun refusing usual foods and wants to eat mashed potatoes and chicken strips at every meal and snack. The child's rate of weight has slowed, but the child remains at the same percentile for weight on a growth chart. What will the primary care pediatric nurse practitioner tell the parents to do? a. a. Allow the child to choose foods for meals to improve caloric intake. b. b. Place a variety of nutritious foods on the child's plate at each meal. c. c. Prepare mashed potatoes and chicken strips for the child at mealtimes. d. d. Suggest cutting out snacks to improve the child's appetite at mealtimes.

ANS: B Young children should have three meals and two nutritious snacks each day. The parents' responsibility is to provide nutritious foods and allow children to choose how much they will eat. Children who are allowed to choose foods will likely make selections that are not healthy. Parents should be discouraged from preparing separate meals for their children. Snacks are necessary to maintain adequate intake and energy.

8. An African-American child has recurrent tinea capitis and has just developed a new area of alopecia after successful treatment several months prior. When prescribing treatment with griseofulvin and selenium shampoo, what else will the primary care pediatric nurse practitioner do? a. Monitor CBC, LFT, and renal function during therapy. b. Order oral prednisone daily for 5 to 14 days. c. Perform fungal cultures on family members and pets. d. Prescribe oral itraconazole or terbinafine.

ANS: C Because asymptomatic carriers may be present in the household, family members and pets should be cultured. It is not necessary to monitor lab work with griseofulvin unless there is a change in clinical status, due to the favorable safety profile of griseofulvin. Prednisone is used when severe inflammation is present. Oral itraconazole or terbinafine is used if resistance to griseofulvin occurs; this child has responded to griseofulvin.

10. The primary care pediatric nurse practitioner is concerned that a toddler may have vesicoureteral reflux based on a history of dysfunctional voiding patterns and a series of urinary tract infections. Which intervention is appropriate? a. Initiating a bladder retraining program b. Ordering a voiding cystourethrogram c. Referral to a urologist for evaluation d. Treatment with prophylactic antibiotics

ANS: C If symptomatic vesicoureteral reflux is suspected, the PNP should refer the child to a urologist for diagnosis and initiation of treatment. A bladder retraining program does not treat the underlying cause. The urologist must order the VCUG and will decide if prophylactic antibiotics are indicated

1. 2. The primary care pediatric nurse practitioner performs a developmental assessment on a 32-month-old child. The child's parent reports that about 70% of the child's speech is intelligible. The pediatric nurse practitioner observes that the child has difficulty pronouncing "t," "d," "k," and "g" sounds. Which action is correct? a. a. Evaluate the child's cognitive abilities. b. b. Obtain a hearing evaluation. c. c. Reassure the parent that this is normal. d. d. Refer the child to a speech therapist.

ANS: C Intelligibility of speech reaches about 66% between the ages of 24 and 36 months. Tongue- contact sounds are more intelligible by age 5 years. This child exhibits normal speech for age. It is not necessary to perform a cognitive assessment based on these findings. Referrals for hearing and speech evaluations are not indicated, since these findings are within normal limits.

6. A school-age child falls off a swing and suffers a closed fracture of the right clavicle. How will this be managed? a. Application of a figure-eight clavicle brace for 6 to 8 weeks b. Hospitalization for traction of the affected extremity and shoulder c. Immobilization with a sling to support the affected extremity d. Referral to an orthopedic specialist for possible surgical reduction

ANS: C Most children with fractured clavicle can be treated with sling immobilization for 3 to 4 weeks. Figure- eight clavicle braces are uncomfortable and have questionable effectiveness; they are used if displacement leads to decreased shaft length. Hospitalization for traction is not indicated. Surgical reduction is uncommon and used for open fractures, neurovascular compromise, multiple trauma, rib cage fractures, and severe displacement.

20. The primary care pediatric nurse practitioner auscultates a new grade II vibratory, mid-systolic murmur at the mid sternal border in a 4-year-old child that is louder when the child is supine. What type of murmur is most likely? a. Pathologic murmur b. Pulmonary flow murmur c. Still's murmur d. Venous hum

ANS: C A Still's murmur is characterized by a vibratory or musical low-grade sound, along the sternal border, which is louder when the child is supine or during inspiration. It is usually heard in children between the ages of 2 and 6 years old. Pathologic murmurs are usually harsh, not vibratory. A pulmonary flow murmur has a soft, blowing sound and radiates to the lung fields. A venous hum has a soft, high-pitched swishing sound.

20. The primary care pediatric nurse practitioner is examining a 2-month-old infant with fever and cough. A WBC is 14,000/mm3 and a chest radiograph is normal. The infant is nursing well and having normal stools. What would be an appropriate next step? a. Admitting the infant to the hospital for LP and IV antibiotics b. Obtaining a blood culture, erythrocyte sedimentation rate, and C-reactive protein c. Performing a catheterized urinalysis to screen for leukocytes and nitrites d. Prescribing empiric, broad-spectrum antibiotics with close follow-up

ANS: C A catheterized urinalysis to rule out UTI is appropriate to help determine the cause of infection. The infant has a reassuring WBC and chest X-ray, so it is not necessary to admit to the hospital or to perform blood cultures. Antibiotics are indicated only if bacterial infection is suspected.

1. 11. The primary care pediatric nurse practitioner learns that the mother of a newborn infant is being tested for tuberculosis after a positive TB skin test. What will the nurse practitioner tell the mother who states a desire to breastfeed her baby? a. a. Breast milk is contraindicated if the mother has tuberculosis. b. b. She may continue to nurse her baby since the risk of transmission is low. c. c. That she can express breast milk and feed that to her infant d. d. To give formula until results of tuberculosis testing are known

ANS: C A maternal diagnosis of active, untreated TB is a contraindication to nursing, but expressed breast milk may be fed to the infant. The mother may feed expressed breast milk

31. An 18-month-old child who developed upper respiratory symptoms 1 day prior is brought to the clinic with a high fever, chills, muscle pains, and a dry, hacking cough. A rapid influenza test is negative and a viral culture is pending. What will the primary care pediatric nurse practitioner do? a. Consider therapy with rimantadine. b. Hospitalize for supportive treatment. c. Prescribe oseltamivir and follow closely d. Wait for cultures to determine treatment.

ANS: C A negative rapid viral culture should not be the determining factor when deciding on a clinical course of treatment when influenza is suspected. Children under age 2 years should be treated with antiviral medications. Rimantadine is not recommended unless susceptibility is reliable

31. An 18-month-old child who developed upper respiratory symptoms 1 day prior is brought to the clinic with a high fever, chills, muscle pains, and a dry, hacking cough. A rapid influenza test is negative and a viral culture is pending. What will the primary care pediatric nurse practitioner do? a. Consider therapy with rimantadine. b. Hospitalize for supportive treatment. c. Prescribe oseltamivir and follow closely d. Wait for cultures to determine treatment.

ANS: C A negative rapid viral culture should not be the determining factor when deciding on a clinical course of treatment when influenza is suspected. Children under age 2 years should be treated with antiviral medications. Rimantadine is not recommended unless susceptibility is reliable. cultures will not be confirmed for several days. It is not necessary to hospitalize unless the child has signs of respiratory distress or cardiac involvement. Antivirals should be initiated within 72 hours of onset of symptoms.

6. The primary care pediatric nurse practitioner enters an exam room and finds a 2-month-old infant in a car seat on the exam table. The infant's mother is playing a game on her smart phone. The nurse practitioner interprets this behavior as : a. a sign that the mother has postpartum depression. b. extremely concerning for potential parental neglect. c. of moderate concern for parenting problems. d. within the normal range of behavior in early parenthood.

ANS: C A parent who seems disinterested in a child raises moderate concerns for parenting problems. It does not necessarily signal postpartum depression. It is not a mark for extreme concern. It is not within the expected range of behaviors.

10. The primary care pediatric nurse practitioner evaluates a school-age child whose body mass index (BMI) is greater than the 97th percentile. The nurse practitioner is concerned about possible metabolic syndrome and orders laboratory tests to evaluate this. Which diagnosis will the nurse practitioner document for this visit? a. Metabolic syndrome b. Nutritional alteration: more than required c. Obesity d. Rule out type 2 diabetes mellitus

ANS: C A problem should never be included on the problem list that is not supported by subjective and objective data found and recorded in the database. This child has a BMI that suggests obesity, so this may be used as a diagnosis. Metabolic syndrome is a diagnosis that is determined by laboratory data, which has not been evaluated yet. Nutritional alteration is a NANDA diagnosis and not acceptable for reimbursement. "Rule out" should not be used as a diagnosis, but may be considered part of a plan.

8. The parent of a 4-month-old infant is concerned that the infant cannot hear. Which test will the primary care pediatric nurse practitioner order to evaluate potential hearing loss in this infant? a. Acoustic reflectometry b. Audiometry c. Auditory brainstem response (ABR) d. Evoked otooacoustic emission (EOAE) testing

ANS: C ABR is not a direct measure of hearing but allows for inferences to be made about hearing thresholds and is useful for identifying hearing loss in a young infant. Although sedation is occasionally required, this test is useful in infants and young children unable to cooperate with EOAE or audiometry. Acoustic reflectometry is used to detect middle ear effusion. Audiometry requires a cooperative child. EOAE is used for universal screening in newborns. The American Academy of Pediatrics (AAP) Bright Futures guidelines (AAP, 2014) recommends pure-tone audiometry at 3, 4, 5, 6, 8, 10, 12, 15, and 18 years of age.

9. The primary care pediatric nurse practitioner learns that a school-age child continues to hope that his parents will remarry 1 year after they have divorced. What will the nurse practitioner tell this child's parents? a. "If one of you remarries, he is more likely to understand that this is permanent." b. "This is a normal response and is an expression of hope that things will be OK." c. "You will need to help him accept the reality of the permanence of the divorce." d. "Your child is most likely blaming himself for your separation and divorce."

ANS: C Accepting the permanence of the divorce is one of the psychological tasks children of divorce must master in order to master normal developmental tasks. Parents need to focus on helping the child achieve these tasks. A remarriage does not necessarily help the child to achieve this task. It is a sign that the child cannot accept the reality of the situation, not of hope, and does not indicate self-blame.

5. Which region globally has the highest infant mortality rate? a. Indonesia b. Southern Asia c. Sub-Saharan Africa d. Syria

ANS: C Although Sub-Saharan Africa and Southern Asia together account for 81% of the infant mortality rate globally, Sub-Saharan Africa has the highest infant mortality rate in the world.

10. A 3-year-old child has had one episode of acute otitis media 3 weeks prior with a normal tympanogram just after treatment with amoxicillin. In the clinic today, the child has a type B tympanogram, a temperature of 102.5°F, and a bulging tympanic membrane. What will the primary care pediatric nurse practitioner order? a. A referral for tympanocentesis b. Amoxicillin twice daily c. Amoxicillin-clavulanate twice daily d. Intramuscular ceftriaxone

ANS: C Amoxicillin-clavulanate should be given for failed therapy with amoxicillin or when the child has had AOM treated with amoxicillin within the past month.

1. 16. A 10-year-old child has a 1-week history of fever of 104°C that is unresponsive to antipyretics. The primary care pediatric nurse practitioner examines the child and notes bilateral conjunctival injection and a polymorphous exanthema, with no other symptoms. Lab tests show elevated ESR, CRP, and platelets. Cultures are all negative. What will the nurse practitioner do? a. a. Begin treatment with intravenous methyl prednisone. b. b. Consider IVIG therapy if symptoms persist one more week. c. c. Order a baseline echocardiogram today and another in 2 weeks. d. d. Reassure the child's parents that this is a self-limiting disorder.

ANS: C An echocardiogram should be obtained as soon as the diagnosis of Kawasaki disease (KD) is established, as a baseline study, with subsequent studies in 2 weeks and in 6 to 8 weeks. This child has fever and only two other symptoms, which may be consistent with atypical KD. Atypical KD is more common in very young children and in children over 9 years of age, and coronary artery involvement is found more frequently in children with atypical KD. Methyl prednisone is given for children with IVIG-resistant disease. IVIG should be begun ideally in the first 10 days of the illness. Although KD is a self-limiting disorder, the risk of coronary artery involvement is high, so this must be evaluated and treated.

4. The primary care pediatric nurse practitioner provides well child care for a community of immigrant children from Central America. The pediatric nurse practitioner is surprised to learn that some of the families are Jewish and not Catholic. This response is an example of cultural : a. collectivism. b. constructivism. c. essentialism. d. individualism.

ANS: C An essentialist view of culture, which dominates the health care literature, portrays an ethnic minority group as having a static set of traits and oversimplifies cultural information, applying traits to all members of the group. Assuming that all people from Central America are Catholic is an example of this oversimplification. Collectivism refers to a member of an ethnic group who perceives himself or herself to be intrinsically part of that group. A constructive view recognizes culture as complex and dynamic and sees people as individuals who may belong to multiple cultures simultaneously. Individualism recognizes the individual, and not the group, as the basic unit of survival.

1. 7. A child is bitten on one arm by a neighbor's dog. The dog is immunized against rabies and the child's last tetanus immunization was 4 years prior. The wound edges are gaping and avulsed. What is an important initial intervention when treating this injury? a. a. Administration of rabies prophylaxis and a tetanus booster b. b. Debriding and suturing the wound to prevent infection c. c. Irrigation of the wounds with high-pressure normal saline d. d. Reporting the animal bite to the local animal control authority

ANS: C Animal and human bites need to be irrigated with normal saline using >5 psi of pressure. The animal has been vaccinated for rabies and the child's tetanus is current, so prophylaxis for both of these is not indicated. There is controversy about whether primary closure is appropriate. Reporting the animal is not a primary action.

11. What will the primary care pediatric nurse practitioner teach the parent of an infant about cleaning the child's teeth? a. To allow the child to control the amount of toothpaste used b. To choose a toothpaste with a mint flavor c. To use a smear of toothpaste and not to rinse the mouth d. To use a toothpaste containing whitening agents

ANS: C As long as very small amounts of toothpaste are used, the child should not rinse the mouth. The adult should control the amount of toothpaste used; too much fluoride can be toxic. Toothpaste should have a sweet flavor. Strongly flavored toothpastes and those containing whitening agents or bleach are contraindicated.

7. The parent of a 10-year-old boy tells the primary care pediatric nurse practitioner that the child doesn't appear to have any interest in girls and spends most of his time with a couple of other boys. The parent is worried about the child's sexual identity. The nurse practitioner will tell the parent a. children at this age who prefer interactions with same-gender peers usually have a homosexual orientation. b. children experiment with sexuality at this age as a means of deciding later sexual orientation. c. this attachment to other same-gender children is how the child learns to interact with others. d. to encourage mixed-gender interactions in order to promote development of sexual values.

ANS: C At age 10, children usually develop an intense same-gender relationship with a peer. This is how the child learns to expand the self, shares feelings, and learns how others manage problems. It does not indicate later sexual orientation and is not a characteristic of experimentation with sexuality. It is not necessary to encourage mixed-gender interactions.

1. 2. The primary care pediatric nurse practitioner is providing anticipatory guidance to the mother of a breastfed 6-month-old infant who asks about "baby-led weaning." What will the nurse practitioner tell her about this practice? a. a. "Foods given for this purpose do not meet all the child's nutritional needs." b. b. "Giving infants control of the feeding process will help prevent obesity." c. c. "Infants are given soft, mashable table foods when able to self-feed." d. d. "Infants must be able to grasp and feed themselves from a spoon to do this."

ANS: C Baby-led weaning is a concept where infants feed themselves, soft, mashable table foods that they grasp rather than being spoon-fed, allowing the infant more control. Foods given either by spoon or finger-fed will meet the child's nutritional needs. More research is needed to determine if this method affords protection from obesity by improving self-regulation. Infants must be able to sit without support and to reach and grasp for objects.

6. An infant with trisomy 21 has a complete AV canal defect. Which finding, associated with having both of these conditions, will the primary care pediatric nurse practitioner expect? a. Crackles in both lungs b. Hepatomegaly c. Oxygen desaturation d. Peripheral edema

ANS: C Because infants with trisomy 21 maintain neonatal high pulmonary vascular resistance, they often do not show signs of CHF but instead will have signs of pulmonary hypertension with loud single S2 and desaturation with agitation or effort. Crackles, hepatomegaly, and edema are signs of CHF.

6. An adolescent has recently begun doing poorly in school and has stopped participating in sports and other extracurricular activities. During the history interview, the adolescent reports feeling tired, having difficulty concentrating, and experiencing a loss of appetite for the past few weeks but cannot attribute these changes to any major life event. Which is an important next step in managing this patient? a. Administering a diagnostic rating scale for depression b. Considering a short-term trial of an antidepressant medication c. Determining suicidal ideation and risk of suicide d. Referring the adolescent to a mental health specialist

ANS: C Because this adolescent exhibits clear signs of depression, the first goals of management are to determine suicidal risk and to intervene to prevent suicide since the risk of suicide is greatest during the first 4 weeks of a depressive episode. A diagnostic rating scale may help in diagnosing the depression, but assessing suicide risk is a priority. Antidepressant medications may be useful but are best initiated by a mental health specialist. The initial response in adolescents should be to determine suicide risk to decide whether to admit to inpatient therapy or refer to a mental health specialist.

19. The primary care pediatric nurse practitioner is offering anticipatory guidance to the parents of a 6- year-old child who has Down syndrome. What will the nurse practitioner tell the parents about physical activity and sports in school? a. Children with Down syndrome get frustrated easily when engaging in sports. b. Children with Down syndrome should not participate in strenuous aerobic activity. c. Their child should have a cervical spine evaluation before participation in sports. d. Their child should only participate in sports sanctioned by the Special Olympics.

ANS: C Because up to 40% of children with Down syndrome have a hypermobility or instability between C1-C2 and up to 61% have occipito-atlantal hypermobility, they should undergo radiological evaluation of the cervical spine to be cleared for strenuous sports. Many children and adolescents with intellectual and developmental disabilities (including those with Down, fragile X, Turner, or Klinefelter syndromes or autism) are capable of performing exercise or strenuous activities. Special needs children should be encouraged to participate in sports to increase physical abilities and increase self-confidence. Children with Down syndrome may benefit from strenuous aerobic activity and may participate in any sports once cervical spine stability is evaluated, not just those sanctioned by the Special Olympics.

11. The primary care pediatric nurse practitioner is performing a well child examination on a 4-year-old child. The parent reports that the child snores frequently, often awakens during the night, and seems cranky during the day. What will the nurse practitioner tell this parent? a. Most sleep disorders are benign and will be outgrown. b. Sleep disorders are symptomatic of underlying behavior problems. c. Sleep disorders at this age can have long-term impacts on learning. d. The child will need longer daytime naps to compensate for lost sleep.

ANS: C Behavioral sleep disorders and sleep-disordered breathing before age 5 can result in increased special education needs in children by 8 years of age. Sleep disorders are not usually benign or outgrown. Sleep disorders can result in behavioral difficulties but are not symptomatic of behavioral problems. Sleep- disordered breathing disrupts all sleep

20. When prescribing topical glucocorticoids to treat inflammatory skin conditions, the primary care pediatric nurse practitioner will a. initiate therapy with a high-potency glucocorticoid. b. order lotions when higher potency is necessary. c. prescribe brand-name preparations for consistent effects. d. use fluorinated steroids to minimize adverse effects.

ANS: C Brand-name preparations often have a more consistent base and potency. PNPs should be familiar with a few high-, medium-, and low-potency products and use those consistently. Therapy should be initiated with the lowest possible potency. Lotions have a lower potency than ointments and creams. Fluorinated steroids have the highest potency and a higher risk of side effects

6. During a well child examination of an infant, the primary care pediatric nurse practitioner notes 10 café au lait spots on the infant's trunk. What is the potential concern associated with this finding? a. Endocrine disorders b. Malignancy c. Neurofibromatosis d. Sturge-Weber syndrome

ANS: C Café au lait spots are significant for neurofibromatosis and should be referred if more than 5 lesions are present. Atypical nevi are concerning for malignancy. Port-wine stains are concerning for Sturge-Weber syndrome. Acanthosis is a sign of diabetes.

1. 6. The mother of a nursing infant expresses concern about whether high-cholesterol foods will increase her infant's risk of hyperlipidemia. What will the primary care pediatric nurse practitioner tell her? a. a. Breastfed infants have lower serum cholesterol levels than those who are not breastfed. b. b. Maternal cholesterol levels affect the cardiovascular risk of breastfed babies. c. c. Maternal dietary cholesterol intake does not affect the infant's serum cholesterol values. d. d. She should limit her dietary cholesterol to prevent hyperlipidemia in her infant.

ANS: C Changes in the maternal diet do not produce changes in cholesterol values in infants. Breastfed infants tend to have higher cholesterol levels than other infants, but cholesterol is necessary for brain and retinal development.

11. The parent of a toddler and a 4-week-old infant tells the primary care pediatric nurse practitioner that the toddler has just been diagnosed with pertussis. What will the nurse practitioner do to prevent disease transmission to the infant? a. Administer the initial diphtheria, pertussis, and tetanus vaccine. b. Instruct the parent to limit contact between the toddler and the infant. c. Order azithromycin 10 mg/kg/day in a single dose daily for 5 days. d. Prescribe erythromycin 10 mg/kg/dose four times daily for 14 days.

ANS: C Chemoprophylaxis for pertussis exposure is recommended for all household and close contacts of infected persons regardless of immunization status. Azithromycin is the drug of choice for infants from 1 month to 6 months of age. Administering the vaccine is not indicated since there isn't sufficient time to develop immunity. Infants under 1 month of age should not receive erythromycin because of the increased risk for pyloric stenosis associated with this drug

8. A 14-year-old child has a headache, unilateral weakness, and blurred vision preceded by fever and nausea. The child's parent reports a similar episode several months prior. The primary care pediatric nurse practitioner will consult with a pediatric neurologist to order a. a lumbar puncture. b. an electroencephalogram (EEG). c. neuroimaging with magnetic resonance imaging (MRI). d. positron emission tomography (PET) scan.

ANS: C Children who have MS exhibit the symptoms described above and are usually diagnosed with a gadolinium enhanced MRI. Lumbar puncture may be performed later to identify oligoclonal bands. An EEG is used to diagnose seizure activity. PET scans are used to detect tumors.

4. A school-age child enjoys playing basketball but doesn't make the intramural team. Which response by the child is characteristic of the concept of a growth mindset? a. "I didn't play well on the day of the tryouts." b. "I'll just have to find another sport I'm good at." c. "I'll need to work more on my outside shot." d. "I'm probably too short to be really good at this sport."

ANS: C Children who have a growth mindset have been taught to believe that hard work is key to success and that effort and practice contribute to growth. By not being discouraged and identifying something to work on, the child is exhibiting a growth mindset. Saying that he didn't play well is making excuses, while stating he needs to find something else he's good at or blaming his failure on a physical characteristic indicates a belief that success is dependent on fixed traits.

4. A child who was treated with amoxicillin and then amoxicillin-clavulanate for acute otitis media is seen for follow-up. The primary care pediatric nurse practitioner notes dull-gray tympanic membranes with a visible air-fluid level. The child is afebrile and without pain. What is the next course of action? a. Administering ceftriaxone IM b. Giving clindamycin orally c. Monitoring ear fluid levels for 3 months d. Watchful waiting for 48 to 72 hours

ANS: C Children with AOM may have effusion up to 3 months after the acute infection. The child should be monitored to ensure that this resolves. Antibiotics are not indicated. There is no acute infection, so watchful waiting for worsening of symptoms is not indicated.

1. 19. An adolescent who has exercise-induced asthma (EIA) is on the high school track team and has recently begun to practice daily during the school week. The adolescent uses 2 puffs of albuterol via a metered-dose inhaler 20 minutes before exercise but reports decreased effectiveness since beginning daily practice. What will the primary care pediatric nurse practitioner do? a. a. Counsel the adolescent to decrease the number of practices each week. b. b. Increase the albuterol to 4 puffs 20 minutes prior to exercise. c. c. Order a daily inhaled corticosteroid medication. d. d. Prescribe cromolyn sodium in addition to the albuterol.

ANS: C Children with EIA should use 2 puffs of a B2-agonist and/or cromolyn MDI 15 to 30 minutes prior to exercise, but, since tolerance may develop if a B2-agonist is used more than a few times a week, it should not be used as a controller monotherapy. Those who exercise regularly should use an ICS as a controller medication. Patients with asthma should be encouraged to exercise to improve overall health. Increasing the albuterol dose will not overcome the tolerance. And ICS is a preferred controller medication.

16. The parent of a 12-year-old child who has sickle cell trait (SCT) asks the primary care pediatric nurse practitioner whether the child may play football. What will the nurse practitioner tell this parent? a. Children with SCT should not play any contact sports. b. Children with SCT may not play for NCAA schools in college. c. Children with SCT should follow heat acclimatization guidelines. d. Children with SCT should not participate in organized sports.

ANS: C Children with SCT may play in sports as long as preventative measures, including heat acclimatization, are taken to prevent sickling crises. They may play contact sports and may play for NCAA teams as long as their sickle cell trait status is known.

12. A 5-year-old child is hit in the face with a baseball bat and is brought to the clinic by a parent. An exam reveals three avulsed front teeth. Radiologic studies are negative for facial fractures. What is the recommended treatment? a. Prescribe tetracycline 4.4 mg/kg twice daily for 7 to 10 days. b. Refer the child to a dentist for reimplantation of the avulsed teeth. c. Refer the child to a dentist immediately for further examination. d. Remove the teeth, place them in saline, and refer the child to a dentist.

ANS: C Children with avulsed or fractured teeth should be referred immediately to a dentist for treatment to avoid tooth abscesses and dental pain. Tetracycline is not recommended in children under age 12. Reimplantation of primary teeth is not recommended. If teeth will be reimplanted, they should be placed in cold milk or physiologic saline.

14. The parent of a child newly diagnosed with epilepsy asks the primary care pediatric nurse practitioner if the child will ever be able to participate in gym or sports. What will the nurse practitioner recommend? a. Bicycle riding is not safe for children with seizures. b. Contact sports should be avoided. c. Direct supervision of some activities is necessary. d. Underwater sports are not recommended.

ANS: C Children with epilepsy may participate in most sports but may require direct supervision in some cases to reduce the risk of injury to self or others if a seizure should occur during sports. Bicycle riding, contact sports, and underwater sports may be engaged in, but certain precautions must be taken (e.g., supervision).

15. The primary care pediatric nurse practitioner is performing a sports physical on an adolescent whose history reveals mild aortic stenosis. What will the nurse practitioner recommend? a. Avoidance of all sports to prevent sudden death b. Clearance for any sports since this is mild c. Evaluation by a cardiologist prior to participation d. Low-intensity sports, such as golf or bowling

ANS: C Children with mild AS may participate in any sport but must have annual cardiac evaluations. Children with severe AS should avoid sports to prevent sudden death. The PNP should not clear the child for sports without a cardiology evaluation. Low-intensity sports are recommended for children with moderate AS.

7. The primary care pediatric nurse practitioner works with families from a variety of cultures and socioeconomic classes. Which is an example of cultural humility in practice? a. Giving health care advice that takes cultural differences into account b. Identification of other cultures that may be superior to one's own culture c. Receptivity to learning about the perspectives of other cultures d. Respecting other cultures while maintaining the views of one's own

ANS: C Cultural humility is defined as the lifelong commitment to developing mutually beneficial, nonpaternalistic partnerships and is based on a model of passive volition, receptivity, and being open to learning from others. Practitioners who have cultural humility are always seeking to learn about other cultures. Cultural humility involves asking questions, rather than giving answers. Cultural humility does not mean identifying one's own culture as inferior. Practitioners who are culturally competent are open to the influence of other cultures.

25. A child is brought to the clinic with a generalized, annular rash characterized by raised wheals with pale centers. On physical examination, the child's lungs are clear and there is no peripheral edema. A history reveals ingestion of strawberries earlier in the day. What is the initial treatment? a. Aqueous epinephrine 1:1000 subcutaneously b. Cetirizine once in clinic and then once daily for 2 weeks c. Diphenhydramine 0.5 to 1 mg/kg/dose every 4 to 6 hours d. Prednisone 1 to 2 mg/kg/day for 1 week with rapid taper

ANS: C Diphenhydramine is given initially as long as anaphylaxis and angioedema are not present. Aqueous epinephrine is used for anaphylaxis and angioedema. Cetirizine is less effective than diphenhydramine. Prednisone is used for refractory episodes.

8. The parent of a 14-year-old child tells the primary care pediatric nurse practitioner that the adolescent has expressed a desire to be a vegetarian, is refusing all meat served at home, and wants the family to eat vegetarian meals. What will the nurse practitioner tell the parent? a. Do not allow a vegetarian diet in order to maintain appropriate limits for the adolescent. b. Provide vegetarian options for the adolescent that preserve adequate nutrition and protein intake. c. Suggest that the adolescent prepare appropriate vegetarian dishes to complement family meals. d. Tell the adolescent that a vegetarian diet may be considered in adulthood but not while living at home.

ANS: C Early adolescents begin to develop their own value system and may try value systems other than the one that they have learned from their family, which is a normal part of establishing personal identity. The parent may allow expression of other values, such as a vegetarian diet, as long as nutritional needs are met and the adolescent takes responsibility for preparing the food.

7. The primary care pediatric nurse practitioner sees a 3-year-old child who chronically withholds stools, in spite of the parents' attempts to stop the behavior, requiring frequent treatments with laxative medications. Which diagnosis will the nurse practitioner use to facilitate third-party reimbursement? a. Altered elimination pattern b. Elimination disorder c. Encopresis d. Parenting alteration

ANS: C Encopresis is a medical diagnosis, classified in the ICD-10-CM, and is recognized for reimbursement purposes. "Altered elimination pattern" and "Parenting alteration" are NANDA International diagnoses and are not recognized for reimbursement. "Elimination disorder" is a developmental diagnosis.

2. The primary care pediatric nurse practitioner is performing an initial well child exam on a 3-year-old child recently adopted from Africa. The adoptive parent has a record of immunizations indicating that the child is fully vaccinated. What will the nurse practitioner do? a. Administer a booster dose of each vaccine to ensure immunity. b. Find out whether the vaccines were provided by reliable suppliers. c. Perform antibody titers and reimmunize the child. d. Record the vaccines in the child's electronic medical record.

ANS: C Even though suppliers of vaccines worldwide produce vaccines that are of adequate quality, vaccine handling can be suspect. The PNP should perform titers and reimmunize if in doubt. If the child has not been adequately vaccinated, the PNP will need to administer each series based on catch-up dosing for age. If the child has been adequately immunized, boosters are not indicated. Performing titers is the best way to assess full immunity, since suppliers can be suspect.

12. The parent of a 5-year-old child who has just begun kindergarten expresses concern that the child will have difficulty adjusting to the birth of a sibling. What will the primary care pediatric nurse practitioner recommend? a. Allowing the child opportunities to discuss feelings about the baby b. Giving the child specific baby care tasks to promote sibling bonding c. Having snack time with the child each day to discuss the school day d. Providing reassurance that the sibling will not replace the child

ANS: C Family routines provide support to children and help them self-regulate, especially during times of change, and serve as a buffer during times of change and transition. This child has two major changes, so setting aside regular time to spend with the child will help stabilize these changes. The other options may be useful as well, but routines and special activities are most important.

2. The parent of a 14-year-old child tells the primary care pediatric nurse practitioner that the child skips classes frequently in spite of various disciplinary measures, such as grounding and extra homework and is earning Cs and Ds in most classes. What will the nurse practitioner recommend? a. Counseling for emotional problems b. Development of an Individual Education Plan c. Evaluation for possible learning disorders d. Referral for a behavioral disorder

ANS: C Frequent school absenteeism, class skipping, and other types of school avoidance may indicate a problem with cognitive ability and should be assessed. When cognitive disorders are ruled out, other issues, such as behavioral and emotional problems may be considered. IEPs are used for children who have identified special physiological or cognitive needs and may be useful if a cognitive disorder is identified.

8. The primary care pediatric nurse practitioner diagnoses a high school basketball player with mononucleosis. The adolescent asks when she may resume play. What will the nurse practitioner tell her? a. After 3 weeks, she may begin lifting weights but not full sports. b. After 4 weeks, she may return to full play and practice. c. At 4 weeks, she must have an exam to determine fitness for play. d. She may engage in moderate exertion and practice after 3 weeks.

ANS: C Full return to play should be determined on a case-by-case basis and is generally considered safe at 4 weeks after symptom onset, assuming physical stamina has returned, all symptoms have resolved, and the sport does not increase intraabdominal pressure during play. Athletes should avoid any form of exertion, including all sports during the first 3 weeks at a minimum and should avoid anything with a risk of chest or abdominal contact or anything that involves increased intra-abdominal pressure. Splenic rupture can occur spontaneously (rare), but the risk of rupture increases when participating in a contact or collision sport or a sport in which there is an increase in intraabdominal pressure. The nurse practitioner should recommend an exam at 4 weeks to determine fitness for play.

2. The parent of a newborn infant asks the primary care pediatric nurse practitioner when to intervene to help the infant's future intellectual growth. What will the nurse practitioner tell the parent? a. Cognitive learning begins during the toddler years. b. Intellectual growth begin when speech develops. c. Language and literacy skills begin at birth. d. Preschool is an optimal time to begin general learning.

ANS: C General learning and acquisition of skills for later reading and writing begin at birth, not in kindergarten or first grade, and these skills grow with everyday loving interactions between infants and caregivers. Cognitive learning changes during toddler years but begins at birth. Intellectual growth is not tied to speech alone.

1. 4. The parent of a school-age child reports that the child is on a gluten-free diet. When questioned about the reason for this diet, the parent states that the child has fewer stomach aches since beginning the diet but has never been diagnosed with celiac disease. The parent reports using gluten-free grain products for all family members. The nurse practitioner will tell this parent that gluten-free diets a. a. are generally low in sugar and fat. b. b. are healthy and help prevent obesity. c. c. may be deficient in essential nutrients. d. d. provide adequate protein to meet daily needs.

ANS: C Gluten-free grain products are often highly processed and not enriched with iron or folate. Many are very low in protein, enough so that they are used for patients with metabolic conditions such as PKU who need severe protein restrictions. They often have sugar and fat added to them to improve taste and do not help prevent obesity.

28. A school-age child is brought to clinic after a pediculosis capitis infestation is reported at the child's school. If this child is positive, what will the primary care pediatric nurse practitioner expect to find on physical examination, along with live lice near the scalp? a. Excoriated macules along the child's collar and underwear lines b. Inflammation and pustules on the face and neck c. Itching of the scalp, with skin excoriation on the back of the head d. Linear or S-shaped lesions in webs of fingers and sides of hands

ANS: C Head lice commonly cause itching of the scalp, especially on the back of the head and neck, along with skin excoriation, and may be the only initial sign. Excoriated lesions along underwear lines are typical of body lice. Inflammation and pustules occur with acne. Linear or S-shaped lesions occur with scabies infestations.

2. An adolescent has acne with lesions on the cheeks and under the chin. Which distribution is this? a. Athletic b. Frictional c. Hormonal d. Pomadal

ANS: C Hormonal acne has a beard distribution. Athletic acne occurs on the forehead, chin, and shoulders, caused by helmets and pads. Frictional occurs where bras, tight clothes, and headbands rub. A pomadal distribution occurs along the temple and forehead, as a result of pomades or oil-based cosmetics.

9. An adolescent has a TB skin test prior to working as a volunteer in a hospital. The adolescent is healthy and has not travelled to or from a TB-endemic area or had close contact with anyone who has TB. The Mantoux skin test shows 10 mm of induration after 48 hours. What will the primary care pediatric nurse practitioner do? a. Ask the adolescent about exposure to homeless persons. b. Order a chest radiograph to rule out active TB. c. Reassure the adolescent that this is a negative screen. d. Refer the adolescent to an infectious disease specialist.

ANS: C In children 4 years and older without risk factors, induration must be at least 15 mm or greater to be considered to be a positive screen. It is not necessary to question the adolescent about possible exposures. Chest radiographs are ordered to evaluate for active TB in persons with a positive screen. Referral to an infectious disease specialist is done if active TB is present.

8. The primary care pediatric nurse practitioner suspects that the parent of a child who is doing poorly in school is being abused by a partner. What is a priority response by the nurse practitioner? a. Notifying the child's school counselor about this problem b. Referring the child and family to a social worker c. Reporting this according to any mandated reporting laws d. Suggesting that the parent avoid the abusive situation

ANS: C In most states, health care providers are mandated to report a child's exposure to Intimate partner violence since it is considered a form of emotional child abuse. The PNP should follow any state laws that mandate this as a priority. Once child protective services is involved, the PNP may assist with notification of school personnel, referrals to social workers, and suggestions to parents.

12. The primary care pediatric nurse practitioner is counseling a school-age child about asthma management strategies. The child states that it is "too much trouble" to remember to use an inhaled corticosteroid medication twice daily and reports feeling fine, in spite of exhibiting expiratory wheezes. Which action uses the health belief and self-efficacy model to teach this child about asthma management? a. Asking the child to try to use the inhaler at least once daily b. Discussing whether the child wants to participate in athletics c. Obtaining pre- and post-treatment spirometry testing d. Providing written information about inhaled corticosteroids

ANS: C In the health belief model, clients need to believe that taking some action will reduce the risk of symptoms and that the benefits of the action will outweigh the costs or effort. Demonstrating pre- and post-treatment spirometry measures can help the child see that symptoms are reversible. Asking the child to make one change may be part of the transtheoretical model, in which small behavior changes may precede commitment to the actual plan. Discussing athletics may be useful when using the health promotion model to encourage the client to participate in behaviors that will promote healthy activities, such as sports. Providing written information is not part of any of the health behavior models.

3. When providing well child care for an infant in the first year of life, the primary care pediatric nurse practitioner is adhering to the most recent American Academy of Pediatrics Recommendations for Preventive Pediatric Health Care guidelines by : a. focusing less on development and more on illness prevention and nutrition. b. following guidelines established by the Bright Futures publication. c. scheduling well-baby visits to coincide with key developmental milestones. d. seeing the infant at ages 2, 4, 6, and 12 months when immunizations are due.

ANS: C In the most recent AAP Recommendations for Preventive Pediatric Health Care, there is a greater emphasis on behavioral and developmental issues and a recommendation that well child care be based on child and family development rather than the periodicity of immunization schedules. This will require a revision of the current recommendations in Bright Futures.

7. The primary care pediatric nurse practitioner is performing a well baby examination on a 2-month-old infant who has gained 25 grams per day in the last interval. The mother is nursing and tells the nurse practitioner that her infant seems fussy and wants to nurse more often. What will the nurse practitioner tell her? a. She may not be making as much breastmilk as before. b. She should keep a log of the frequency and duration of each feeding. c. The infant may be going through an expected growth spurt. d. The infant should stay on the previously established nursing schedule.

ANS: C Infants may have a growth spurt at 6 to 8 weeks, and mothers who are breastfeeding may be concerned that they are not making enough milk when they notice that the infant is fussy and wanting to nurse more often. The PNP should reassure the mother that this is expected. It is not necessary, since the infant is gaining weight appropriately, for the mother to keep a log. The mother should follow the infant's cues for feeding since the extra suckling will increase the milk supply to meet the growing infant's needs.

1. 3. The primary care pediatric nurse practitioner is performing an assessment on a 1- week-old newborn with a slightly elevated bilirubin who is breastfeeding well and who has gained 30 grams in the past 24 hours. The infant is stooling and voiding well. The nurse practitioner suspects breast milk jaundice. Which action is correct? a. a. Order home phototherapy and closely monitor bilirubin levels. b. b. Reassure the mother that the bilirubin level will drop in a few days. c. c. Recheck the serum bilirubin and infant's weight in 24 hours. d. d. Recommend that the mother pump her breast milk for a couple of days.

ANS: C Infants with breast milk jaundice who are gaining weight and thriving should continue to breastfeed and be monitored for the development of pathologic jaundice. It is not necessary to order phototherapy or discontinue breastfeeding unless pathologic jaundice is present. The bilirubin may remain elevated up to 3 months.

12. An 18-month-old child has a 1-day history of intermittent, cramping abdominal pain with non-bilious vomiting. The child is observed to scream and draw up his legs during pain episodes and becomes lethargic in between. The primary care pediatric nurse practitioner notes a small amount of bloody, mucous stool in the diaper. What is the most likely diagnosis? a. Appendicitis b. Gastroenteritis c. Intussusception d. Testicular torsion

ANS: C Intussusception is characterized by intermittent pain associated with drawing up the legs, "currant jelly" stools, and lethargy in between episodes. Appendicitis is characterized by pain localizing to the RLQ and is not intermittent. Gastroenteritis is likely when vomiting precedes symptoms of pain or discomfort. Testicular torsion involves the testicles and thus has different physical findings and would not be accompanied with bloody stools.

14. A 12-month-old infant who had cardiopulmonary bypass with RBC and plasma infusions during surgery at 8 months is seen for a well child examination. Which vaccine may be administered at this visit? a. MMR b. OPV c. PCV-13 d. Varivax

ANS: C Live vaccines should be delayed until 6 months after cardiopulmonary bypass and exposure to RBCs and plasma. The PCV-13 is not a live-virus vaccine and the others are.

32. When reviewing a white blood cell (WBC) count, the primary care pediatric nurse practitioner suspects a viral infection when which WBC element is elevated? a. Bands b. Leukocytes c. Lymphocytes d. Neutrophils

ANS: C Lymphocytes are usually elevated during viral infections. Bands and neutrophils are generally elevated with bacterial infections. Leukocytes comprise all WBCs and are usually, although not always, elevated during bacterial infections.

1. During a well child exam, the primary care pediatric nurse practitioner learns that the parents of a young child fight frequently about finances. The parents state that they do not fight in front of the child and feel that the situation is temporary and related to the father's job layoff. What will the nurse practitioner do? a. Reassure them that the child is too young to understand. b. Recommend that they continue to not argue in front of the child. c. Suggest counseling to learn ways to handle stress. d. Tell them that the conflict will resolve when the situation changes.

ANS: C Marital problems can result in child behavior difficulties and anxieties, and conflict can be picked up by the child. The parents should try to learn to modify unhealthy behaviors, such as increased conflict during stressful situations. Even when children do not understand, they pick up on cues from the parents about anxiety and stress and can internalize these feelings. Avoiding arguments in front of the child does not alleviate the underlying conflict and stress. The behavior of fighting during this stressful situation may indicate a pattern of response to stress and will only recur with each subsequent stressful period.

4. The parent of a 4-year-old who has difficulty initiating and maintaining sleep has tried several nonpharmacological methods with variable success and asks about medications. What will the primary care pediatric nurse practitioner recommend? a. Diphenhydramine b. Lorazepam c. Melatonin d. Zolpidem

ANS: C Medications to treat dyssomnias are generally discouraged in children, since they have side effects and since the mainstay of treatment is behavioral therapy and sleep hygiene. If medications are used, melatonin is the most commonly prescribed. Diphenhydramine can lead to parasomnias in some children. Benzodiazepines, such as lorazepam, can cause dependence. Sedatives, such as zolpidem, have high levels of side effects.

24. A child has small, firm, flesh-colored papules in both axillae which are mildly pruritic. What is an acceptable initial approach to managing this condition? a. Application of trichloroacetic acid 25% to 50% using a dropper b. Applying liquid nitrogen for 2 to 3 seconds to each lesion c. Reassuring the parents that these are benign and may disappear spontaneously d. Referral to a dermatologist for manual removal of lesions with curettage

ANS: C Molluscum contagiosum is a benign viral skin infection; most lesions disappear within 6 months to 2 years. An initial "wait and see" approach is acceptable. If itching is severe, the risk is autoinoculation and spread of lesions, along with increased discomfort and then other treatment measures may be attempted, depending on the severity. Topical medications, such as trichloroacetic acid or liquid nitrogen may be used if the lesions become uncomfortable or persist and should be used with caution. More severe outbreaks may require removal with curettage.

6. A 4-year-old child with an upper respiratory tract infection has cloudy nasal discharge and moderate nasal congestion interfering with sleep. The parent asks what product to use to help with symptoms. What will the primary care pediatric nurse practitioner recommend? a. Antihistamines b. Decongestant sprays c. Saline rinses d. Zinc supplements

ANS: C Normal saline nose drops, nasal rinses, or sprays are helpful for all ages of children to clear nasal passages. The use of decongestants, antihistamines, and cough medicine does not shorten the course of a disease. While their use may help with relieving nasal symptoms, their use is not recommended for children younger than 6 years old. Zinc is not recommended in children because of potential side effects and questionable efficacy.

8. The parent of a school-age child who is overweight tells the primary care pediatric nurse practitioner that the child seems to crave high-calorie, high-carbohydrate foods, even when full. The nurse practitioner learns that the child is often irritable and sleepy at school in spite of sleeping 9 or 10 hours each night. What will the nurse practitioner recommend? a. Assessment of leptin and ghrelin hormone levels b. Consultation with a dietician to develop an appropriate diet c. Referral to a sleep disorder clinic for a sleep study d. Taking one or two naps each day to increase the amount of sleep

ANS: C Obstructive sleep apnea has been suggested to be a contributing factor to the pathogenesis of obesity by inducing leptin resistance and increasing ghrelin levels, two hormones that regulate satiety. The child shows symptoms of these abnormalities by craving high-calorie comfort foods. The child should be evaluated for this underlying cause. Assessment of these hormone levels is not routinely done. Consultation with a dietician may be necessary at some point but does not get at the underlying problem. Increasing sleep time with naps has not been shown to counteract the obesity effect.

7. The parent of a 2-month-old infant is reluctant to have the baby vaccinated. What is an initial step in responding to these concerns? a. Inform the parent that all vaccines may be given without thimerosol. b. Providing Vaccine Information Statements for the parent to review. c. Question the parent's reasons for concern about immunizations. d. Remind the parent that the infant is exposed to thousands of germs each day.

ANS: C PNPs should question and listen carefully to parents' concerns about vaccines. Once concerns are identified and understood, the PNP can address the issues. The presence of thimerosol in vaccines is just one concern and should be addressed if that is identified. Providing a Vaccine Information Statement (VIS) should be done as part of the discussion to provide information to the parent. Unless the parent expresses concerns that vaccines will overwhelm the child's immune system, it is not necessary to bring up this possibility .

1. 10. The mother of a 3-year-old child takes the child to a play group once a week. She expresses concern that the child plays with toys but does not interact with the other toddlers. What will the primary care pediatric nurse practitioner counsel the mother? a. a. The child probably is very shy but will outgrow this tendency with repeated exposure to other children. b. b. The toddler may have a language delay that interferes with socialization with other children. c. c. Toddlers may be interested in other children but usually do not engage in interactive play. d. d. Toddlers need more structured play to encourage interaction and socialization with others.

ANS: C Parallel play is common among toddlers who, although they may be fascinated by other children, generally do not engage with peers in an interactive manner. This does not mean that the child is shy or has a language delay, although in preschool years, the development of symbolic language increases interactive play. Children need both structured and free play, but structured play will not increase interaction during this normally parallel period.

1. 5. The mother of a 6-year-old child tells the primary care pediatric nurse practitioner that the child only wants to eat French fries and hamburgers and refuses most vegetables. What will the nurse practitioner recommend? a. a. Giving the child a multivitamin since this is a phase b. b. Having the child eat vegetables before getting the hamburger c. c. Providing a variety of healthy foods at each meal d. d. Putting extra lettuce and tomatoes on hamburgers

ANS: C Parents are responsible for the foods their children eat, and it is their responsibility to provide healthful foods. Children should be exposed to a variety of healthy foods but not forced to eat any of them. However, parents should not bribe children with the reward of getting the unhealthy food that they want. With a well-balanced diet, not eating a vegetable prepared at one meal, for example, will not compromise the child's health, so a multivitamin is not necessary if the overall diet is healthy. Allowing hamburgers and adding desired foods again puts the child in control.

5. What will the primary care pediatric nurse practitioner recommend to the parent of an infant who is teething who asks about comfort measures? a. Administer oral ibuprofen or apply topical salicylates. b. Apply a topical anesthetic such a benzocaine to the gums. c. Give the infant a cold teething ring or wet washcloth to chew. d. Try Baby Orajel on the infant's gums several times daily.

ANS: C Parents may be counseled to give infants a cool teething ring or a cool, wet washcloth to chew on. Topical medications are not recommended and may traumatize the gums or cause toxicity.

19. An adolescent female has a history of repaired tetralogy of Fallot. Which long-term complication is a concern for this patient? a. Aortic stenosis b. Chronic cyanosis c. Mitral valve prolapse d. Ventricular failure

ANS: C Patients with repaired TOF, especially adolescent females, are at risk for mitral valve prolapse. Aortic aneurysm is a long-term risk for those with a history of left-sided lesions. Chronic cyanosis is a concern for lesions causing Eisenmenger syndrome or defects causing right ventricular outflow obstruction. Ventricular failure can occur with prolonged aortic or pulmonic valvar stenosis.

12. A 16-year-old sexually active female has a fever, bilateral lower abdominal pain, and malaise. A speculum and bimanual exam reveals adnexal tenderness. The urinalysis is normal and cervical cultures are pending. What medications will the primary care pediatric nurse practitioner prescribe for this patient? a. Azithromycin, doxycycline, and penicillin b. Cefotaxime, azithromycin, and penicillin c. Ceftriaxone, doxycycline, and metronidazole d. Doxycycline, penicillin, and metronidazole

ANS: C Patients with suspected PID may be given ceftriaxone 250 mg IM once, doxycycline 100 mg PO bid for 14 days, and metronidazole 500 mg PO bid for 14 days. The other options are not recommended by the CDC.

5. A child with Down syndrome who has sleep-disordered breathing with obstructive sleep apnea continues to have symptoms in spite of tonsillectomy and adenoidectomy and treatment with a leukotriene receptor antagonist medication and a nasal steroid spray. The primary care pediatric nurse practitioner will refer the child to a sleep disorder clinic to discuss which therapy? a. Craniofacial surgery b. Oral appliances c. Positive airway pressure therapy d. Supplemental oxygen

ANS: C Positive airway pressure therapy can be used to treat sleep-disordered breathing in children who have failed other therapies and even developmentally delayed children show improvement in behaviors after this therapy. Craniofacial surgery may be used in the presence of maxillofacial deformities that affect sleep-disordered breathing but is a last option. Oral appliances may be used if deformities can be corrected in this manner. Supplemental oxygen helps with oxygen saturations but not with disordered breathing patterns themselves.

1. The primary care pediatric nurse practitioner learns that an African-American family lives in a neighborhood with a high crime rate and suggests that they try moving to another neighborhood for the safety of their children. This is an example of a. cultural sensitivity. b. group bias. c. individual privilege. d. racial awareness.

ANS: C Privilege can be individual- or group-based and refers to the often unconscious lack of understanding of what other groups must deal with. The PNP is not aware that the family may lack the resources to move, may be fearful of moving to a "white" neighborhood, or may even feel safe around people that they know. Cultural sensitivity is an awareness of and respect for other cultures. Group bias is a prejudice, based on cultural, racial, or ethnic differences, toward a group of people. Racial awareness would describe an awareness of cultural differences based on race.

1. During a well child assessment of a preschool-age child, the parent voices concerns that, because the child has behavior problems at school, the child may have a mental health disorder. Which initial approach will provide the best information? a. Ask the parent whether other caregivers have voiced similar concerns. b. Interview the child separately from the parent to encourage sharing of feelings. c. Take time to actively listen to the parent's and child's perceptions of the problem. d. Use a validated screening tool to ensure that all aspects of behaviors are evaluated.

ANS: C Providers can get a clearer picture of the situation by taking the time to sit down and actively listen to both the parent's and child's concerns about the issue. It is important to remember that many parents have been trying to cope with the problem for some time, so asking for validation from others may be viewed as belittling. School-age children and adolescents should be interviewed separately to encourage sharing of information. Validated screening tools may be used later, but allowing parents to voice concerns without restricting the history is the initial focus.

3. During a well child exam on a 13-year-old female, the primary care pediatric nurse practitioner notes that the child is at Tanner Stage 3. During the exam, when the nurse practitioner initiates a conversation about healthy sexuality education, the parent states that this topic is "off limits." What will the nurse practitioner do? a. Ask the adolescent whether she wishes to discuss these matters since she is becoming an adult. b. Separate the parent from the adolescent to discuss the adolescent's concerns in private. c. Spend private time with the parent to discuss how sexuality education reduces the risk of early sexual intercourse and risky sexual behaviors. d. Tell the parent that this information is a routine part of adolescent well child examinations and must be included.

ANS: C Research has shown that sexuality education leads to a reduction in early onset of sexual intercourse and risky sexual behaviors. It is important for the PNP to be sensitive to the values of the family but also to advocate for the child. The child should be told, especially when she shows an interest in sexual relationships, that she may seek care independently of her parent and that it will remain confidential. When possible, the parent's wishes should be taken into account and both the adolescent and the parent should be encouraged to begin an open dialogue about these matters.

2. The primary care pediatric nurse practitioner understands that, to achieve the greatest world-wide reduction in child mortality from pneumonia and diarrhea, which intervention is most effective? a. Antibiotics b. Optimal nutrition c. Vaccinations d. Water purification

ANS: C Rotavirus is the most common cause of diarrhea globally and Strep pneumonia is the leading cause of pneumonia, and together these are the leading infectious causes of childhood morbidity and mortality globally. Both are vaccine-preventable diseases. Antibiotics to treat pneumonia, optimal nutrition, and clean water all help to reduce morbidity and mortality, but vaccination prevents the diseases from occurring.

27. A school-age child has fever of 104°F, sore throat, vomiting and malaise. The primary care pediatric nurse practitioner observes that the tonsils, oropharynx, and palate are erythematous and covered with exudate. the tongue is coated and red and there is a red, sandpaper-like rash on the child's neck, trunk, and extremities. A rapid strep test is positive. What will the nurse practitioner do to manage this child's illness? a. Administer intramuscular ceftriaxone. b. Hospitalize for further diagnostic tests. c. Prescribe oral amoxicillin. d. Refer to a pediatric infectious disease specialist.

ANS: C Scarletina is caused by erythrogenic toxin from Group A streptococcus. Treatment is the same as for Group A streptococcus unless complications occur. IM antibiotics are not indicated. The child does not need hospitalization or referral to a specialist.

and there is a red, sandpaper-like rash on the child's neck, trunk, and extremities. A rapid strep test is positive. What will the nurse practitioner do to manage this child's illness? a. Administer intramuscular ceftriaxone. b. Hospitalize for further diagnostic tests. c. Prescribe oral amoxicillin. d. Refer to a pediatric infectious disease specialist.

ANS: C Scarletina is caused by erythrogenic toxin from Group A streptococcus. Treatment is the same as for Group A streptococcus unless complications occur. IM antibiotics are not indicated. The child does not need hospitalization or referral to a specialist.

8. The parent of an 8-year-old child tells the primary care pediatric nurse practitioner that the child has begun to ask questions about why a schoolmate has "2 daddies" and wonders how to talk to the child about this. What will the nurse practitioner recommend? a. Beginning a discussion about different types of sexual relationships and same-sex partners b. Discussing the issue with the child in terms of the parent's religious values and norms c. Explaining that not all families are the same and what is most important is that they love and care for their children d. Telling the child that some adult relationships are complicated and will be understood when the child is older

ANS: C School age is a good time for parents to reinforce the notion that there is diversity in families within which parents and adults love and care for their children. It is not necessary to be explicit but to establish a good history of communication and to explain complex issues to children at a level of the child's understanding. In this way, the child will know that parents are accessible and open to discussion of complex and/or puzzling issues.

2. A 12-year-old child whose weight and BMI are in the 75th percentile has a diastolic blood pressure that is between the 95th and 99th percentiles for age, sex, and height on three separate occasions. Initial tests for this child will include a. complete blood count. b. erythrocyte sedimentation rate. c. renal function and plasma renin tests. d. urinalysis and electrolytes.

ANS: C Since the majority of children with stage 1 or 2 hypertension have renal or renovascular causes for elevated BP, renal function and plasma renin tests should be performed. Children under 10 years of age with stage 2 hypertension should have more aggressive laboratory evaluation, including CBC, ESR, UA, and electrolytes.

7. The parent of a 5-year-old child tells the primary care pediatric nurse practitioner that the child has been using the toilet to urinate for since age 3 but continues to defecate in "pull-ups." The nurse practitioner learns that the child has predictable bowel movements and a physical examination is normal. What will the nurse practitioner recommend? a. Providing a reward system to offer incentives when the child uses the toilet b. Put the child back in diapers and resume toilet training in a few months. c. Putting the child on the toilet for 5 to 10 minutes at the usual time of defecation d. Use of polyethylene glycol until the child is able to use the toilet regularly

ANS: C Since this child has predictable bowel patterns, the parent can put the child on the toilet for 5 to 10 minutes at these times to encourage toilet use. Rewards may be used at some point, but it is not recommended since the child is learning to do what is to be expected. Younger children may be put back in diapers and retrained in a few months. The child is not constipated and does not need medication.

1. 10. The primary care pediatric nurse practitioner is examining a school-age child who has had several hospitalizations for bronchitis and wheezing. The parent reports that the child has several coughing episodes associated with chest tightness each week and gets relief with an albuterol metered-dose inhaler. What will the nurse practitioner order? a. a. Allergy testing b. b. Chest radiography c. c. Spirometry testing d. d. Sweat chloride test

ANS: C Spirometry testing is the gold standard for diagnosing asthma and is then used on a regular basis to monitor, evaluate, and manage asthma. Allergy testing should be considered but is not diagnostic of asthma. Chest radiography should not be routine. A sweat chloride test is used based on history.

1. 4. An adolescent will begin taking the combination oral contraceptive pill (OCP). Which supplement will the primary care pediatric nurse practitioner caution the adolescent to avoid while taking OCPs? a. a. Ginkgo b. b. Kava c. c. St. John's wort d. d. Valerian

ANS: C St. John's wort may increase blood pressure and increase clearance of OCPs, causing breakthrough bleeding. Ginkgo increases the risk of bleeding and should not be given with salicylates, APAP, or NSAIDs. Kava and valerian can increase drug effects of sleepiness, especially with benzodiazepines and anesthetics.

10. A 6-year-old child has a systolic blood pressure between the 95th and 99th percentile for age, sex, and height and a diastolic blood pressure between the 90th and the 95th percentile on three separate clinic visits. This child's blood pressure is classified as a. normotensive. b. pre-hypertensive. c. stage 1 hypertensive. d. stage 2 hypertensive.

ANS: C Stage 1 hypertensive pressure ranges from the 95th percentile or from 120/80 mm Hg to 5 mm Hg above the 99th percentile for age, sex, and height for either systolic or diastolic pressure. Normotensive pressure is below the 90th percentile. Pre-hypertensive pressure is between the 90th and 95th percentiles. Stage 2 hypertensive pressure is greater than the 99th percentile.

8. The primary care pediatric nurse practitioner elicits positive Ortolani and Barlow signs in a 6-month- old infant not previously noted in the medical record. What is the correct treatment? a. Pavlik harness b. Spica cast c. Surgical intervention d. Triple diapering

ANS: C The 6- to 18-month-old infant with a dislocated hip is likely to require either closed manipulation or open reduction. The other interventions should be used in younger infants.

4. The parent of a 6-year-old child expresses concern that the child may have ADHD. Which screening tool will the primary care pediatric nurse practitioner use to evaluate this possibility? a. Behavioral and Emotional Screening System for Children (BESS-2) b. Behavioral Assessment for Children - 2nd ed. (BASC-2) c. Conner's 3 Parent and Teacher Rating Scale d. Pediatric Symptom Checklist (PSC)

ANS: C The Conner's Parent and Teacher Rating Scale is used to assess ADHD symptoms in children aged 6 to 18 years. The BESS-2 is used to evaluate social emotional and mental health in children. The BASC-2 is used to further assess children who have positive findings on the BESS-2. The PSC is used to assess cognitive, emotional, and behavioral problems in children.

6. The primary care pediatric nurse practitioner prescribes a new medication for a child who develops a previously unknown adverse reaction. To report this, the nurse practitioner will : a. access the BPCA website. b. call the PREA hotline. c. log onto the FDA Medwatch website. d. use the AAP online PediaLink program.

ANS: C The FDA Medwatch website is available for reporting of drug-related adverse effects, and all providers are encouraged to report these here. BPCA and PREA are legislative acts and do not have a hotline or website for adverse effects reporting. The AAP PediaLink program is a source for labeling changes of drugs.

1. The parent of a toddler is concerned that the child may have autism. The primary care pediatric nurse practitioner completes a Modified Checklist for Autism in Toddlers (M-CHAT) tool, which indicates several areas of concern. What will the nurse practitioner do? a. Administer a Childhood Autism Rating Scale (CARS) in the clinic. b. Consult a specialist to determine appropriate early intervention strategies. c. Refer the child to a behavioral specialist for further evaluation. d. Tell the parent that this result indicates that the child has autism.

ANS: C The M-CHAT is a screening tool and is useful for detecting behaviors that may indicate autism. This instrument has been found to have acceptable sensitivity, specificity, and significant positive predictive value. If these behaviors are detected, the PNP should refer the child to a specialist for further assessment, using more diagnostic tools. The CARS may be used but requires specialty training and proper credentials. Until the diagnosis is determined, strategies for intervention are not discussed. The M-CHAT is a screening tool and is not diagnostic.

3. A 16-year-old female reports dull, achy cramping pain in her lower abdomen lasting 2 or 3 hours that occurs between her menstrual periods each month. The adolescent is not sexually active. What is the treatment for this condition? a. Abdominal ultrasound to rule out ovarian cyst b. Oral contraceptives to suppress ovulation c. Prostaglandin inhibitor analgesics and a heating pad d. Referral to a pediatric gynecologist

ANS: C The adolescent is experiencing mittelschmerz pain, which is thought to occur when the follicle ruptures at the time of ovulation. Unless the pain is severe, the adolescent should be reassured and offered strategies to relieve discomfort, such as a heating pad and NSAIDs. The pain is intermittent and occurs between periods; if it were persistent and severe, abdominal US would be indicated. Oral contraceptives are rarely used to suppress ovulation when symptoms are severe. Referral to a pediatric gynecologist is not indicated.

2. The primary care pediatric nurse practitioner is evaluating a 5-year-old child who has frequent soiling of stool associated with stomach aches and decreased appetite for the past 2 months. The parent states that the child has two or fewer formed bowel movements each week and has been toilet trained for about 2 years. Which initial assessment will the nurse practitioner make? a. History of neurogenic conditions b. Recent adjustments in the family c. Recent illnesses, fluid intake, changes in diet d. Toilet training history

ANS: C The child has been toilet trained and has recently developed chronic constipation. The first step is to evaluate recent illnesses or dietary changes that could cause constipation and painful stools that resulted in stool withholding. The other options represent more underlying physiologic or psychological pathology and should be explored if simple physiologic causes are not present.

7. During a well child assessment of an 18-month-old child, the primary care pediatric nurse practitioner observes the child becoming irritable and uncooperative. The parent tells the child to stop fussing. What will the nurse practitioner do? a. Allow the parent to put the child in a "timeout." b. Ask the parent about usual discipline practices. c. Offer the child a book or a toy to look at. d. Stop the exam since the child has reached a "meltdown."

ANS: C The child has exhibited early signs of misbehavior. At this stage, distraction and active engagement may be used to stop more problems from occurring. It is not necessary to use a timeout because the child hasn't reached the point where cooperation is impossible. The PNP should model appropriate interventions by offering the child a distraction and may ask the parent about discipline practices later in the visit. The child is not at a "meltdown" state.

9. A young adolescent reports chest pain associated with coughing and lifting. Physical examination reveals tenderness over several ribs, radiating to the back. Auscultation of the heart, lungs, and abdomen are normal. There is no history of injury. What will the primary care pediatric nurse practitioner do? a. Obtain a chest radiograph to evaluate possible causes for these symptoms. b. Order an electrocardiogram to rule out potential cardiovascular disease. c. Recommend NSAIDs, stretching exercises, and ice packs to the area. d. Refer the child to a pediatric orthopedist for evaluation and treatment.

ANS: C The child has symptoms and a history consistent with costochondritis. Treatment is symptomatic with analgesics, ice packs, and exercise and rest. Chest radiographs offer no diagnostic value, except to rule out other causes. Unless heart sounds are abnormal, or there are other signs of cardiovascular disease, an ECG is not indicated. Referral is not necessary unless symptoms persist.

8. The mother of a 12-month-old uncircumcised male infant reports that the child seems to have pain associated with voiding. A physical examination reveals a tight, pinpoint opening of the foreskin, which thickened and inflamed. What will the primary care pediatric nurse practitioner do? a. Attempt to retract the foreskin to visualize the penis. b. Order corticosteroid cream 3 times daily for 4 weeks. c. Refer the child to a pediatric urologist. d. Teach the mother to gently stretch the foreskin with cleaning.

ANS: C The child has symptoms consistent with pathologic phimosis and should be referred for possible circumcision. The foreskin should never be forcefully retracted. Non-pathologic phimosis can usually be managed by normal cleansing and gentle stretching. Corticosteroid cream is used for persistent, non- pathologic phimosis.

2. The primary care pediatric nurse practitioner is examining a 6-year-old child who attends first grade. The child reports "hating" school. The parent states that the child pretends to be sick frequently in order to stay home from school. To further assess this situation, the nurse practitioner will first ask the child : a. about school performance and grades. b. why school is so distressing. c. to name one or two friends. d. whether bullying is taking place.

ANS: C The earliest school-age psychosocial milestone occurs when children learn to separate easily from family, allowing them to go to school. Mastery of these skills enables them to develop and maintain peer friendships. Social interaction skills are necessary in order to develop mastery over school activities. Asking the child to describe why school is distressing may not elicit information, since the child may not be able to articulate this. Bullying is not the only reason for disliking school, but, if it is, will emerge during a discussion about friends and schoolmates.

9. When meeting with a new family, the primary care pediatric nurse practitioner develops a database that identifies family members and others living in the household, relationships with others outside the household, and significant behavioral and emotional problems. Which tool will the nurse practitioner use to record this information? a. CRAFFT b. Ecomap c. Genogram d. Pedigree

ANS: C The genogram is an approach to developing a family database to provide a graphic representation of family structure, roles, and problems of recurring significance in a family. The CRAFFT tool is used to assess substance abuse in adolescents. The ecomap is used to identify relationships in the family and community that are supportive or harmful. The pedigree is used to identify potential genetic disorders.

25. A 9-month-old infant has had a fever of 103°F for 2 days and now has a diffuse, maculopapular rash that blanches on pressure. The infant's immunizations are up-to-date. What will the primary care pediatric nurse practitioner do? a. Administer immunoglobulin G to prevent fulminant illness. b. Perform serologic testing for human herpes virus -6 and human herpes virus -7. c. Reassure the parent that this is a mild, self-limiting disease. d. Recommend avoiding contact with pregnant women.

ANS: C The infant has symptoms consistent with roseola infantum, which is a benign, self-limiting disease. It is not necessary to administer IgG or perform serologic testing or to avoid contact with pregnant women.

8. A 5-year-old child has enlarged tonsils and a history of four throat infections in the previous year with fever, cervical lymphadenopathy, and positive Group A Streptococcus pyogenes (GABHS) cultures. The parent reports that the child snores at night and expresses concerns about the child's quality of sleep. The next step in managing this child's condition is to : a. continue to observe the child for two or more GABHS infections. b. prescribe prophylactic antibiotics to prevent recurrent infection. c. refer to a pulmonologist for polysomnography evaluation. d. refer to an otolaryngologist for possible tonsillectomy.

ANS: C The potential for sleep apnea should be evaluated since the parent reports snoring and concerns with sleep in a child with frequent throat infections. This child has not had a high enough number of GABHS throat infections to warrant tonsillectomy and should be watched. Prophylactic antibiotics are not indicated.

12. The primary care pediatric nurse practitioner obtains a tympanogram on a child that reveals a sharp peak of -180 mm H2O. What does this value indicate? a. A normal tympanic membrane b. Middle ear effusion c. Negative ear pressure d. Tympanic membrane perforation

ANS: C The type C tympanogram has a sharp peak between -100 and -200 mm H2O and reflects negative ear pressure. A normal tympanogram has a sharp positive peak or a type A tympanogram. Middle ear effusion and a TM perforation both cause a type B tympanogram with either no peak or a flattened wave.

1. 4. The mother of a newborn asks the primary care pediatric nurse practitioner about the benefits of breastfeeding. What will the nurse practitioner tell her? a. a. Breastfeeding for 9 months or longer will reduce the incidence of food allergies. b. b. Breast milk is an excellent source of vitamin D, iron, and other essential nutrients for the baby. c. c. Nursing her baby exclusively for at least 4 months will help her infant to resist infections. d. d. There is a decreased risk of atopic dermatitis in babies who nurse for 12 months or longer.

ANS: C There is evidence that infants who exclusively breastfeed for at least 4 months have less risk for infection than infants breastfed for less time. However, infants who breastfeed exclusively for 9 months or for longer than 12 months may have increased risks for food hypersensitivities and atopic dermatitis. Breast milk is a poor source of vitamin D and iron.

1. During a well child exam, the primary care pediatric nurse practitioner notes yellowish-white serpentine-bordered lesions on the anterior portion of a child's tongue. What will the nurse practitioner do? a. Order chlorhexidine gluconate rinses to treat the lesions. b. Prescribe oral acyclovir to shorten the course of the disease. c. Reassure the parent that these are benign lesions. d. Refer the child to a pediatric dentist for evaluation.

ANS: C These lesions are characteristic of benign migratory glossitis or "geographic tongue" and are benign. Chlorhexidine gluconate rinses, oral acyclovir, and referral to a dentist are not necessary.

16. An adolescent is diagnosed with functional abdominal pain (FAP). The child's symptoms worsen during stressful events, especially with school anxiety. What will be an important part of treatment for this child? a. Informing the parents that the pain is most likely not real b. Instituting a lactose-free diet along with lactobacillus supplements c. Teaching about the brain-gut interaction causing symptoms d. Using histamine2-blockers to help alleviate symptoms

ANS: C This child has symptoms associated with stress, and treatment should be aimed at biobehavioral methods, beginning with teaching about the brain-gut interaction. Even though the pain is functional, it is real. Lactose-free diets and lactobacillus supplements may be used with documented lactose intolerance, although there is a lack of high-quality evidence of their effectiveness. H2-blockers should not be used unless dyspepsia is present.

3. A female infant who was developing normally stops meeting developmental milestones at age 12 months and then begins losing previously acquired skills. What will the primary care pediatric nurse practitioner expect to tell the parents about this child's prognosis? a. Cognitive development will be normal but motor skills will be lost. b. Physical and speech therapy will help the infant regain lost skills. c. The child's intellectual development will not progress further. d. This is a temporary condition with full recovery expected.

ANS: C This child has symptoms of Rett syndrome, which affects females more than males and is characterized by a plateau of development with eventual loss of milestones. Intellectual development remains at the level of plateau. Physical therapy, occupational therapy, and speech therapy help to preserve functional abilities but do not improve skills. The condition is progressive, with variable life expectancy.

15. A 14-year-old child has a 2-week history of severe itching and tearing of both eyes. The primary care pediatric nurse practitioner notes redness and swelling of the eyelids along with stringy, mucoid discharge. What will the nurse practitioner prescribe? a. Saline solution or artificial tears b. Topical mast cell stabilizer c. Topical NSAID drops d. Topical vasoconstrictor drops

ANS: C This child has symptoms of allergic conjunctivitis. Topical NSAIDs work for acute symptoms to reduce inflammation and may be used in children over age 12 years. Saline solution or artificial tears are useful for milder symptoms. Topical mast cell stabilizers are useful for chronic symptoms and maintenance therapy. Topical vasoconstrictors should be avoided because of rebound hyperemia.

1. The primary care pediatric nurse practitioner performs a well baby examination on a 7-day-old infant who is nursing well, according to the mother. The nurse practitioner notes that the infant weighed 3250 grams at birth and 2990 grams when discharged on the second day of life. The infant weighs 3080 grams at this visit. Which action is correct? a. Follow up at the 2-month checkup. b. Refer to a lactation consultant. c. Schedule a weight check in 1 week. d. Suggest supplementing with formula.

ANS: C This infant lost about 8% of its birth weight, which is normal and, since discharge home, has gained at least 15 grams per day, which is also normal. The PNP should schedule a weight check in a week to make sure the infant regains its birth weight, since most should regain this in 10 to 14 days and since this loss of birth weight is at the high end of normal. It is not necessary to refer to a lactation consultant or supplement with formula, since the infant is gaining weight adequately.

18. A school-age child has a rash without fever or preceding symptoms. Physical examination reveals a 3- cm ovoid, erythematous lesion on the trunk with a finely scaled elevated border, along with generalized macular, ovoid lesions appearing in a "Christmas tree" pattern on the child's back. What is the initial action? a. Obtain a KOH preparation of a skin scraping to verify the diagnosis. b. Prescribe topical steroid creams to shorten the course of the disease. c. Reassure the child's parents that the rash is benign and self-limited. d. Recommend topical antihistamines and emollients to control the spread.

ANS: C This rash is typical of pityriasis rosea, a benign, self-limited papulosquamous disease that is not contagious. Patients may be reassured that this is the case. Because the herald lesion is characteristic, it is not necessary to obtain a KOH scraping to look for tinea corporis. Topical steroids do not alter the course of the disease. Topical antihistamines and emollients may be used if itching occurs, but this is not the initial management action.

1. 5. The parent of a 24-month-old child asks the primary care pediatric nurse practitioner when toilet training should begin. How will the pediatric nurse practitioner respond? a. a. "Begin by reading to your child about toileting." b. b. "Most children are capable by age 2 years." c. c. "Tell me about your child's daily habits." d. d. "We should assess your child's motor skills."

ANS: C To assess the parent's understanding of toilet readiness, the nurse practitioner will ask the parents about the child's daily habits and routines to see if the child has predictable patterns that can be the basis for toilet training. While providing storybooks about toileting can help children learn, the first step is to assess toilet readiness. Even though many children are capable at this age, evaluating personal readiness is key to beginning toilet training. Assessment of motor skills may be a second step.

7. The primary care pediatric nurse practitioner is treating a toddler who has a lower respiratory tract illness with a low-grade fever. The child is eating and taking fluids well and has normal oxygen saturations in the clinic. The nurse practitioner suspects that the child has a viral pneumonia and will : a. order an anti-viral medication and schedule a follow-up appointment. b. prescribe a broad-spectrum antibiotic until the lab results are received. c. teach the parents symptomatic care and order labs to help with the diagnosis. d. write a prescription for an antibiotic to be given if the child's condition worsens.

ANS: C To decrease antibiotic overuse and resistance, the PNP should order an antibiotic only if laboratory data confirm a bacterial infection. This child is mildly ill and can be treated symptomatically. It is not necessary to treat with an anti-viral medication. A broad-spectrum antibiotic will only increase the risk of antibiotic resistance. Writing a prescription for the parents to fill if needed is not recommended; parents may give an antibiotic believing that it is indicated when it is not.

1. The single mother of a 4-year-old who attends day care tells the primary care pediatric nurse practitioner that she had difficulty giving her child a twice-daily amoxicillin for 10 days to treat otitis media during a previous episode several months earlier because she works two jobs and is too busy. The child has an ear infection in the clinic today. What will the nurse practitioner do? a. Administer an intramuscular antibiotic. b. Order twice-daily amoxicillin for 5 days. c. Prescribe azithromycin once daily for 5 days. d. Reinforce the need to adhere to the plan of care.

ANS: C To improve adherence, the PNP should shorten the length of treatment, if possible and, if possible, reduce the number of times per day that a medication is given. This mother indicated that she had difficulty giving two doses per day, so a once daily for 5 days medication is ideal. It is not necessary to give an IM injection unless the child refuses to take the medication. Reinforcing the need to adhere to the plan is important but does not address the underlying difficulty associated with scheduling.

5. An adolescent has acne characterized by papules and pustules mostly on the forehead and chin. What will the primary care pediatric nurse practitioner prescribe? a. Azelaic acid applied daily at nighttime b. Benzoyl peroxide applied twice daily c. Topical erythromycin with benzoyl peroxide d. Tretinoin applied nightly after washing the face

ANS: C Topical antibiotics combined with BPO are more effective than either drug alone and are especially effective in mild to moderate inflammatory acne or as adjunctive therapy with oral antibiotics. Azelaic acid is useful in persons with sensitive or dark skin and used for non-inflammatory acne. Topical antibiotics are best used in conjunction with BPO. Tretinoin is a keratolytic, useful for non-inflammatory acne.

2. A school-age child has had herpes stomatitis for a week and continues to complain of pain. What will the primary care pediatric nurse practitioner recommend? a. Administration of a topical antiviral medication b. Taking oral acyclovir for 5 to 7 days c. Topical application of diphenhydramine and Maalox d. Using a chlorhexidine gluconate rinse

ANS: C Topical treatment with diphenhydramine and Maalox is useful for symptomatic relief of herpes stomatitis. Topical antiviral medications are not effective. Oral acyclovir is only effective if initiated within 3 days of onset. Chlorhexidine gluconate is not indicated.

15. The primary care pediatric nurse practitioner reviews the immunization records of an 18-month-old child and notes that the child received an MMR immunization 2 days prior to the first birthday. What will the nurse practitioner do? a. Administer a reduced dose of MMR to ensure adequate immunity. b. Obtain mumps, measles, and rubella titers to determine immunity. c. Recommend the next dose of MMR vaccine at 4 to 5 years of age. d. Repeat the MMR vaccine since the first dose was given too soon

ANS: C Vaccine doses may be given 4 days prior to or later than minimum intervals or ages to provide schedule flexibility. The next dose will be given at age 4 to 5, so this child may remain on schedule. It is never recommended to give reduced doses for any reason

15. The primary care pediatric nurse practitioner reviews the immunization records of an 18-month-old child and notes that the child received an MMR immunization 2 days prior to the first birthday. What will the nurse practitioner do? a. Administer a reduced dose of MMR to ensure adequate immunity. b. Obtain mumps, measles, and rubella titers to determine immunity. c. Recommend the next dose of MMR vaccine at 4 to 5 years of age. d. Repeat the MMR vaccine since the first dose was given too soon

ANS: C Vaccine doses may be given 4 days prior to or later than minimum intervals or ages to provide schedule flexibility. The next dose will be given at age 4 to 5, so this child may remain on schedule. It is never recommended to give reduced doses for any reason. reimmunization if in doubt is not harmful. Titers are not indicated unless the vaccine quality or storage is in doubt.

14. A 16-year-old female has not had a menstrual period yet and is concerned. She denies sexual activity. An exam reveals an adult sexual maturity rating. Which laboratory test will the primary care pediatric nurse practitioner order initially? a. Genetic test for Turner syndrome b. Pituitary hormone tests c. Pregnancy test d. Thyroid function tests

ANS: C When amenorrhea occurs, initial laboratory studies should include a pregnancy test regardless of sexual history. Other tests are ordered after pregnancy is ruled out.

4. The primary care pediatric nurse practitioner is examining a newborn infant recently discharged from the neonatal intensive care unit after a premature birth. The parent is upset and expresses worry about whether the infant will be normal. What will the nurse practitioner do in this situation? a. Explain to the parent that developmental delays often do not manifest at first. b. Perform a developmental assessment and tell the parent which delays are evident. c. Point out the tasks that the infant can perform while conducting the assessment. d. Refer the infant to a developmental specialist for a complete evaluation.

ANS: C When discussing developmental delays with parents, it is important to be positive and to initially focus on strengths. Explaining that developmental delays develop over time is true but does not reassure the parent or help the parent cope with feelings. Referrals are not indicated unless delays are present and may take time.

14. An adolescent female reports unilateral headache pain associated with abdominal pain and nausea occurring just prior to periods each month. The adolescent has been using naproxen sodium for 6 months but reports little relief from symptoms. What will the primary care pediatric nurse practitioner do? a. Add acetaminophen and ondansetron to the naproxen regimen. b. Consider prophylactic therapy with a beta-blocker or anticonvulsant drug. c. Prescribe sumatriptan nasal spray at the onset of headache and every 2 hours. d. Refer the adolescent to a pediatric neurologist for neuroimaging studies.

ANS: C When there is no response from analgesics, sumatriptan can be started as abortive therapy using the nasal spray formulation. Adding acetaminophen and ondansetron is not indicated

21. A child has several circular, scaly lesions on the arms and abdomen, some of which have central clearing. The primary care pediatric nurse practitioner notes a smaller, scaly lesion on the child's scalp. How will the nurse practitioner treat this child? a. Obtain scrapings of the lesions for fungal cultures. b. Order prescription-strength antifungal creams. c. Prescribe oral griseofulvin for 2 to 4 weeks. d. Recommend OTC antifungal creams and shampoos.

ANS: C Whenever tinea lesions occur on the scalp or nails, oral griseofulvin must be given for 2 to 4 weeks. Unless the infection is resistant to treatment, fungal cultures are not necessary. Topical medications alone are not effective for tinea capitus.

12. A school-age child has an abrupt onset of sore throat, nausea, headache, and a temperature of 102.3°F. An examination reveals petechiae on the soft palate, beefy-red tonsils with yellow exudate, and a scarlatiniform rash. A Rapid Antigen Detection Test (RADT) is negative. What is the next step in management for this child? a. Consider a sexual abuse diagnosis. b. Obtain an anti-streptococcal antibody titer. c. Perform a follow-up throat culture. d. Prescribe amoxicillin for 10 days.

ANS: C While an RADT has a high specificity, it has variable sensitivity, and a negative test does not mean that streptococcal infection is not present. A culture should be performed to confirm the diagnosis. If the throat culture is negative for GABHS, other causes, such as gonococcal infection, may be considered but are less likely. The RADT does not assess for sexual abuse. An ASO titer is not useful in the diagnosis of acute pharyngitis, since the titers remain elevated for months after an acute infection. Amoxicillin is not indicated unless infection is confirmed.

5. The parent of a high school basketball player tells the primary care pediatric nurse practitioner that the adolescent becomes short of breath only when exercising. What will the nurse practitioner recommend? a. Permanent discontinuation of all strenuous and aerobic activities b. Enrollment in a conditioning program to improve performance c. Evaluation for underlying cardiac causes of this symptom d. Treatment for exercise-induced asthma with a bronchodilator

ANS: C While shortness of breath may indicate several more benign causes, athletes who exhibit this symptom should be evaluated for underlying cardiac causes to prevent sudden cardiac death. Once this is ruled out, other causes may be considered, such as EIA or poor conditioning.

1. 3. The primary care pediatric nurse practitioner is offering anticipatory guidance to the parents of a 12-month-old child. The parents are bilingual in Spanish and English and have many Spanish-speaking relatives nearby. They are resisting exposing the child to Spanish out of concern that the child will not learn English well. What will the pediatric nurse practitioner tell the parents? a. a. Children who learn two languages simultaneously often confuse them in conversation. b. b. Children with multi-language proficiency do not understand that others cannot do this. c. c. Learning two languages at an early age prevents children from developing a dominant language. d. d. Most bilingual children are able to shift from one language to another when appropriate.

ANS: D Most children who are bilingual are able to sort out the languages in conversation but may "code switch" at times for clarity as they speak. They seem to understand that not everyone has this ability. Most children who are bilingual develop a dominant language.

1. 7. The parent of a child who has numerous allergies reports using herbal remedies to help treat the child because they are "natural" products. What will the primary care pediatric nurse practitioner suggest to this parent about using these products? a. a. Brand-name herbal products have been tested for efficacy. b. b. Natural ingredients are usually safe for use in children. c. c. There are few interactions between drugs and supplements. d. d. Use single-herb supplements whenever possible.

ANS: D Single-herb supplements should be used when possible to prevent side-effect confusion. Brand- name products have not necessarily been tested; randomized controlled trials have not been widely performed. "Natural" does not mean "safe." There are many drug-supplement interactions.

11. The primary care pediatric nurse practitioner performs a well baby exam on a term 4-month-old infant and observes flattening of the left occiput, bossing of the right occiput, and anterior displacement of the left ear. The parents report performing various positioning maneuvers, but say that the baby's head shape has worsened. What will the nurse practitioner recommend to correct this finding? a. Allow the infant to sleep on the tummy when the parents are in the room. b. Lay the infant in the "back to sleep" position, alternating the left and right occiput. c. Order a head CT to evaluate the infant for craniosynostosis. d. Refer the infant for orthotic cranial molding helmet therapy.

ANS: D This infant was term and likely has positional plagiocephaly, which has not responded to repositioning efforts, so a referral should be made for an orthotic helmet. Tummy time is performed when the infant is awake and the parents are present. The "back to sleep" position with alternation of left and right is a repositioning maneuver. Craniosynostosis is characterized by bossing and deformity that follow cranial suture lines.

8. A parent asks about ways to promote dental health in school-age children while on a family vacation that are convenient while camping and picnicking. What will the primary care pediatric nurse practitioner recommend? a. Getting fluoride varnish treatments prior to vacations b. Giving the children fluoridated water after meals c. Having the children use a chlorhexidine gluconate oral rinse d. Offering gum containing xylitol after meals

ANS: D Xylitol has been demonstrated to prevent and control tooth decay when used from 3 to 7 times daily, with topical effects. Chewing gum containing xylitol is effective. It is not necessary to have fluoride varnish treatments prior to vacations. Giving fluoridated water after meals is not indicated. Chlorhexidine gluconate rinses do not prevent caries.

9. During a well child assessment, the primary care pediatric nurse practitioner auscultates a harsh, blowing grade IV/VI murmur in a 6-month-old infant. What will the nurse practitioner do next? a. Get a complete blood count to rule out severe anemia. b. Obtain an electrocardiogram to assess for arrhythmia. c. Order a chest radiograph to evaluate for cardiomegaly. d. Refer to a pediatric cardiologist for further evaluation.

ANS: D A harsh, blowing murmur is suspicious for pathology, so a cardiology referral is warranted. The cardiologist will determine which tests and procedures should be performed.

13. The primary care pediatric nurse practitioner reviews a child's complete blood count with differential white blood cell values and recognizes a "left shift" because of a. a decreased eosinophil count. b. a decreased lymphocyte count. c. an elevated monocyte count. d. an elevated neutrophil count.

ANS: D A left shift occurs when there is an increase in the number of circulating immature neutrophils and indicates a bacterial infection or an inflammatory disorder. Eosinophils are associated with an antigen- antibody response and are elevated with exposure to allergens, inflammation of skin, or parasites. Lymphocytes are non-granulocytes that are elevated with viral infections. Monocytes are non- granulocytes and are elevated in infections and inflammation and some leukemias, elevations of non- granulocytes are referred to as a "right shift."

2. A toddler who was born prematurely refuses most solid foods and has poor weight gain. A barium swallow study reveals a normal esophagus. What will the primary care pediatric nurse practitioner consider next to manage this child's nutritional needs? a. Consultation with a dietician b. Fiberoptic endoscopy evaluation c. Magnetic resonance imaging d. Videofluoroscopy swallowing study

ANS: D A videofluoroscopy swallowing study will evaluate other structural defects that may interfere with swallowing and is relatively non-invasive. A dietician consult may be a part of the overall plan, but the toddler first needs a thorough evaluation of potential problems. Fiberoptic endoscopy is invasive. MRI may be performed if videofluoroscopy is inconclusive, but this is an expensive test.

11. The mother of a 16-year-old male was recently divorced after several years of an abusive relationship and tells the primary care pediatric nurse practitioner that the adolescent has begun skipping school and hanging out with friends at the local shopping mall. When she confronts her child, he responds by saying that he hates her. What will the nurse practitioner tell this mother? a. Adolescence is marked by an inability to comprehend complex situations. b. Adolescence is typically marked by tempestuous and transient episodes. c. Adolescents normally have extreme, disruptive conflicts with parents. d. Adolescents often need counseling to help them cope with life events.

ANS: D Adolescent brains respond differently to toxic stress, so counseling is indicated to help them manage serious events, such as family abuse and divorce. Early adolescents have concrete thinking, but the formal operational thinking occurs later. "Storm and stress" are not the norm in adolescence nor are disruptive periods of conflict.

9. The primary care pediatric nurse practitioner is performing an exam on an adolescent male who asks about sexual identity because of concern that a friend is worried about being gay. Which response will the nurse practitioner make in this situation? a. Provide the teen with a questionnaire to gain information about his sexuality. b. Remind the adolescent that mandatory reporting requires disclosure to parents. c. Suggest that the adolescent discuss sexual concerns with his parents. d. Tell the adolescent that, unless he is at risk, what he says will be confidential.

ANS: D Adolescents should be encouraged to divulge information about their sexuality to providers by assuring them that confidentiality will be maintained unless the health of the child or others is at risk. The adolescent may be trying to ask questions about himself in a manner that doesn't implicate his own sexuality, so the PNP should attempt to gain his confidence. Questionnaires may be useful when collecting information, but this adolescent has already begun a discussion about the topic. An adolescent who is concerned about being gay may not be ready to come out to his parents.

6. The parents of a 12-year-old child are concerned that some of the child's older classmates may be a bad influence on their child, who, they say, has been raised to believe in right and wrong. What will the primary care pediatric nurse practitioner tell the parent? a. Allowing the child to make poor choices and accept consequences is important for learning values b. Children at this age have a high regard for authority and social norms, so this is not likely to happen c. Moral values instilled in the early school-age period will persist throughout childhood d. The pressures from outside influences may supersede parental teachings and should be confronted

ANS: D Although early school-age children learn values from their parents, these may be challenged as children learn that others have different values. Parents must confront and negotiate these issues daily with their children. While children may make poor choices and subsequently learn from the consequences, it is best for parents to actively discuss these issues with their children. Children do have a high regard for authority and social norms but may easily transfer this authority to other, less reliable people, such as peers. Moral values may not persist if other sources of authority become prominent.

14. The primary care pediatric nurse practitioner is obtaining a medical history about a child. To integrate both nursing and medical aspects of primary care, which will be included in the medical history? a. Complementary medications, alternative health practices, and chief complaint b. Developmental delays, nutritional status, and linear growth patterns c. Medication currently taking, allergy information, and family medical history d. Speech and language development, beliefs about health, and previous illnesses

ANS: D An assessment model that integrates the nursing and medical aspects of primary care uses three domains: developmental problems (speech and language development), functional health problems (beliefs about health), and diseases (chief complaint). The other examples all use domains associated with the traditional medical model and do not contain nursing aspects associated with functional health problems.

6. During a well child assessment of an African-American infant, the primary care pediatric nurse practitioner notes a dark red-brown light reflex in the left eye and a slightly brighter, red-orange light reflex in the right eye. The nurse practitioner will a. dilate the pupils and reassess the red reflex. b. order auto-refractor screening of the eyes. c. recheck the red reflex in 1 month. d. refer the infant to an ophthalmologist.

ANS: D Any asymmetry, dark or white spots, opacities, or leukokoria should be referred immediately to a pediatric ophthalmologist. The PNP does not dilate pupils or order auto-refractor exams; these are done by an ophthalmologist. Because retinoblastoma is a concern, any unusual finding should be immediately referred.

4. During a well child examination on an infant who has colic, the primary care pediatric nurse practitioner learns that the infant's mother is 17 years old and that the father, who is in the military, was deployed to wartime duty shortly after the baby was born. To determine the immediate risk of child maltreatment for this infant, the nurse practitioner will ask about : a. childrearing and parenting styles. b. role responsibilities of the parents. c. spiritual beliefs and religious practices. d. the location of extended family members.

ANS: D Assessment of resources, including the support of extended family members, is a key dimension of family functioning. In this case, the mother is young and alone and may lack the skills needed to cope with an infant with colic. Childrearing and parenting styles can affect the emotional and physical health of children who misbehave or who are learning how to behave in the world. An assessment of role responsibilities is important when there are disagreements about shared responsibilities. Assessing spiritual beliefs helps to determine the values ascribed to events. While all of these are important assessments, there is an urgent need to determine the level of support available to this mother.

15. The parent of a school-age child is concerned because the child has started to express anger about a grandparent's death even though this occurred when the child was a toddler. What will the primary care pediatric nurse practitioner tell the parent? a. Anger is an abnormal reaction to bereavement and loss in this age child. b. Counseling is needed since the child has had sufficient time to resolve this issue. c. Grief and bereavement lasting longer than a year may require medication. d. The significance of this loss must be reworked at each developmental level.

ANS: D At any given developmental stage, children resolve the effect of the death only at that developmental level and thus must rework the significance of the loss at each stage of development. Anger is a common reaction to loss in school-age children. Counseling and medication are not indicated since this is a normal response. 16. The primary care pediatric nurse practitioner attempts to learn more about the emotional health of an

13. The primary care pediatric nurse practitioner is assessing a toddler whose weight and body mass index (BMI) are below the 3rd percentile for age. The nurse practitioner learns that the child does not have regular mealtimes and is allowed to carry a bottle of juice around at all times. The nurse practitioner plans to work with this family to develop improved meal patterns. Which diagnosis will the nurse practitioner use for this problem? a. Failure to thrive b. Home care resources inadequate c. Nutrition alteration - less than required d. Parenting alteration

ANS: D Because the PNP is planning to intervene by helping the parents to provide appropriate food habits, the correct diagnosis should be "Parenting alteration." "Failure to thrive" is a medical diagnosis and requires a medical and social evaluation to rule out organic causes or detect neglect. "Home care resources inadequate" would be used if the PNP suspects that the family lacks adequate funds to purchase food. "Nutrition alteration" is a NANDA diagnosis and would be used if the PNP planned to consult with a dietician or give nutritional information.

3. An infant is brought to clinic with bright erythema in the neck and flexural folds after recent treatment with antibiotics for otitis media. What is the treatment for this condition? a. 1% hydrocortisone cream to affected areas for 1 to 2 days b. Oral fluconazole 6 mg/kg on day 1, then 3 mg/kg/dose for 14 days c. Topical keratolytics and topical antibiotics for 7 to 10 days d. Topical nystatin cream applied several times daily

ANS: D Candida skin infections can occur in intertriginous areas in the neck, axilla, and groin, and appear as a bright erythematous rash. Topical nystatin is first-line therapy. Fifteen percent hydrocortisone is used if inflammation is severe but not instead of topical antifungal therapy. Oral fluconazole is used if resistant to treatment. Keratolytics and antibiotics are used to treat superficial folliculitis.

16. During a well child exam on a 4-year-old child, the primary care pediatric nurse practitioner notes that the clinic nurse recorded "20/50" for the child's vision and noted that the child had difficulty cooperating with the exam. What will the nurse practitioner recommend? a. Follow up with a visual acuity screen in 6 months. b. Refer to a pediatric ophthalmologist. c. Re-test the child in 1 year. d. Test the child's vision in 1 month.

ANS: D Children age 4 years and older who have difficulty cooperating with a vision screen should be retested in 1 month; if they continue to have difficulty cooperating, they should be referred for a formal examination. Children who are 3 years old should be re-evaluated in 6 months.

1. 1. A child has a 1-cm laceration on the forehead proximal to the hairline after running into a pole while playing sports. To minimize the risk of infection, the primary care pediatric nurse practitioner will irrigate the wound and a. a. allow the wound to heal by secondary intention. b. b. delay closure of the wound for several days. c. c. refer the child to a plastic surgeon for wound closure. d. d. suture the wound within 6 hours.

ANS: D Children are less likely than adults to get wound infections, with an infection rate from sutured lacerations at 2%. The PNP should clean and suture the wound. Wounds from animal bites are often left to heal by secondary intention to prevent infection. Referral to a plastic surgeon is necessary for cosmetic reasons. Delaying closure for several days is recommended for heavily contaminated wounds and those caused by high-velocity missile injuries, crush injuries, and explosion injuries.

1. 1. The parent of a 4-year-old points to a picture and says, "That's your sister." The child responds by saying, "No! It's my baby!" This is an example of which type of thinking in preschool-age children? a. a. Animism b. b. Artificialism c. c. Egocentrism d. d. Realism

ANS: D Children at this age are developing their ability to establish causality. Nominal realism occurs when children think that one type of thing can only be called by one name. All dogs are dogs and not various breeds. Animism refers to the belief that objects possess person-like qualities. Artificialism occurs when children think things are caused by a controlling force. Egocentrism is when children see things only as they relate to themselves.

5. The primary care pediatric nurse practitioner is counseling a family whose parents are divorcing. To help support the children and reduce their stress through this process, the nurse practitioner will recommend a. allowing children to choose the custodial parent. b. being open about ongoing parental conflicts. c. establishing a single custody living arrangement. d. maintaining a civil relationship when discussing children.

ANS: D Children make more successful adjustments and suffer less stress when parents are able to develop a civil relationship that focuses on what is best for them. Custody arrangements vary and may change over time and should be determined by what is best for the children. Custody disputes and exposure to parental conflict place additional stress on children and can increase their feelings of insecurity

1. 7. An adolescent who has asthma and severe perennial allergies has poor asthma control in spite of appropriate use of a SABA and a daily high-dose inhaled corticosteroid. What will the primary care pediatric nurse practitioner do next to manage this child's asthma? a. a. Consider daily oral corticosteroid administration. b. b. Order an anticholinergic medication in conjunction with the current regimen. c. c. Prescribe a LABA/inhaled corticosteroid combination medication. d. d. Refer to a pulmonologist for omalizumab therapy.

ANS: D Children older than 12 years who have moderate to severe allergy-related asthma and who react to perennial allergens may benefit from omalizumab as a second-line treatment when symptoms are not controlled by ICSs. The PNP should refer children to a pulmonologist for such treatment. Daily oral corticosteroid medications are not recommended because of the adverse effects caused by prolonged use of this route. Anticholinergic medications are generally used for acute exacerbations during in-patient stays or in the ED. A LABA/ICS combination will not produce different results.

18. A 13-year-old child has exhibited symptoms of mild depression for several weeks. The parent reports feeling relieved that the symptoms have passed but concerned that the child now seems to have boundless energy and an inability to sit still. What will the primary care pediatric nurse practitioner do? a. Administer an ADHD diagnostic scale and consider an ADHD medication. b. Consult with a child psychiatrist to prescribe an antidepressant medication. c. Reassure the parent that this behavior is common after mild depressive symptoms d. Refer the child to a child psychiatrist for evaluation of bipolar disorder.

ANS: D Children who have ADHD symptoms and depression should be evaluated by a child psychiatrist for bipolar disorder. Medications are not appropriate until the disorder is correctly diagnosed. Stimulant medications are not effective in treating bipolar disorder. Antidepressants may potentiate manic responses. Providers should carefully evaluate and refer any child treated for ADHD who does not respond to therapy or who experiences a sudden worsening of agitation while using ADHD medications.

13. A 12-year-old child whose BMI is greater than the 95th percentile has a blood pressure at the 98th percentile for age, sex, and height. After lifestyle changes that include diet and exercise, the child's BMI drops to the 90th percentile, but the blood pressure remains the same. What is the primary care pediatric nurse practitioner's next step in treating this child? a. Continued close monitoring of blood pressure b. Ordering an echocardiogram or MRI c. Prescribing an ACE inhibitor medication d. Referral to a nephrologist or cardiologist

ANS: D Children who have persistent BP elevation after lifestyle changes are made should be referred to a nephrologist or cardiologist who has experience using antihypertensive agents in children. The specialist orders necessary tests and medications, not the primary care provider.

2. A 7-month-old infant has had two prior acute ear infections and is currently on the 10th day of therapy with amoxicillin-clavulanate after a failed course of amoxicillin. The primary care pediatric nurse practitioner notes marked middle ear effusion and erythema of the TM. The child is irritable and has a temperature of 99.8°F. What is the next step in management of this child's ear infection? a. Order a second course of amoxicillin-clavulanate. b. Perform tympanocentesis for culture. c. Prescribe clindamycin twice daily. d. Refer the child to an otolaryngologist.

ANS: D Children who have persistent infection who have failed appropriate therapy and those who have had three or more episodes of AOM in 6 months should be referred to an otolaryngologist. Ceftriaxone is ordered when Augmentin fails. The PNP does not perform tympanocentesis. Clindamycin is used for ceftriaxone failure but only if the susceptibilities are known.

9. The pediatric nurse practitioner provides primary care for a 5-year-old child who has cerebral palsy who exhibits athetosis and poor weight gain in spite of receiving high-calorie formula to supplement intake. The child has had several episodes of pneumonia in the past year. Which specialty consultation is a priority for this child? a. Feeding clinic to manage caloric intake b. Neurology to assess medication needs c. Pulmonology for possible tracheotomy d. Surgery for possible fundoplication and gastrostomy

ANS: D Children with CP who have athetosis often have increased calorie needs up to 50% to 100% higher than others. This child is unable to gain adequate weight in spite of receiving extra calories. The child also has possible aspiration pneumonia, probably due to difficulty swallowing or GERD. A fundoplication and gastrostomy can help to prevent GERD and to provide nutrition that doesn't involve swallowing. The feeding clinic would increase calories and nutrients but, without a gastrostomy, cannot increase actual intake. The child is not having seizures or drooling that contribute to this problem, so medications aren't necessary. Unless there is an airway problem, tracheotomy is not indicated.

1. A 5-year-old child who had a repair for transposition of the great arteries shortly after birth is growing normally and has been asymptomatic since the surgery. The primary care nurse practitioner notes mild shortness of breath with exertion and, upon questioning, learns that the child has recently complained of dizziness. What will the nurse practitioner do? a. Order an echocardiogram and chest radiograph. b. Perform pulmonary function testing. c. Reassure the parent that these symptoms are common. d. Refer the child to the cardiologist immediately.

ANS: D Children with a history of d-TGA who have a history of palpitations, syncope, or shortness of breath should be referred to a cardiologist. Echocardiograms should be performed annually under the supervision of the cardiologist. Pulmonary function testing is not indicated. These symptoms may represent problems in patency with the coronary arteries and are not common.

1. 6. A school-age child who uses a SABA and an inhaled corticosteroid medication is seen in the clinic for an acute asthma exacerbation. After 4 puffs of an inhaled short-acting B2-agonist (SABA) every 20 minutes for three treatments, spirometry testing shows an FEV1 of 60% of the child's personal best. What will the primary care pediatric nurse practitioner do next? a. a. Administer an oral corticosteroid and repeat the three treatments of the inhaled SABA. b. b. Admit the child to the hospital for every 2 hour inhaled SABA and intravenous steroids. c. c. Give the child 2 mg/kg of an oral corticosteroid and have the child taken to the emergency department. d. d. Order an oral corticosteroid, continue the SABA every 3 to 4 hours, and follow closely.

ANS: D Children with an incomplete response (FEV1 between 40% and 69% of personal best) should be given oral steroids and instructed to continue the SABA every 3 to 4 hours with close follow- up. Hospitalization is not necessary unless severe distress occurs. An FEV1 less than 40% after treatment indicates a need to be seen in the ED.

15. A 4-year-old child who has asthma has teeth with smooth, cupped-out teeth on the chewing surfaces. Which is the most likely explanation for this finding? a. Bruxism b. Bulimia c. Decreased saliva d. Gastroesophageal reflux

ANS: D Children with asthma have higher rates of GERD, so this is the most likely cause of dental erosion. Bruxism causes marked wear of teeth on the chewing surfaces. Bulimia can cause dental erosion but is unlikely in a 4-year-old child. Decreased saliva occurs with some medications.

1. 5. A school-age child sustained a contusion on the front of one thigh while playing football and reports some difficulty flexing his foot on the affected side. What will the primary care pediatric nurse practitioner do to treat this injury? a. a. Place the child on crutches and limit weight-bearing until symptoms subside. b. b. Prescribe acetaminophen with hydrocodone along with NSAIDs. c. c. Recommend rest, ice packs, compression, and elevation of the extremity. d. d. Refer the child to an orthopedic specialist for immediate evaluation and treatment.

ANS: D Children with contusions that restrict movement or sensation and those affecting the quadriceps muscle may include compartment syndrome. These children should be referred to orthopedic specialists immediately so that the compartment pressure does not result in irreplaceable damage. The other options may be performed in consultation with a specialist.

7. A child has a difficult temperament. What will the primary care pediatric nurse practitioner tell the parent about managing this child's behavior? a. A difficult temperament is its own risk factor for maladjustment disorders. b. Children with difficult temperaments need strict adherence to rules. c. Having a difficult temperament limits intelligence and emotional maturity d. It is important for the parent to learn to manage criticism and power struggles.

ANS: D Children with difficult temperaments tend to engender parental criticism and irritability, power struggles, and restrictive parenting, and this dynamic leads to difficulties with psychosocial adjustment. Parents of children with difficult temperaments must be aware of behavioral manifestations as temperament expressions in order to reframe their own responses to these behaviors. A difficult temperament alone is not a risk factor for maladjustment. Restrictive parenting only increases power struggles and maladaptive behaviors. Temperament is unrelated to IQ but does affect academic outcomes because of behavior issues in school.

10. A school-age child has frequent nosebleeds. Nasal visualization reveals fresh clots and excoriated nasal mucosa but no visible site of bleeding. Coagulation studies are normal. In spite of symptomatic measures, the child continues to have nosebleeds. What is the next course of action? a. Cauterize the mucosa with silver nitrate sticks. b. Order a topical vasoconstrictor medication. c. Prescribe a barrier agent such as petrolatum jelly. d. Refer to an otolaryngologist for further evaluation.

ANS: D Children with persistent epistaxis should be referred for evaluation and treatment after usual symptomatic measures are ineffective. Cautery works well for exposed vessels, but the site must be easily accessible, visible, and not bleeding briskly. Topical vasoconstrictors are occasionally used. Petrolatum jelly has not been shown to be effective.

3. The primary care pediatric nurse practitioner is evaluating a 16-year-old adolescent male who is on the high school wrestling team and whose weight fluctuates as much as 7 or 8 pounds before matches. The child is eager to talk about the various trophies he has won. When he expresses confidence that he will get a wrestling scholarship for college, his father remarks that his grades will never be good enough for college, causing him to blame his teachers. The nurse practitioner may identify potential problems with a. body image. b. personal identity. c. role performance. d. self-esteem.

ANS: D Children with poor self-esteem seek attention, importance, and security and may become self-absorbed with external markers of self-worth, such as performance in a sport. Another mark of insecurity is defensiveness, which this child exhibits by blaming his teachers for his poor grades. Children with body image problems become overly concerned with appearance and compare themselves to others. This child is losing and gaining weight to be better at wrestling, not to look different. Children with personal identity issues internalize negative perceptions of others and manifest feelings of inferiority. Children with role performance problems feel incompetent and are hesitant to try new things or become perfectionists to overcompensate.

1. 20. An adolescent female reports poor sleep, fatigue, muscle and joint paint, and anxiety lasting for several months. The primary care pediatric nurse practitioner notes point tenderness at several sites. What will the nurse practitioner do next? a. a. Evaluate the adolescent's pain using a numeric pain scale. b. b. Obtain ANA, CBC, liver function, and muscle enzymes tests. c. c. Reassure the adolescent that this condition is not life-threatening. d. d. Refer the adolescent to a rheumatologist for further evaluation.

ANS: D Children with widespread musculoskeletal pain and painful point tenderness may have fibromyalgia and should be referred. The Widespread Pain Index is used to define the degree of pain. Laboratory studies are of little benefit when diagnosing fibromyalgia. Even though children need reassurance that this disease is not life-threatening, this is not the next action.

11. The primary care pediatric nurse practitioner is counseling the parent of an 8-year-old child who has primary nocturnal enuresis. The nurse practitioner recommends an enuresis alarm, but the parent wishes to use medication. What will the nurse practitioner tell the parent? a. Anticholinergic medications are most commonly used for enuresis. b. Drug therapy is an effective way to achieve long-term control. c. Drug therapy is safest when the nasal spray form is used. d. The combination of alarm therapy and intermittent drug therapy is best.

ANS: D Drug therapy in combination with alarm therapy is effective, but parents must be cautioned to use drug therapy only when the child must be dry at night, such as when on a sleepover. Anticholinergic medications can cause constipation, which may exacerbate the problem and are not recommended as first- line treatments. Drug therapy alone can result in relapse of nocturnal enuresis when the drug is withdrawn and is not effective for long-term control. The nasal spray form of desmopressin has a black box warning because of a risk of hyponatremia.

1. The primary care pediatric nurse practitioner sees a 17-year-old client who quit smoking almost a year prior but who reports having renewed cravings when around friends who smoke. Using knowledge of the maintenance stage of change, the primary care pediatric nurse practitioner will a. go over with the adolescent about the health risks associated with smoking. b. recommend avoiding friends who smoke and making new friends. c. remind the adolescent about the struggles associated with quitting smoking. d. suggest that the teen consider taking up a sport or other physical activity.

ANS: D During the maintenance stage, the PNP should help the client with plans to prevent relapse, including establishing new behaviors as long-term changes. Adding a sport can help the client enjoy the benefits of not smoking, while substituting one activity for another. The adolescent already knows about the health risks and the difficulties of quitting. Suggesting giving up friends may be seen as another hardship and is not part of the maintenance stage.

3. A school-age child has a 3-month history of dull, aching epigastric pain that worsens with eating and awakens the child from sleep. A complete blood count shows a hemoglobin of 8 mg/dL. What is the next step in management? a. Administration of H2RA or PPI medications b. Empiric therapy for H. pylori (HP) c. Ordering an upper GI series d. Referral for esophagogastroduodenoscopy (EGD)

ANS: D EGD is the procedure of choice in children for detecting PUD because it allows direct visualization of mucosa, localization of the source of bleeding, and collection of tissue specimens. Empiric therapy for HP is not recommended due to increased antibiotic resistance. An upper GI series may have false negative findings. Once peptic ulcer disease is diagnosed, H2RA or PPI medications are first-line drugs.

9. The primary care pediatric nurse practitioner performs a vision screen on a 4-month-old infant and notes the presence of convergence and accommodation with mild esotropia of the left eye. What will the nurse practitioner do? a. Patch the right eye to improve coordination of the left eye. b. Reassure the parents that the infant will outgrow this. c. Recheck the infant's eyes in 2 to 4 weeks. d. Refer the infant to a pediatric ophthalmologist.

ANS: D Esotropia that continues or occurs at 3 to 4 months of age is abnormal, so the infant should be referred to a pediatric ophthalmologist. The PNP does not determine whether an eye patch should be used. Because it is abnormal at this age, the PNP will not reassure the parents that the infant will outgrow this. Esotropia after 3 to 4 months of age must be evaluated by a specialist and not reevaluated in 2 to 4 weeks.

9. A 4-year-old female who has had two urinary tract infections has persistent dysuria and genital redness. The physical exam reveals a thin, flat membrane from the posterior fourchette almost to the clitoris. Which treatment is indicated? a. Application of A&D ointment b. Counseling about hygiene c. Reassurance and observation d. Use of estrogen-containing cream

ANS: D Estrogen-containing 1% cream should be used when children are symptomatic, especially when UTI and pain occur. A&D ointment is used for milder symptoms and when the opening allows drainage. Reassurance, observation, and counseling about hygiene are indicated when UTI and obstruction do not occur.

14. A child whose parents have refused vaccines has been exposed to chickenpox, and the parents ask whether the child may attend day care. What will the primary care pediatric nurse practitioner tell them? a. The child may attend day care as long as no rash is present even with mild fever or other symptoms. b. The child should remain home and receive oral acyclovir for 5 days to prevent onset of symptoms. c. The child should stay home until the 21-day incubation period has passed even if symptom free. d. The child should stay home if any symptoms occur and may return in 1 week if no rash develops.

ANS: D Exposed children can attend school for about a week but should be kept out of school if any symptoms develop for 1 week and may return if no rash develops. Any symptomatic child should be kept home for 1 week. Oral acyclovir is not effective in preventing disease onset. It is not necessary to remain home for the duration of the incubation period.

3. The primary care pediatric nurse practitioner in a community health center meets a family who has recently immigrated to the United States who speak only Karon. They arrive in the clinic with a church sponsor, who translates for them. The pediatric nurse practitioner notices that the sponsor answers for the family without giving them time to speak. The pediatric nurse practitioner will : a. ask the sponsor to allow the family to respond. b. develop the plan of care and ask the sponsor to make sure it is followed. c. request that the sponsor translate written instructions for the family. d. use the telephone interpreter service to communicate with the family.

ANS: D Federally funded managed care networks and community health centers are required to have interpreters accessible for clients with limited English proficiency. A commercial telephone interpreter service has been shown to be as effective as an "in-person" interpreter. Relying on family members or community members may not be reliable and may jeopardize patient confidentiality. This interpreter is answering for the clients without hearing what they have to say, which can compromise care.

6. A 15-year-old female basketball player who has secondary amenorrhea is evaluated by the primary care pediatric nurse practitioner who notes a BMI in the 3rd percentile. What will the nurse practitioner counsel this patient? a. That amenorrhea in female athletes is not concerning b. That she should begin a program of plyometrics and strength trainin c. To consider a different sport, such as volleyball d. To work with a dietician to improve healthy weight gain

ANS: D Female athletes who have amenorrhea have an increased risk of stress fractures. The adolescent should work to attain a healthy weight, which should allow normal periods to return and reduce this risk. Even though amenorrhea in female athletes is common, it is concerning. Plyometrics and volleyball can increase the risk of stress fractures since both involve jumping and thus not be suggested.

13. A 30-month-old girl who has been toilet trained for 6 months has daytime enuresis and dysuria and a low-grade fever. A dipstick urinalysis is negative for leukocyte esterase and nitrites. What is the next step? a. Begin empiric treatment with trimethoprim-sulfamethoxazole. b. Discuss behavioral interventions for toilet training. c. Reassure the child's parents that the child does not have a urinary tract infection. d. Send the urine to the lab for culture.

ANS: D Girls over age 24 months have a higher risk than boys for UTI. This child is symptomatic, so her urine should be cultured even though the leukocyte esterase and nitrites are negative; urine in the bladder less than 4 hours may be tested as negative for leukocyte esterase. Empiric treatment may be initiated if the child had signs of sepsis. Behavioral interventions are not indicated - the child has dysuria and fever along with enuresis. Until the culture is found to be negative, it is not certain that the child does not have an UTI, and thus reassurance is not the correct action.

1. 12. When counseling an adolescent with a family history of hyperinsulinemia and type 2 diabetes, the primary care pediatric nurse practitioner will recommend avoiding a. a. baked potato chips. b. b. canned vegetables. c. c. high-fiber cereals. d. d. processed breads.

ANS: D High-glycemic foods, such as soda, sweetened juices, and processed breads, pastries, and crackers are more quickly converted to serum glucose and stimulate a sharp rise in insulin production and a subsequent rapid shift into hypoglycemia. To help prevent this in a child with a family history of this disorder, the PNP should recommend avoiding processed breads, pastries, and crackers. High levels of fructose and low fiber intake also contributes to this phenomenon. Baked potato chips, canned vegetables, and high-fiber cereals do not contribute to excess insulin production.

1. 3. A school-age child with asthma is seen for a well child checkup and, in spite of "feeling fine," has pronounced expiratory wheezes, decreased breath sounds, and an FEV1 less than 70% of personal best. The primary care pediatric nurse practitioner learns that the child's parent administers the daily medium-dose ICS but that the child is responsible for using the SABA. A treatment of 4 puffs of a SABA in clinic results in marked improvement in the child's status. What will the nurse practitioner do? a. a. Have the parent administer all of the child's medications. b. b. Increase the ICS medication to a high-dose preparation. c. c. Reinforce teaching about the importance of using the SABA. d. d. Teach the child and parent how to use home PEF monitoring.

ANS: D Home PEF monitoring is useful for children to identify when symptoms are worsening. This child does not appear to notice the presence of airway tightness or wheezing and so might benefit from PEF monitoring to know when to use the SABA. School-age children should be learning how to manage their chronic disease, so having the parent administer all medications is not the best choice, especially since use of the SABA is still dependent on the child's report of symptoms. Since the child responded well to administration of the SABA, increasing the dose of ICS should not be done unless better management is not effective. Reinforcing the teaching is part of the plan but, unless the child is aware of symptoms, may not occur.

1. 8. A child is bitten by a snake near a swimming pool in an area where copperhead snakes are known to inhabit, although the parents cannot describe the snake. An examination of the bite reveals a severe local reaction at the site with edema and intense pain. What will the primary care pediatric nurse practitioner do first? a. a. Administer narcotic analgesics to provide comfort. b. b. Begin treatment with oral amoxicillin-clavulanate for 5 days. c. c. Clean the wound and administer tetanus prophylaxis. d. d. Transport the child by ambulance to a medical center.

ANS: D If a venomous snakebite is suspected, rapid transportation to a medical center with referral to appropriate specialists and antivenin therapy is indicated. Narcotics may impair clinical evaluation. Non-venomous snakebites are treated with oral antibiotics if signs of infection are present after the wound is cleaned and tetanus prophylaxis is given.

16. The clean catch urine specimen of a child with dysuria, frequency, and fever has a colony count between 50,000 and 100,000 of E. coli. What is the treatment for this child? a. Obtain a complete blood count and C-reactive protein. b. Perform sensitivity testing before treating with antibiotics. c. Repeat the culture if symptoms persist or worsen. d. Treat with antibiotics for urinary tract infection.

ANS: D If children are symptomatic and have more than 10,000 colonies of a single pathogen, they are considered to have a UTI and are treated. If pyelonephritis symptoms such as flank pain and sepsis are present, CBC and CRP are useful tests. Sensitivity testing is done for patients who appear toxic, have pyelonephritis, or are non-responsive to antibiotics.

10. The primary care pediatric nurse practitioner is evaluating a 12-year-old girl who reports penile penetration of her vagina by her mother's boyfriend the day before yesterday. The PNP reports this to the local child abuse hotline. What is the PNP's next action? a. Attaining a history of the abuse from the child b. Obtaining urethral specimens for STI testing c. Performing a colposcopic examination to evaluate for trauma d. Referring the child to the ED for forensic specimen collection

ANS: D If sexual abuse has occurred within 72 hours, it is required that appropriate forensic specimens be collected. Getting a history from the child is part of the child abuse evaluation and will be done by the child abuse team, as well as obtaining urethral specimens for STI. Colposcopic exams should be done by an expert in sexual abuse if trauma is suspected but is not performed by the PCPNP.

1. A parent is concerned that a 12-month-old child is "bow-legged." A physical examination reveals internal tibial torsion bilaterally. A radiograph reveals asymmetric bowing of the legs with an angle greater than 15 degrees. What is the correct action for the primary care pediatric nurse practitioner? a. Observe the child's condition over time to assess progression. b. Order physical therapy to prevent progression of symptoms. c. Reassure the parent that the child will outgrow this deformity. d. Refer to a pediatric orthopedic specialist for treatment.

ANS: D In Blount disease, the bowing is asymmetrical; children with this disorder need immediate referral to an orthopedist. Physiologic bowing may self-resolve and may be managed with observation, physical therapy, and reassurance.

4. The mother of a 6-week-old breastfeeding infant tells the primary care pediatric nurse practitioner that her baby, who previously had bowel movements with each feeding, now has a bowel movement once every third day. What will the nurse practitioner tell her? a. Her baby is probably constipated. b. It may be related to her dietary intake. c. She should consume more water. d. This may be normal for breastfed babies.

ANS: D Infants begin to have fewer bowel movements and may have bowel movements ranging from once or twice daily to once every other day when breastfed. Unless there are other signs, the baby is probably not constipated. The mother does not need to change her intake of foods or water, unless constipation is present.

1. 2. The mother of a 2-month-old infant tells the primary care pediatric nurse practitioner that she is afraid her breast milk is "drying up" because her baby never seems satisfied and wants to nurse all the time. Which action is correct? a. a. Recommend pumping her breasts after feedings. b. b. Refer the mother to a lactation consultant. c. c. Suggest supplementation with formula. d. d. Weigh the infant to assess for a growth spurt.

ANS: D Infants have growth spurts about every 3 to 4 weeks that increase their breast milk needs. Until the mother's milk supply catches up, the infant will act hungry and want to nurse more frequently. The PNP should evaluate for this growth spurt and then instruct the mother to feed her baby more often to increase her milk supply. Since the infant is hungry, the infant should nurse. It is not necessary to refer for a lactation consultation or to supplement with formula.

1. The primary care pediatric nurse practitioner is treating an infant with lacrimal duct obstruction who has developed bacterial conjunctivitis. After 2 weeks of treatment with topical antibiotics along with massage and frequent cleansing of secretions, the infant's symptoms have not improved. Which action is correct? a. Perform massage more frequently. b. Prescribe an oral antibiotic. c. Recommend hot compresses. d. Refer to an ophthalmologist.

ANS: D Infants treated for a secondary bacterial conjunctivitis with lacrimal duct obstruction who do not improve after 1 to 2 weeks of topical antibiotic therapy must be referred to an ophthalmologist for possible lacrimal duct probe. Performing the massage more often or applying hot compresses will not help clear the infections. Oral antibiotics are not indicated.

1. 5. The primary care pediatric nurse practitioner sees a 3-day-old nursing infant whose newborn metabolic screen is positive for galactosemia. The nurse practitioner refers the newborn to a specialist for immediate evaluation and will tell the mother a. a. to continue to breastfeed her infant. b. b. to give the infant a cow's milk formula. c. c. to supplement breast milk with formula. d. d. to stop breastfeeding immediately.

ANS: D Infants with galactosemia cannot consume galactose, which is in both cow's milk and breast milk. Since there is a potential for a life-threatening response, the mother should be instructed to stop nursing immediately.

3. During a well baby examination of a 6-week-old infant, the primary care pediatric nurse practitioner notes poor weight gain, acrocyanosis of the hands and feet, and a respiratory rate of 60 breaths per minute. Oxygen saturation on room air is 93%. The remainder of the exam is unremarkable. Which action is correct? a. Follow-up in 1 week to assess the infant's weight. b. Order a chest radiograph and an electrocardiogram. c. Reassure the parents that the exam is within normal limits. d. Refer the infant to a pediatric cardiologist.

ANS: D Infants with oxygen saturation less than 95% and those with poor feeding should be referred emergently to a cardiologist. The infant may have CHF and will need to be evaluated.

2. The mother of a 3-month-old male infant tells the primary care pediatric nurse practitioner that she occasionally notices he has a penile erection just after nursing. What will the nurse practitioner tell the mother? a. Infants should be prevented from masturbating. b. The infant is conscious of the pleasure associated with nursing. c. This is a form of infantile priapism. d. This is a normal, reflexive behavior at this age.

ANS: D Newborn infants are reflexive beings, and sexual reflexes, which are present prenatally, are easily stimulated. A penile erection may occur while nursing. Infants explore with their hands and may touch their own genitalia for pleasure and for the purpose of soothing, and this is normal. A penile erection at this young age is reflexive and not conscious and intentional. It is not a form of priapism.

1. 8. The primary care pediatric nurse practitioner performs an initial well baby exam on a 1-week-old infant who is breastfeeding and who is at birth weight. The mother tells the nurse practitioner that her baby is already sleeping 5 or 6 hours at night. What will the nurse practitioner recommend? a. a. Consultation with a lactation specialist to assess intake b. b. Pumping her breast during the night to maintain milk supply c. c. Supplementing the last feeding of the day with formula d. d. Waking the infant up at least every 3 hours to nurse

ANS: D Newborn infants should nurse every 2 to 3 hours in order to establish a routine and stimulate milk supply. The infant has regained birth weight, which is expected by age 2 weeks, so weight is not a concern and a lactation consultation is not warranted. The infant should continue to nurse during the night, so there is no need for the mother to pump her breasts or to supplement with formula. Using formula from a bottle also causes nipple confusion and should be avoided.

4. A 10-month-old infant has an erythematous, fluctuant, non-draining abscess on the right buttock after 10 days of treatment with amoxicillin for impetigo. What is the next step in managing this infant's care? a. Consultation with a pediatric infectious disease specialist b. Culture of any superficial open surface wounds c. Empiric treatment with clindamycin d. Incision and drainage of the abscess with culture

ANS: D Non-draining, fluctuant abscesses should be incised, drained, and cultured to determine the causative organism. Consultation with an infectious disease specialist is necessary for seriously ill children, those who are immunocompromised, or those who have an increased risk for myocarditis. Superficial wounds should not be cultured because of the chance of sample contamination. Empiric treatment may be considered for severe infection, but many mild abscesses may not need antibiotic therapy after I&D.

17. An adolescent female has grouped vesicles on her oral mucosa. To determine whether these are caused by HSV-1 or HSV-2, the primary care pediatric nurse practitioner will order which test? a. Direct fluorescent antibody test b. Enzyme-linked immunosorbent assay c. Tzanck smear d. Viral culture

ANS: D Oral lesions are possible with both forms of herpesvirus. Viral culture is the gold standard for distinguishing HSV-1 from HSV-2. DFA and ELISA are usually used only with severe forms of infection. Tzanck smear dose not distinguish HSV-1 from HSV-2.

1. 7. The parent of a toddler tells the primary care pediatric nurse practitioner that the family has adopted a plant-based diet and the child is receiving rice and almond milk instead of cow's milk. The nurse practitioner will counsel the parents about a. a. calcium deficiency. b. b. excess caloric intake. c. c. excess fat intake. d. d. protein deficiency.

ANS: D Plant-based milks are low in protein and young children who consume them are at risk for protein deficiency. They are not low in calcium, high in calories, or high in fat.

17. A child whose family has been camping in a region with endemic Lyme disease suffered several tick bites. The parents report removing the ticks but are not able to verify the type or the length of time the ticks were attached. The child is asymptomatic. What is the best course of action? a. Administer a prophylactic single dose of doxycycline. b. Perform serologic testing for IgG or IgM antibodies. c. Prescribe amoxicillin three times daily for 14 to 21 days. d. Teach the parents which signs and symptoms to report.

ANS: D Prophylaxis should not be given if the type of tick or the timeline for attachment cannot be verified

17. A child whose family has been camping in a region with endemic Lyme disease suffered several tick bites. The parents report removing the ticks but are not able to verify the type or the length of time the ticks were attached. The child is asymptomatic. What is the best course of action? a. Administer a prophylactic single dose of doxycycline. b. Perform serologic testing for IgG or IgM antibodies. c. Prescribe amoxicillin three times daily for 14 to 21 days. d. Teach the parents which signs and symptoms to report.

ANS: D Prophylaxis should not be given if the type of tick or the timeline for attachment cannot be verified. however, parents should be encouraged to report signs of Lyme disease if they occur. Prophylaxis is given if the tick is reliably identified as a nymph or adult Ixodes scapularis species. Serologic testing may be performed if symptoms occur. Amoxicillin tid for 2 to 3 weeks is indicated for early localized disease.

1. 17. The primary care pediatric nurse practitioner is reviewing the rheumatology plan of care for a child who is diagnosed with SLE. Besides reinforcing information about prescribed medications, what will the nurse practitioner teach the family to help minimize flaring of episodes? a. a. Have the child rest between activities. b. b. Obtain regular ophthalmology exams. c. c. Participate in low-impact exercises. d. d. Use UVA and UVB sunscreen daily.

ANS: D Sunlight is a known trigger of SLE so patients should be advised to use a UVA and UVB sunscreen both indoors and out. Resting between activities is recommended for children with JIA. Children should participate in low-impact activities, but this does not reduce the number of flares. Ophthalmology exams are recommended for children with JIA.

1. 8. During a well child assessment of an 18-month-old child, the primary care pediatric nurse practitioner observes the child point to a picture of a dog and say, "Want puppy!" The nurse practitioner recognizes this as an example of a. a. holophrastic speech. b. b. receptive speech. c. c. semantic speech. d. d. telegraphic speech.

ANS: D Syntax, or the structure of words in sentences or phrases, is developed in stages between the ages of 8 months and 3.5 years. Telegraphic speech begins at about 18 months of age when children speak in phrases with many words omitted, so that the sentence sounds like a telegram. Holophrastic speech is the use of a single word to express a complete idea. Receptive speech refers to the ability to understand a word without necessarily being able to use the word. Semantics is the understanding that words have specific meanings.

3. A 9-month-old infant with a history of three urinary tract infections is diagnosed with grade II vesicoureteral reflux. Which medication will be prescribed? a. Amoxicillin 10 mg/kg as a single daily dose b. Ceftriaxone IM 50 mg/kg as a single daily dose c. Nitrofurantoin 1-2 mg/kg as a single daily dose d. TMP-SMX; TMP 2 mg/kg as a single daily dose

ANS: D TMP-SMX is a first-line medication for grade II VUR prophylaxis. TMP-SMX: TMP 2 mg/kg as a single daily dose or 5 mg/kg twice/wk (based on TMP component) if older than 1 month.Amoxicillin is a first- line treatment in infants younger than 2 months. It is not necessary to give IM medications. Nitrofurantoin is expensive and poorly tolerated.

12. A 4-year-old child who has previously met developmental milestones is not toiled trained. The primary care pediatric nurse practitioner notes decreased reflexes in the lower extremities and observe a dimple above the gluteal cleft. Which diagnosis may be considered for this child? a. Arnold-Chiari malformation b. Reye syndrome c. Spina bifida cystica d. Tethered cord

ANS: D Tethered cord occurs when the caudal end of the spinal cord, causing abnormal stretching and damage to nerve cells, fibers, and blood vessels. This can cause symptoms of neurologic deterioration such as incontinence of bladder and bowel and loss of reflexes and sensation in the legs. Arnold-Chiari malformation involves a downward herniation of the caudal end of the cerebellar vermis, which can cause brainstem and upper cervical cord compression. Reye syndrome involves swelling in the brain and signs of increased intracranial pressure. Spina bifida cystica is a myelomeningocele, with symptoms present at birth.

1. 3. The parent of a 12-month-old infant asks the primary care pediatric nurse practitioner why 2% cow's milk is recommended instead of whole milk. What will the nurse practitioner tell this parent? a. a. Whole milk is usually not fortified with vitamin D. b. b. 2% milk is higher in essential proteins and minerals. c. c. Young children don't need the extra calories found in whole milk. d. d. Younger children need a limited amount of fats.

ANS: D The AAP recommends transitioning to unflavored 2% or fat-free cow's milk in the second year of life. Whole milk contains vitamin D and contains similar amounts of proteins and minerals. Children can get calories from other sources

1. 7. The primary care pediatric nurse practitioner performs a physical examination on a 9-month-old infant and notes two central incisors on the lower gums. The parent states that the infant nurses, takes solid foods three times daily, and occasionally takes water from a cup. What will the pediatric nurse practitioner counsel the parent to promote optimum dental health? a. a. To begin brushing the infant's teeth with toothpaste b. b. To consider weaning the infant from breastfeeding c. c. To discontinue giving fluoride supplements d. d. To make an appointment for an initial dental examination

ANS: D The American Academy of Pediatric Dentistry recommends a first dental examination at the time of eruption of the first tooth and no later than 12 months old. Parents should be counseled to clean the infant's teeth but with water only. Weaning from breastfeeding is not indicated, although mothers should not let the infant nurse while sleeping to prevent milk from bathing the teeth. Fluoride supplements should not be discontinued.

8. The primary care pediatric nurse practitioner performs a physical examination on a 9-month-old infant and notes two central incisors on the lower gums. The parent states that the infant nurses, takes solid foods three times daily, and occasionally takes water from a cup. What will the pediatric nurse practitioner counsel the parent to promote optimum dental health? a. To begin brushing the infant's teeth with toothpaste b. To consider weaning the infant from breastfeeding c. To discontinue giving fluoride supplements d. To make an appointment for an initial dental examination

ANS: D The American Academy of Pediatric Dentistry recommends a first dental examination at the time of eruption of the first tooth and no later than 12 months old. Parents should be counseled to clean the infant's teeth but with water only. Weaning from breastfeeding is not indicated, although mothers should not let the infant nurse while sleeping to prevent milk from bathing the teeth. Fluoride supplements should not be discontinued.

8. A child is in the clinic for evaluation of an asthma action plan. The primary care pediatric nurse practitioner notes that the child's last visit was for a pre-kindergarten physical and observes that the child is extremely anxious. What will the nurse practitioner do initially? a. Ask the child's parent why the child is so anxious. b. Perform a physical assessment to rule out shortness of breath. c. Reassure the child that there is nothing to be afraid of. d. Review the purpose of this visit and any anticipated procedures.

ANS: D The PNP should remember that young children are learning "scripts" for health care visits and may be stressed when recalling previous visits, especially if those involved immunizations. The PNP should explain the purpose and any anticipated procedures for this visit to help put the child at ease.

13. The mother of two school-age children tells the primary care pediatric nurse practitioner that she and the children's father are divorcing and asks for advice to help the children cope with the situation. The nurse practitioner will counsel her to a. allow visitation only on weekends. b. maintain her own social life. c. notify the children's teachers. d. use a social support network.

ANS: D The availability of a social support system is a key factor in coping with divorce. Visitation patterns should be consistent and collaborative but not necessarily only on certain days. Although the mother should eventually maintain a social life, it is not a key early on to helping children cope with a divorce. Notifying teachers does not necessarily help children to cope.

4. The primary care pediatric nurse practitioner is assessing the health literacy of the parent of a toddler. Which tool will the nurse practitioner use to estimate reading level? a. Flesch-Kincaid Readability Test b. Gunning Fog Index c. Number of children's books in the home d. SMOG

ANS: D The best tool to estimate reading level is the SMOG. The Gunning Fog Index and the Flesch-Kincaid Readability Test are used as well but are not considered the best indices. The number of children's books in the home can be asked as a screen but is not a reliable estimate of true literacy.

4. After 14 days of treatment with amoxicillin 45 mg/kg/day for acute rhinosinusitis, a child continues to have mucopurulent nasal discharge along with induration, swelling, and erythema of both eyelids. What is the next course of treatment? a. Amoxicillin 80 mg/kg/day for 14 days b. Amoxicillin-clavulanate for 10 to 14 days c. Antibiotic ophthalmic drops for 5 to 7 days d. Referral to a pediatric otolaryngologist

ANS: D The child has symptoms consistent with periorbital or preseptal inflammation and needs a referral to an otolaryngologist or infectious disease specialist. For uncomplicated persistent rhinosinusitis, amoxicillin- clavulanate should be prescribed. These symptoms are not consistent with conjunctivitis, so antibiotic eyedrops are not indicated. Although increasing the dose of amoxicillin may be part of the treatment, referral to a pediatric otolaryngologist is a priority.

17. The primary care pediatric nurse practitioner evaluates a child who awoke with a sore throat and high fever after a nap. The child appears anxious and is sitting on the parent's lap with the neck hyperextended. The physical exam reveals stridor, drooling, nasal flaring, and retractions. What will the nurse practitioner do next? a. Administer a broad-spectrum intravenous antibiotic. b. Obtain blood and throat cultures and start antibiotic therapy. c. Send the child to radiology for a lateral neck radiograph. d. Transport the child to the hospital via emergency medical services.

ANS: D The child has symptoms of epiglottitis and should be transported immediately for emergency treatment via ambulance. All of the other options may be initiated at the hospital once the diagnosis is more certain. If the possibility of epiglottitis is thought to be remote, a lateral neck radiograph may be obtained prior to visualizing the throat. If epiglottitis is suspected, visualizing the throat is contraindicated.

13. The primary care pediatric nurse practitioner observes a tender, swollen red furuncle on the upper lid margin of a child's eye. What treatment will the nurse practitioner recommend? a. Culture of the lesion to determine causative organism b. Referral to ophthalmology for incision and drainage c. Topical steroid medication d. Warm, moist compresses 3 to 4 times daily

ANS: D The child has symptoms of hordeolum, or stye. Although these often rupture spontaneously, warm, moist compresses may hasten this process. It is not necessary to culture the lesion unless symptoms do not resolve. Referral to ophthalmology is made if the hordeolum does not rupture on its own. Steroids are not indicated.

2. The primary care pediatric nurse practitioner conducts a well baby exam on an infant and notes mild gross motor delays but no delays in other areas. Which initial course of action will the nurse practitioner recommend? a. Consult a developmental specialist for a more complete evaluation. b. Prepare the parents for a potentially serious developmental disorder. c. Refer the infant to an early intervention program for physical therapy. d. Teach the parents to provide exercises to encourage motor development.

ANS: D The child who has mild delays in only one area may be managed initially by having the parent provide appropriate exercises. If this is not effective, or if delays become more severe, referrals for evaluation or early intervention services are warranted. A mild delay does not necessarily signal a serious disorder, so this action is not indicated.

6. The primary care pediatric nurse practitioner is providing anticipatory guidance to the parent of a school-age boy. The parent expresses concerns that the child prefers to play with dolls, is worried that the child will be a homosexual, and asks what can be done to prevent this from happening. What will the nurse practitioner tell this parent? a. Homosexual identity formation cannot be predicted by early childhood behavior. b. Masculinizing boys from an early age helps to determine heterosexual orientation. c. Sexual orientation identification begins late in adolescence and not in childhood. d. The development of sexual orientation is generally a multifaceted process.

ANS: D The etiology and age of preferred sexual orientation is unknown, and the sequential developmental signs are debated. The development of sexual orientation is most likely multifaceted and cannot be predicted by one phenomenon, such as playing with dolls alone. Early childhood behavior can predict homosexual orientation as girls may feel "unfeminine" and boys may exhibit feminine tendencies. It is clear that psychosocial components and parenting do not cause or prevent homosexuality. 7. The primary care pediatric nurse practitioner is performing a well child examination on a 3-year-old. The child's parent reports that the child has recently begun masturbating. What will the nurse practitioner counsel this parent? a. To allow the behavior whenever it occurs, since it is normal b. To discuss sexuality with the child c. To explore whether the child is being abused d. To teach the child about privacy and hand hygiene ANS: D Masturbation is normal at this age and children do this because it is pleasurable. Parents should be taught to discuss privacy and hygiene with the child and to encourage the child to limit the activity to a private place. At this age, the behavior is not associated with sexual fantasies, so a discussion of sexuality is not warranted. Masturbation at this age is common and is not usually an indication of abuse.

6. During a well child examination of a 2-year-old child, the primary care pediatric nurse practitioner palpates a unilateral, smooth, firm abdominal mass which does not cross the midline. What is the next course of action that? a. Order a CT scan of the chest, abdomen, and pelvis. b. Perform urinalysis, CBC, and renal function tests. c. Reevaluate the mass in 1 to 2 weeks. d. Refer the child to an oncologist immediately.

ANS: D The finding is consistent with Wilms tumor, and referral, diagnosis, and treatment are urgent. Palpating a mass too vigorously could lead to the rupture of a large tumor into the peritoneal cavity so care should be taking in conducting the physical examination. The other tests may be ordered by the oncology team. Treatment and diagnosis must occur immediately.

9. A 5-month-old infant who has a 3-day history of cough and rhinorrhea has developed symptoms of respiratory distress with audible expiratory wheezes and increased coughing. The infant's immunizations are up-to-date. The physical exam reveals a respiratory rate of 50 breaths per minute, coarse expiratory wheezing, and prolonged expiration. An oxygen saturation is 96% on room air. What is the recommended treatment for this infant? a. Administer a trial of bronchodilators. b. Obtain a viral culture of nasal washings. c. Order an oral corticosteroid medication. d. Recommend increased fluids and close follow-up.

ANS: D The infant has bronchiolitis and is stable. Increasing fluids and following up closely are indicated as long as oxygen saturations and hydration are normal. Bronchodilator trials are not recommended because of the risk of adverse effects and questionable efficacy. Viral cultures are performed if hospitalization is necessary or when symptoms are severe. Corticosteroid medications are not indicated.

6. During a well child assessment of a 13-year-old male, the primary care pediatric nurse practitioner notes small testicles and pubic and axillary hair. To further evaluate these findings, the nurse practitioner will ask the patient about a. alcohol and tobacco use. b. changes in voice. c. increase in height and weight. d. participation in sports.

ANS: D The initial sign of puberty in males is testicular enlargement. If this does not precede other changes, the PNP should consider whether the boy is taking exogenous anabolic steroids, common among those who wish to improve athletic ability. These findings are not concerning for alcohol or tobacco use. Voice changes and rapid growth may occur with pubic hair development, but the primary concern is anabolic steroid use.

5. The primary care pediatric nurse practitioner is performing a well child examination on a fussy toddler who has red hair. The child's parent tells the toddler to stop being fussy and says, "red hair gives him such a temper." Which common error that erodes self-esteem is this? a. Dwelling on negatives b. Expecting too much c. Negating the child's feelings d. Stereotyping and typecasting

ANS: D The parent is typecasting the child by associating having a temper with the trait of red hair, which can limit his sense of possibilities. Dwelling on negatives describes making critical remarks instead of positive remarks. A parent who expects too much creates pressures for behaviors that are beyond the reach of the child and makes the child feel inadequate. Negating feelings is a rejection of the child's emotions.

7. A 3-month-old infant who was previously healthy now has a persistent cough, bilateral lung crackles, and poor appetite. The primary care pediatric nurse practitioner auscultates a grade III/VI, low- pitched, holosystolic murmur over the left lower sternal border and palpates the liver at one centimeter below the ribs. What diagnosis is likely? a. Atrial septal defect b. Coarctation of the aorta c. Patent ductus arteriosis d. Ventricular septal defect

ANS: D The symptoms above are characteristic of a VSD and may not present at birth but appear later as CHF becomes more pronounced. An ASD typically does not have a murmur until the child is 2 or 3 years old, but the provider can often hear a split S2 sound. Coarctation of the aorta may cause a systolic ejection murmur. A PDA has a characteristic machinery-like murmur.

11. An adolescent female who is sexually active and who has not had the HPV vaccine asks if she may have it. What will the primary care pediatric nurse practitioner tell her? a. Getting the vaccine now will still protect her from HPV oncogenic types even if already exposed b. Receiving the HPV vaccine series will replace the need for regular cervical cancer screening c. She will need to have Papanicolaou and pregnancy screening prior to receiving the vaccine d. The vaccine will not protect her from any HPV oncogenic types acquired previously

ANS: D There is no protection for HPV oncogenic types acquired prior to the vaccine. The vaccine does not eliminate the need for cervical cancer screening. It is not necessary to perform PAP or pregnancy testing prior to vaccine administration.

12. An adolescent male comes to the clinic reporting unilateral scrotal pain, nausea, and vomiting that began that morning. The primary care pediatric nurse practitioner palpates a painful, swollen testis and elicits increased pain with slight elevation of the testis (a negative Phren's sign). What will the nurse practitioner do? a. Administer IM ceftriaxone and prescribe doxycycline twice daily for 10 days. b. Encourage bed rest, scrotal support, and ice packs to the scrotum as tolerated. c. Prescribe NSAIDs, limited activities, and warm compresses to the scrotum. d. Refer the adolescent immediately to a pediatric urologist or surgeon.

ANS: D These symptoms are indicative of testicular torsion. Testicular torsion causes a sudden onset of unilateral pain and is distinguished from epididymitis when elevation of the scrotum causes an increase in pain (Phren's sign). It is a surgical emergency and should warrant immediate referral. Epididymitis is caused by infection and requires antibiotics, bed rest, scrotal support, and ice packs. Testicular appendix torsion is self-limited and can be managed with NSAIDs, bed rest or limited activities, and warm compresses.

19. A preschool-age child is brought to clinic for evaluation of a rash. The primary care pediatric nurse practitioner notes an intense red eruption on the child's cheeks and circumoral pallor. What will the nurse practitioner tell the parents about this rash? a. This rash may be a prodromal sign of rubella or roseola. b. The child will need immunization boosters to prevent serious disease. c. This is a benign rash with no known serious complications. d. Expect a lacy, maculopapular rash to develop on the trunk and extremities.

ANS: D This "slapped cheek" rash is consistent with fifth disease, or erythema infectiosum, and will be followed by a lacy, maculopapular all-over rash. It is not a prodrome of rubella or roseola, and immunizations are not indicated. Although it is mostly benign, there can be serious sequelae, especially for pregnant women.

19. An adolescent who recently spent time in a hot tub while on vacation has discrete, erythematous 1- to 2-mm papules that are centered around hair follicles on the thighs, upper arms, and buttocks. How will the primary care pediatric nurse practitioner manage this condition? a. Culture the lesions and treat with appropriate IM antibiotics. b. Hospitalize for incision and drainage and intravenous antibiotics. c. Order an antistaphylococcal beta-lactamase-resistant antibiotic. d. Prescribe topical keratolytics and topical antibiotics.

ANS: D This adolescent has hot-tub folliculitis that is superficial at this point and may be treated with topical keratolytics and topical antibiotics. Culture is indicated if the lesions are resistant to treatment. IV and oral antibiotics and I&D are indicated for more severe episodes.

3. A school-age child is hit in the face with a baseball bat and reports pain in one eye. The primary care pediatric nurse practitioner is able to see a dark red fluid level between the cornea and iris on gross examination, but the child resists any exam with a light. Which action is correct? a. Administer an oral analgesic medication. b. Apply a Fox shield and reevaluate the eye in 24 hours. c. Instill anesthetic eyedrops into the affected eye. d. Refer the child immediately to an ophthalmologist.

ANS: D This child has a traumatic injury with hyphema to the eye, and an ophthalmologist must examine the eye to rule out orbital hematoma or retinal detachment. Any further attempt to examine the child may result in further injury. A Fox shield is used once more serious injury is excluded.

13. A 9-year-old girl has a history of frequent vomiting and her mother has frequent migraine headaches. The child has recently begun having more frequent and prolonged episodes accompanied by headaches. An exam reveals abnormal eye movements and mild ataxia. What is the correct action? a. Begin using an anti-migraine medication to prevent headaches. b. Prescribe ondansetron and lorazepam to help manage symptoms. c. Reassure the parent that this is expected with cyclic vomiting syndrome. d. Refer to a pediatric gastroenterologist for further workup.

ANS: D This child has an abnormal neurologic examination, which is a red flag warranting referral for further workup for children with cyclic vomiting syndrome. Anti-migraine medications are used in children over age 12 years and therefore should not be used for this patient. Ondansetron and lorazepam may be useful for unrelenting nausea and poor sleep, but this child needs to be referred to evaluate neurologic symptoms. These signs are not expected.

16. A 2-year-old child is brought to the clinic after developing a hoarse, bark-like cough during the night with "trouble catching his breath" according to the parent. The history reveals a 2 day history of low- grade fever and upper respiratory symptoms. On exam, the child has a respiratory rate of 40 breaths per minute, occasional stridor when crying, and a temperature of 101.3°F. What is the next step in treatment for this child? a. Administer intramuscular dexamethasone. b. Admit the child for inpatient hospitalization. c. Give the child a racemic epinephrine treatment in the office. d. Prescribe oral dexamethasone for 2 days.

ANS: D This child has croup with milder symptoms and may be managed at home with oral steroids. IM steroids are given to children who are vomiting. Inpatient admission is considered for children with respiratory distress (rates between 70 and 90 breaths per minute, severe retractions, and stridor at rest). Racemic epinephrine is useful for more severe symptoms, especially for stridor, but is done in the ED or hospital and should always be combined with a steroid.

9. A 4-year-old child has clusters of small, clear, tense vesicles with an erythematous base on one side of the mouth along the vermillion border, which are causing discomfort and difficulty eating. What will the primary care pediatric nurse practitioner recommend as treatment? a. Mupirocin ointment applied to lesions 3 times daily b. Oral acyclovir 20 to 40 mg/kg/dose for 7 to 10 days c. Topical acyclovir applied to lesions 4 times daily d. Topical diphenhydramine and magnesium hydroxide

ANS: D This child has lesions consistent with HSV-1. Topical anesthetics may be used for comfort and may be applied with cotton-tipped swabs. Mupirocin ointment is used for secondary bacterial infection. Oral acyclovir is used in more severe cases and most often with HSV-2. Topical acyclovir is useful for initial genital herpes infections.

9. A toddler is seen in the clinic after a 2-day history of intermittent vomiting and diarrhea. An assessment reveals an irritable child with dry mucous membranes, 3-second capillary refill, 2- second recoil of skin, mild tachycardia and tachypnea, and cool hands and feet. The child has had two wet diapers in the past 24 hours. What will the primary care pediatric nurse practitioner recommend? a. Anti-diarrheal medication and clear fluids for 24 hours b. Bolus of IV normal saline in the clinic until improvement c. Hospital admission for IV rehydration and oral fluids d. Oral rehydration solution with follow-up in 24 hours

ANS: D This child has mild to moderate dehydration, according to vital signs and symptoms, and may be managed with oral rehydration solution with good follow-up. Anti-diarrheal medications are generally not useful, but antiemetics may be used. It is not necessary to administer IV fluids or to hospitalize unless more severe dehydration occurs.

13. A child with a recent history of URI reports tingling and pain in one ear followed by sagging of one side of the face. The primary care pediatric nurse practitioner observes that the child cannot close the eye or mouth on the affected side but does not elicit limb weakness on that side. What will the nurse practitioner do? a. Initiate a short course of antibiotic therapy. b. Perform diagnostic testing to rule out serious causes. c. Prepare the parents for lifelong complications. d. Prescribe oral prednisone 1 mg/kg/day initially.

ANS: D This child has symptoms and a history consistent with Bell's palsy. Oral prednisone is given to reduce inflammation causing paralysis for 1 week with a 1 week taper. Antibiotics are not indicated, since a viral illness often precipitates symptoms. Diagnostic testing is not indicated unless symptoms persist longer than 6 weeks or if other neurologic symptoms occur. Approximately 85% of children recover completely without facial weakness.

1. A child who had GABHS 2 weeks prior is in the clinic with periorbital edema, dyspnea, and elevated blood pressure. A urinalysis reveals tea-colored urine with hematuria and mild proteinuria. What will the primary care pediatric nurse practitioner do to manage this condition? a. Prescribe a 10- to 14-day course of high-dose amoxicillin. b. Prescribe high-dose steroids in consultation with a nephrologist. c. Reassure the parents that this condition will resolve spontaneously. d. Refer the child to a pediatric nephrologist for hospitalization.

ANS: D This child has symptoms of post-streptococcal glomerulonephritis and signs indicating a need for hospitalization: elevated BP, edema, and dyspnea. The PNP should refer the child to a nephrologist for hospital admission and care. Amoxicillin is not indicated; this condition is an immunologic response to GABHS and not an infection. Steroids are not effective in treating this disease. Although the condition usually does self-resolve, the child needs hospitalization for close monitoring and follow-up.

7. A school-age child comes to the clinic for evaluation of excessive bruising. The primary care pediatric nurse practitioner notes a history of an upper respiratory infection 2 weeks prior. The physical exam is negative for hepatosplenomegaly and lymphadenopathy. Blood work reveals a platelet count of 60,000/mm3 with normal PT and aPTT. How will the nurse practitioner manage this child's condition? a. Admit to the hospital for IVIG therapy. b. Begin a short course of corticosteroid therapy. c. Refer to a pediatric hematologist. d. Teach to avoid NSAIDs and contact sports.

ANS: D This child has symptoms, a history, and lab work that indicate idiopathic thrombocytopenic purpura. Since platelets are greater than 20,000/mm3, management without specific therapy may be done on an outpatient basis by teaching the family to avoid things that contribute to bleeding. IVIG therapy is used for children with active, severe bleeding. Corticosteroids are given for platelet counts less than 20,000/mm3. Referral to a hematologist is necessary for more severe cases.

1. The parent of a 3-year-old child tells the primary care pediatric nurse practitioner that after falling asleep in the living room and being awakened to go to bed one evening, the child appeared confused and disoriented for a period of time. What will the nurse practitioner counsel this parent? a. That if this occurs again, to question the child about nightmares b. That this is a sign of sleep walking and could be dangerous c. That this is a type of sleep terror which will resolve over time d. That this is probably a benign, temporary type of a sleep disorder

ANS: D This child most likely exhibits confusional arousal, which occurs when a child is awakened from a deep sleep during the first part of the night. It is most likely benign and temporary, usually diminishing by age 5 years. It is not a sign of nightmares or night terrors. It may be the start of sleep walking but is less likely.

19. The parent of a preschool-age child reports that the child often appears anxious and nervous and that this is associated occasionally with a rapid heart rate and tremors. What is the best type of referral that the primary care pediatric nurse practitioner could recommend? a. Cognitive-behavioral therapy b. Family therapy c. Medication therapy d. Play therapy

ANS: D Toddlers and preschoolers who are experiencing anxiety often respond to play therapy. School-age children and adolescents benefit from cognitive-behavioral therapy. Family therapy may be a secondary choice. Medications are not indicated as first-line treatment for anxiety.

2. The primary care pediatric nurse practitioner is counseling the parents of a toddler about sleep. The parents report that the toddler has recently begun resisting sleep and is often more irritable during the day. What will the nurse practitioner recommend? a. Co-sleeping with the child to help alleviate possible nighttime fears b. Referral to a sleep disorders clinic for evaluation of sleep-disordered breathing c. Reintroducing a second, morning nap time to compensate for lost sleep d. Understanding that sleep resistance is a common developmental problem

ANS: D Toddlers may develop sleep resistance as a normal part of their behaviors associated with increased autonomy or may have nighttime fears or night terrors. Parents should understand that this is common and transient. Co-sleeping may be practiced in some cultures but is not recommended. It is not necessary to refer to a sleep disorders clinic unless there are specific symptoms, such as snoring or restless sleep or sleepiness in spite of adequate sleep.

1. 6. The primary care pediatric nurse practitioner is preparing to close a laceration on a child's forehead using topical skin adhesive. What is the correct way to apply this product? a. a. Apply the adhesive between the wound margins and then hold the edges together. b. b. Apply the adhesive to the wound and then secure the edges with surgical tape. c. c. Have the child remain still for 15 to 20 minutes after the adhesive is applied. d. d. Hold the wound edges together and apply the adhesive on top of the skin.

ANS: D Topical adhesive is applied by holding the wound edges together (approximating the wound edges) and then applying the adhesive on top, often requiring two or three applications of the adhesive but allowing skin cooling between applications. The adhesive should not be applied between the wound margins or in the wound. Surgical tape and bandages are not used with topical adhesive. It is not necessary for the child to remain still after the adhesive is applied, since it dries quickly.

26. A child has an area of inflammation on the neck that began after wearing a hand-knot woolen sweater. On examination, the skin appears chafed with mild erythematous patches. The lesions are not pruritic. What is an appropriate initial treatment? a. Application of a lanolin-based emollient b. Burow solution soaks and cool compresses c. Oral antihistamines given 4 times daily d. Topical corticosteroids applied 2 to 3 times daily

ANS: D Topical corticosteroids are useful for contact dermatitis. Lanolin-based emollients are contraindicated when inflammation is present. Burow solution soaks are useful for vesicular rashes. Oral antihistamines are not indicated unless itching and scratching occur.

1. 5. Which label on a CAM therapy product verifies that a product meets standards for contamination, adulteration, manufacturing processes, and pharmacologic properties? a. a. Generally Recognized as Safe (GRAS) b. b. National Sanitation Foundation International (NSF) c. c. Natural Products Association (NPA) d. d. United States Pharmacopeia (USP)

ANS: D USP labeling verifies meeting of standards for contamination, adulteration, manufacturing processes, and pharmacologic properties. GRAS labeling recognizes a product as being generally safe for its intended purpose. NSF labeling sets standards for cleanliness, maintenance, and documented quality checks. NPA labeling ensures ingredient quality and purity.

4. To evaluate brain tissue disorders in infants, which test is useful? a. Computerized tomography b. Head radiographs c. Magnetic resonance imaging d. Ultrasonography

ANS: D Ultrasonography is used to evaluate brain tissue in infants. CT scans expose patients to high levels of radiation, so they are not used unless indicated. Radiographs have relatively diagnostic value for the neurologic system. Magnetic resonance imaging is useful but is expensive and usually requires sedation.

10. A school-age child has several annular lesions on the abdomen characterized by central clearing with scaly, red borders. What is the first step in managing this condition? a. Fluoresce the lesions with a Wood's lamp. b. Obtain fungal cultures of the lesions. c. Perform KOH-treated scrapings of the lesion borders. d. Treat empirically with antifungal cream.

ANS: D Unless the diagnosis is questionable, or if treatment failure occurs, tinea corporis is treated empirically with topical antifungal creams; therefore, it is not necessary to fluoresce the lesions, culture the lesions, or complete KOH testing of scrapings as an initial management step.

13. A child has an acute infection causing lower airway obstruction. Which initial symptom is expected in this child? a. Atelectasis b. Barrel chest c. Over-inflation d. Wheezing

ANS: D Wheezing is the principal sound patients make if the obstruction allows enough air to pass through the narrowed lumen when lower airway obstruction occurs. Eventually, over-inflation and atelectasis occur. Barrel chest is the result of chronic over-inflation.

3. The parent of a school-age child who has asthma tells the primary care pediatric nurse practitioner that the child often comes home from school with severe wheezing after gym class and needs to use his metered-dose inhaler right away. What will the nurse practitioner do? a. Recommend that the child go to the school nurse when symptoms start. b. Review the child's asthma action plan and possibly increase his steroid dose. c. Suggest asking the school to excuse the child from gym class. d. Write the prescription for two metered-dose inhalers with spacers.

ANS: D When children have to take a medication at school or day care, the PNP should dispense two units of the medication so that one can remain at school and one at home to avoid missed doses. The school nurse will not be able to order a medication that the child does not have available. The child is missing his rescue medication and just needs access to his inhaler. It is not necessary to excuse the child from gym class if his symptoms can be controlled.

5. The primary care pediatric nurse practitioner applies fluorescein stain to a child's eye. When examining the eye with a cobalt blue filter light, the entire cornea appears cloudy. What does this indicate? a. The cornea has not been damaged. b. There is too little stain on the cornea. c. There is damage to the cornea. d. There is too much stain on the cornea.

ANS: D When fluorescein stain is applied and the entire cornea appears cloudy, it means that there is too much of the stain. Damaged areas of the cornea should appear greenish after staining with fluorescein dye.

7. The parent of an infant asks about using a probiotic medication. What will the primary care pediatric nurse practitioner tell this parent? a. Probiotic medications have demonstrated efficacy in treating colic. b. Probiotics are not safe to use to treat infants who have colic. c. There are no studies showing usefulness of probiotics to manage colic. d. There is no conclusive evidence about using probiotics to treat colic.

ANS: D While small studies have shown promise in treating colic in infants, the research is contradictory and there is no conclusive evidence about effectiveness. There is no evidence that probiotics are not safe.

12. A preschool-age child who attends day care has a 2-day history of matted eyelids in the morning and burning and itching of the eyes. The primary care pediatric nurse practitioner notes yellow-green purulent discharge from both eyes, conjunctival erythema, and mild URI symptoms. Which action is correct? a. Culture the conjunctival discharge. b. Observe the child for several days. c. Order an oral antibiotic medication. d. Prescribe topical antibiotic drops.

ANS: D Young children with bacterial conjunctivitis may be treated with topical antibiotic drops. Culturing the eyes is not necessary unless there is no improvement. While most cases of bacterial conjunctivitis are self-limiting, using a topical antibiotic will hasten the return to day care. Oral antibiotics are not indicated.

5. During a well child examination of a 6-year-old girl, the primary care pediatric nurse practitioner notes that the child becomes embarrassed and resists taking off her underwear for the exam. What should the nurse practitioner infer from this observation? a. The child has been sexually molested. b. The child is feeling violated by the examiner. c. The parent is exhibiting regressive behavior. d. This is a normal reaction in a child of this age.

ANS: D Young school-age children can be extremely modest and embarrassed and resist taking off their clothes for an examiner. Since this is normal, it does not indicate a history of sexual abuse unless other signs are present. Older school-age children more commonly feel violated during an exam, not younger children. This response of increased modesty is age-appropriate and not regressive.


Ensembles d'études connexes

Module 6: Home Safety & the Aging Adult: Elderly Client's Home

View Set

Introduction to Human Resource Management

View Set

Muscle, Origin, Insertion, Action, Blood, Nerve of Brachium Musculature

View Set

Chapter 49: Drugs Used to Treat Anemias (COMBINED)

View Set

Unit 2: Cell Structure and Function

View Set

Personal Career Development Ch. 2

View Set

KINE: 3000 Early Motor Development Ch.6

View Set

CHEM 3375 Learning Curves Chapters: 1,2,3,4,5,6,7,10

View Set

Systems Analysis and Design- Ch 3

View Set